Anda di halaman 1dari 102

PAKET 1

PAKET TES KEMAMPUAN DASAR UMUM

1
WAKTU : 100 MENIT
JUMLAH SOAL : 60
MATEMATIKA DASAR

MATEMATIKA DASAR

1. Bentuk sederhana dari √7 + √48 adalah 5. Persamaan garis yang melalui titik potong
... . garis 4x + 7y – 15 = 0 dan 14y = 9x – 4
serta tegak lurus pada garis 21x + 5y = 3
(A) √8 + √7
adalah ... .
(B) √7 + √6
(A) 21x – 5y = 3
(C) √6 + 1 (B) 11x – 21y = 5
(D) √5 + √2 (C) 5x – 2y = –11
(E) √4 + √3 (D) 5x + 21y = –11
(E) 5x – 21y = 11
2. Bentuk sederhana dari : (x6y3)2/3 adalah ...
. 6. Nilai a agar persamaan kuadrat x2 – 8x +
(A) y 2a = 0 mempunyai dua akar yang
(B) x berlainan dan positif adalah ... .
(C) xy2
(D) (x2y)2 (A) a>0
(E) x2y2 (B) a<8
(C) 0<a<8
3. Jika x memenuhi 2log 3log (x + 2) = 1 dan (D) a>8
y memenuhi (alog (3y – 1)) (2log a) = 3 (E) a<0
maka nilai x + y adalah ... .
(A) 16 7. Jika {x ϵ R | a < x < b} adalah himpunan
(B) 13 penyelesaian dari pertidaksamaan :
(C) 10 (𝑥 − 1)2 + √(𝑥 − 1)2 < 6 maka nilai a + b
(D) 9 adalah ... .
(E) 4 (A) 4
(B) 2
4. Di berikan a dan b bilangan real dengan a (C) 1
𝑎
> 1 dan b > 1. Jika ab = ab dan 𝑏 = a3b, (D) –2
maka nilai a adalah ... . (E) –4
(A) 0
(B) 1 8. Nilai maksimum dari 2x + y yang
(C) 3 memenuhi x – y + 3 ≥ 0, 3x + 2y – 6 ≥ 0,
(D) 4 x ≥ 0, y ≥ 0 adalah ... .
(E) 5 (A) 0
(B) 3
(C) 4
(D) 5
(E) 6

bimbelocean.com
UTUL UGM 67
PAKET 1
9. Diberikan segiempat ABCD seperti pada
𝑎 3 2
gambar. (D) −√ √𝑥 2 − 1

𝑎 3 2
(E) −√ √𝑥 2 − 1

cos 𝑥−sin 𝑥
14. Jika 𝑓 (𝑥 ) = cos 𝑥+sin 𝑥 dengan cos x + sin x
≠ 0 maka f’(x) = ... .
(A) 1 – (f(x))2
Luas segiempat ABCD adalah ... . (B) –1 + (f(x))2
65
(A) 60 + √3 cm2 (C) –(1 + (f(x))2)
2
(D) 1 + (f(x))2
(B) 30 + 136√3 cm2
(E) (f(x))2
(C) 30 + 65√3 cm2
65
(D) 30 +
2
√3 cm2 15. Suku ke-5 dari barisan geometri adalah
65
(E) 10 + √3 cm 2 243, hasil bagi suku ke-9 dengan ke-6
2
adalah 27. Suku ke-2 adalah ... .
𝑥2 𝑥2 (A) 3
10. lim ( − ) = ... . (B) 5
𝑥→∞ 2𝑥−1 2𝑥+1
(A) 2 (C) 7
(B) 1 (D) 9
1
(C) (E) 12
2
1
(D) 4 16. Diketahui deret aritmetika dengan beda 1.
(E) 0 Jika jumlah pangkat tiga dari tiga suku
pertamanya adalah 18 lebih besar dari 3
1 1
11. lim (𝑥 − 𝑥 cos 𝑥) = ... . kali pangkat tiga dari suku ke-2 maka
𝑥→0
(A) –1 jumlah tiga suku pertamanya adalah ... .
(B) 1 A. 6
(C) 2 B. 9
1 C. 12
(D)
2 D. 15
(E) 1 E. 18

12. Jika fungsi y = x3 – 3x + 3 didefinisikan 17. Diketahui kejadian A dan kejadian B


3
pada 5 , maka nilai terbesar dari y adalah dua kejadian yang saling bebas.
2≤𝑥≤
2 1
adalah ... . Jika diketahui P(A) = dan P(AcUBc) = 7
3
(A) 3 maka P(Ac∩Bc) = ... .
1 (A) 0
(B) 4
8 2
(C) 5 (B) 9
1 2
(D) 11 8 (C) 3
1 7
(E) 15 8 (D) 9
(E) 1
3
2 2 2
𝑑𝑦
13. Jika 𝑦 = (𝑎3 − 𝑥 3 ) , maka adalah ... . 18. Sumbangan rata-rata warga untuk korban
𝑑𝑥
(A) –1 bencana alam adalah Rp 40.000,00. Jika
33 sumbangan dari seorang warga bernama
(B) − 2 √𝑎2 − 𝑥 2
Ali digabungkan dalam kelompok warga
𝑎2
(C) – √𝑥 2 − 1 tersebut,, maka sumbangan rata-rata 26

bimbelocean.com
68
UTUL UGM
PAKET 1

warga sekarang menjadi Rp 41.000,00.


Hal ini berarti sumbangan Ali sebesar ... .
(A) Rp 40.000,00
(B) Rp 57.000,00
(C) Rp 65.500,00
(D) Rp 66.000,00
(E) Rp 92.000,00

19. Apabila x dan y memenuhi persamaan


1 −2 𝑥 −1
matriks [ ] [ ] = [ ] maka x + y
−1 3 𝑦 2
= ... .
(A) 1
(B) 2
(C) 3
(D) 4
(E) 5

20. Diketahui deret geometri dengan Un =


(xlog 3)n, x > 0, x ≠ 1. Jika jumlah tak
hingga deret tersebut ada, maka x harus
memnuhi syarat ... .
1
(A) 𝑥 ≤ 3 atau 𝑥 ≥ 3
1
(B) <𝑥<3
3
1
(C) 𝑥 > 3 atau 0 < 𝑥 < 3
1
(D) 𝑥 ≥ 3 atau 0 < 𝑥 ≤ 3
1
(E) 𝑥 < 3 atau 𝑥 > 3

BAHASA INDONESIA

Bacalah teks berikut dengan cermat, perubahan iklim di Montreal yang dihadiri 189
kemudian jawablah pertanyaan nomor 21 negara. Air pasang yang tinggi karena badai
sampai dengan nomor 23. menjadi semakin besar dalam tahun-tahun
terakhir dan menyebabkan Lateu tidak lagi
Naiknya permukaan air laut telah berpenghuni karena sering disapu banjir
mendorong 100 orang penduduk sebuah antara 4 hingga 5 kali dalam setahun. "Kami
pulau di Samudera Pasifik berpindah ke lokasi melihat pasang yang tinggi menyapu pulau-
yang lebih tinggi. Ini adaiah dampak pulau di sana," katanya. Program Lingkungan
pemanasan global yang jelas terlihat PBB (UNEP) menyatakan bahwa wilayah
kerugiannya bagi sebuah komunitas. Pohon- Lateu menjadi salah satu –kalau tidak boleh
pohon kelapa yang ada di pinggir pantai telah dikatakan yang pertama- daerah yang secara
terendam air dan para penduduk Lateu di formal berpindah karena pengaruh buruk
Pulau Tegue, Vanuatu, mulai membongkar perubahan iklim.
rumah kayunya dan berpindah ke pulau di Panel ilmuwan yang memberi saran
dekatnya yang 600 meter lebih tinggi. pada PBB memperkirakan bahwa permukaan
“Mereka tidak dapat linggal lebih lama air laut akan naik paling tidak satu meter pada
lagi di pantai," kata Taito Nakalevu, seorang 2100 karena melelehnya es sebagai dampak
ahli perubahan iklim di Secretariat Program pemanasan global yang dipicu terjebaknya
Lingkungan untuk Regional Pasifik. saat panas di atmosler. Gas yang dihasilkan dari
menghadiri konferensi untuk melawan pembakaran bahan bakar fosil di pembangkil

bimbelocean.com
UTUL UGM 69
PAKET 1
listrik, pabrik, dan mobil adalah penyebab efek 24. Penulisan nama dan jabatan yang tepat
yang disebut rumah kaca itu. terdapat pada kalimat
Banyak komunitas pantai lainnya yang (A) Ayahnya adalah seorang Gubernur.
terancam karena kenaikan air laut, misalnya (B) Siapa Bupati yang baru dilantik itu?
New Orleans di AS, Venesia di ltalia, atau (C) Seminggu yang lalu Kolonel Haryono
pemukiman di Kutub Utara. Mencairnya meninggal dunia.
gletser dan gunung es menyebabkan erosi (D) Brigjen Sumadi baru dilantik menjadi
pantai oleh gelombang air laut. Mayor Jenderal.
21. Topik yang dibicarakan dalam kutipan di (E) Presiden baru saja melantik
atas mengenai beberapa Menteri.
(A) banjir di kepulauan Samudera
Pasifik 25. Pemakaian tanda baca yang tepat
(B) penyebab naiknya permukaan air terdapat dalam kalimat
laut (A) Berbagai persoalan yang harus
(C) perubahan iklim global diselesaikan meliputi persoalan di
(D) hilangnya sebuah pulau di kawasan bidang politik, ekonomi, sosial dan
Samudera Pasifik. keamanan.
(E) dampak pemanasan global. (B) Berbagai persoalan yang harus
diselesaikan meliputi persoalan di
22. Pernyataan yang sesuai dengan isi bidang politik, sosial, ekonomi, dan
bacaan di atas adalah keamanan.
(A) Gas hasil pembakaran bahan bakar (C) Berbagai persoalan yang harus
fosil menjadi salah satu penyebab diselesaikan meliputi persoalan di
efek rumah kaca. bidang politik, sosial dan ekonomi
(B) Penduduk Lateu tinggal di Papua serta keamanan.
New Guinea. (D) Berbagai persoalan yang harus
(C) Para ilmuwan memperkirakan bahwa diselesaikan meliputi persoalan di
permukaan air laut akan naik lebih bidang politik, sosial ekonomi dan
dari satu meter. keamanan.
(D) Pulau Tegua adalah pulau yang (E) Berbagai persoalan yang harus
terancam banjir akibat naiknya diselesaikan meliputi persoalan di
permukaan air laut. bidang politik, sosial, dan ekonomi
(E) Mencairnya gletser dan gunung es di serta keamanan.
Kutub Utara menyebabkan banjir di
sebagian besar kawasan Samudera 26. Penulisan gabungan kata yang sesuai
Pasifik. dengan EYD dalam kalimat di bawah ini
adalah
23. Pernyataan yang tidak benar yang (A) Setiap Hari Raya Idul Fitri keluarga
berhubungan dengan Pulau Tegua besar itu selalu mengadakan acara
adalah halalbihalal.
(A) Rumah kayu merupakan rumah khas (B) Jika ingin membaca, ayah selalu
penduduk di Pulau Tegua harus mengenakan kaca mata.
(B) Di Pulau Tegua terdapat banyak (C) Adik saya selalu membawa sapu
pohon kelapa tangan hijau kesayangannya kalau
(C) Badai menyebabkan air pasang sedang pergi ke luar kota.
yang tinggi di Pulau Tegua (D) Mereka memberikan bantuan secara
(D) Pulau Tegua berpenduduk 100 suka rela kepada para korban
orang bencana alam itu.
(E) Penduduk Lateu berpindah ke (E) Mereka turut berduka cita atas
daerah lain karena tempat tinggalnya meninggalnya salah seorang teman
sering dilanda banjir mereka.

bimbelocean.com
70
UTUL UGM
PAKET 1

27. Penggunaan bentuk kata majemuk dapat seorang anak adalah satu hal yang
ditemukan dalam kalimat berikut, sepatutnya diperhatikan.
KECUALI (D) Sebelum memilih program studi
(A) Akibat jatuh dari sepeda motor, kaki bahwa penelusuran minat dan bakat
tangan Agus menderita patah tulang seorang anak adalah satu hal yang
dan terpaksa harus dirawat sepatutnya diperhatikan.
beberapa minggu di rumah sakit. (E) Merupakan satu hal yang sepatutnya
(B) Masyarakat Desa Makmur Sari diperhatikan bahwa penelusuran
mencurigai sepak terjangnya karena minat dan bakat seorang anak
ia memiliki kemiripan wajah dengan sebelum memilih program studi.
penjahat yang menjadi buronan
polisi. 30. ICEL (Indonesian Center for
(C) Untuk memperoleh kredit Environmental Law) yang melakukan
perumahan rakyat, para konsumen advokasi SUTET bersama sejumlah LSM
harus menyetor uang muka sebesar yang terbagung dalam Konsorsium
25 persen dari harga tunai. Dampak Jaringan Listrik melaporkan
(D) Setelah musim tanam usai, kasus listrik tegangan tinggi yang mulai
kebanyakan para petani yang naik ke permukaaan sejak 1989.
berasal dari desa pergi ke kota untuk Unsur inti katimat tersebut adalah
mencari pekerjaan sambilan, seperti (A) ICEL, melaporkan, kasus listrik
tukang angkut barang, penarik tegangan tinggi.
becak, dan sebagainya. (B) ICEL dan LSM, melaporkan, kasus
(E) Warga Desa Kayu Wangi yang tanah listrik sejak 1989.
perkarangannya terkena pelebaran (C) ICEL, advokasi SUTET, LSM,
jalan memperoleh ganti rugi dari melaporkan, kasus listrik tegangan
pemerintah. tinggi.
(D) ICEL, advokasi SUTET, LSM,
28. Kata yang dibentuk secara tidak baku Konsorsium Dampak Jaringan Listrik
terdapat pada kalimat melaporkan, kasus listrik tegangan
(A) Untuk meningkatkan daya saing, tinggi, sejak 1989.
cara penyajian harus dirubah. (E) ICEL. SUTET, LSM, kasus listrik.
(B) Perubahan dalam hal penyajian
merupakan syarat mutlak.
(C) Mereka memprotes keras 31. Cahaya matahari berlomba menyusup
pengubahan jadwal penandingan itu. lekukan kaku dinding-dinding beton
(D) Panitia hendaknya tidak sekehendak kelam dijantung kota Berlin.
hati mengubah jadwal pertandingan. Pola kalimat yang sama dengan kalimat
(E) Jadwal yang berubah setiap saat tersebut adalah
akanmenyusahkan semua pihak. (A) Bangunan itu dikelilingi dinding
sepanjang 100 meter yang
29. Kalimat yang mengandung pilihan Kata berpahatkan nama kota dan desa
secara tepat adalah tempat peristiwa itu berlangsung.
(A) Adalah satu hal yang sepatutnya (B) Terobosan manajer televisi itu
diperhatikan apabila penelusuran dilakukan untuk menggaet iklan agar
minat dan bakat seorang anak melimpah.
dilakukan sebelum memilih program (C) Holocaust Memorial berada di
studi. tengah megahnya Brandenburger
(B) Penelusuran minat dan bakat Tor, lambang bersatunya dua
seorang anak sebelum memilih Jerman.
program studi adalah satu hal yang (D) Matanya memandang jauh
sepatutnya diperhatikan. menyusuri lekuk-lekuk pekuburan
(C) Bahwa sebelum memilih program Yahudi tertua di Eropa itu.
studi, penelusuran minat dan bakat

bimbelocean.com
UTUL UGM 71
PAKET 1
(E) Bangunan-bangunan megah nan (A) Garis keturunan dari pihak ibu masih
anggun itu jelas kontras dengan sulit diakomodasi dalam RUU
ribuan batu nisan yang menyelimuti Kewarganegaraan.
areal seluas dua kali lapangan sepak (B) Menginjak tahun 1970-an kemajuan
bola. ilmu dan teknologi di segala bidang
maju pesat dengan ditemukkannya
32. Kalimat di bawah ini yang merupakan microchip di mana merupakan
kalimat majemuk campuran adalah komponen dasar komputer.
(A) Dia sudah melamar pekerjaan ke (C) Dari dialog itu tampak banyak sekali
manamana, tetapi belum juga kelemahan daripada Undang-
berhasil mendapatkan pekerjaan. Undang Nomor 62 itu.
(B) Selang beberapa waktu masuklah (D) Ketidakadilan dalam RUU itu terlihat
mereka. terutama dalam ketentuan yang
(C) Saya menjaga rumah, sedangkan mana mengatur perkawinan antar-
ibu pergi ke pasar. bangsa.
(D) Adik mendapat nilai buruk, padahal (E) Banyak pengusaha naka! Yang
dia sudah belajar giat. melarikan diri keluar negeri dengan
(E) Ketika ayah datang, adik bermain membawa asset dari negara secara
bola dan kakak membaca buku. melanggar hukum.

36. Dua atau beberapa universitas yang


33. Yang termasuk kalimat tunggal berikut ini besar bergabung menjadi satu. Istilah
adalah bergabung
(A) Kulitnya menghitam dan rambutnya dalam konteks ini sepadan dengan kata
menipis. (A) kombinasi.
(B) la mengakui ia pelakunya. (B) merger.
(C) Dalam kesempatan ini kami ingin (C) akomodasi.
menyampaikan ribuan terima kasih (D) konvergensi.
kepada hadirin sekalian yang telah (E) amalgamasi.
berjasa dalam proses pembangunan
Aceh pasca Tsunami ini. 37. Bank Indonesia menilai bahwa gejolak
(D) la lupa bahwa hari ini ada rapat. harga minyak internasional yang terjadi
(E) Walaupun hari ini libur, ia tetap saat ini tidak akan berpengaruh yang
bekerja. signifikan terhadap nilai tukar rupiah.
Alasannya, bahwa kebutuhan dollar AS
34. Para mahasiswa melakukan protes turun untuk impor minyak cenderung menurun.
ke jalan untuk menentang kebijakan Kata signifikan pada kalimat tersebut
pemerintah menaikkan harga BBM. mengandung makna yang sama dengan
Kalimat ini terbentuk dari perluasan kata berikut ini.
kalimat inti (A) secara berlebihan.
(A) Pemerintah menaikkan harga BBM. (B) secara bermakna.
(B) Mahasiswa turun ke jalan. (C) besar.
(C) Mahasiswa menentang kebijakan (D) secara kuat.
pemerintah. (E) secara merata.
(D) Mahasiswa melakukan protes.
(E) Mahasiswa menentang kenaikan 38. Dalam pasal 9 ayat 2 RUU Bahasa
BBM. menyebutkan bahwa pidato kenegaraan
yang disampaikan baik di dalam maupun
35. Di antara kalimat berikut ini yang di luar negeri harus menggunakan
menggunakan ragam kalimat baku Bahasa Indonesia yang baik dan benar.
adalah Agar kalimat tersebut baku, kata yang
perlu dihilangkan adalah
(A) dalam (sebelum kata pasal)

bimbelocean.com
72
UTUL UGM
PAKET 1

(B) menyebutkan (A) Dari karakter fisik manusia Indonesia


(C) bahwa menunjukkan warna kulit, lipatan
(D) harus kelopak mata, dan rambut yang
(E) dalam (sesudah kata depan di) secara umum digolongkan ke dalam
ras Mongoloid dan
39. ....... panen padi belum merata, sejumlah Auslralomelanesid.
pengumpul gabah mengaku kesulitan (B) Dari karakter fisik manusia Indonesia
mencari stok gabah kering panen ........ ditunjukkan bahwa warna kulit,
ada, rendemen hasil panen padi lokal lipatan kelopak mata, dan rambut
jauh lebih rendah ........ biasanya. yang secara umum digolongkan ke
Kelangkaan stok gabah kering panen dalam ras Mongoloid dan
........ disebabkan tidak adanya Australomelanesid.
panen,........ berubahnya sistem (C) Karakter fisik manusia Indonesia
pemesanan gabah. menunjukkan bahwa warna kulit,
Kata penghubung yang tepat untuk lipatan kelopak mata, dan ambut
mengisi titik-titik di atas adalah yang secara umum digolongkan ke
(A) karena, jika, karena, dan. dalam ras Mongoloid dan
(B) sebab, ketika, jika, maka. Australomelanesid.
(C) akibat, kalaupun, daripada, bukan, (D) Karakter fisik manusia Indonesia
melainkan. ditunjukkan oleh warna kulit, lipatan
(D) karena, ketika, dari, tidak, tetapi. kelopak mata, dan rambut yang
(E) akibat, ketika, dan, kalau, maka. secara umum digolongkan ke dalam
ras Mongoloid dan
40. Dan karakter fisik manusia Indonesia Australomelanesid.
menunjukkan warna kulit, lipatan kelopak (E) Karakter fisik manusia Indonesia
mata, rambut yang secara umum menunjukkan bahwa warna kulit,
digolongkan ke dalam ras Mongoloid dan lipatan kelopak mata, dan rambut
Australomelanesid. secara umum digolongkan ke dalam
Kalimat tersebut menjadi baku jika diubah ras Mongoloid dan
menjadi Australomelanesid.

BAHASA INGGRIS

I. Read the text carefully; then choose the best alternative to answer each of the questions.

These days, people spend a great deal of time on their health and fitness. Exercises,
nutrition and an emphasis on general wellness are important to people not only for medical
reasons, but for social ones as well. Everyone wants to feel and look their best.
Unfortunately, people around the world suffer from a condition that cannot be cured at a
health club, spa or even a hospital: chronic bad breath
5 It is estimated that over 80 million people worldwide suffer from bad breath, or halitosis.
In the past, treatment has consisted of masking the odor with mouthwashes or mints,
flooding the mouth with alcohol-based rinses, or the latest craze, popping pills that claim to
cure the problem in the stomach. None of these treatments work, because halitosis is caused
by bacteria on the back of the tongue and upper throat that produce sulfurous gases. The way
to stop bad breath is to stop this process.
10 Because halitosis originates in the mouth, it is virtually undetectable by your own sense of
smell.You may notice a bitter, sour taste in your mouth or a whitish coating on the back of your
tongue, but you generally find out there is a problem when a family member, friend or co-worker
brings it to your attention. At that point, you need an effective, long-lasting and easy-to-use
method of eliminating the problem. Without proper treatment, chronic bad breath can lead to a
loss of confidence and self- esteem, and it can even result in depression. The problem can
adversely affect your marriage, social life. career and relationship with family members. What is
needed is a quick and effective treatment that works naturally with no side effects.

bimbelocean.com
UTUL UGM 73
PAKET 1
As a dentist with a degree in bacteriology, Dr. Katz has been keenly aware of the
widespread nature of this problem. It was not until his daughter came to him about her
halitosis, however, that he began to research the problem in earnest. His studies led him to an
amazing discovery about the source of bad breath: it does not originate in the digestive system,
and the food you eat has no direct effect on your breath.Certain foods, however, contribute to
the production of sulfurous gases in the back of the mouth.
Acids in coffee and proteins in dairy products exacerbate the problem. Mints and
mouthwashes intended to mask or prevent bad breath actually worsen the condition
because sugar and alcohol dries out the mouth. Many common medications for
everything from high blood pressure to depression have the same drying effect, resulting in
the formation of odorous gases. Mucous from postnasal drip contains dense proteins that are
full of sulfur. Some treatments for halitosis contain Sodium Lauryl Sulfate, which can cause
canker sores. The only effective means of eliminating the sulfur gas production is to introduce
oxygen to the bacteria, causing them to produce tasteless, odorless sulfates.

Extracted from "Finally a cure for bad breath" by Jason Williams,


World Traveler, August 1999

41. The passage tells us about which of the 43. The word "it" in line 12 refers to ...
following?
(A) halitosis
(A) The causes of and solution to halitosis. (B) sense of smell
(B) People who suffer from halitosis (C) proper treatment
(C) Dr. Katz's problems in carrying out his (D) problem
research (E) method
(D) Breath, mouthwashes and popping pills
(E) A research in digestive system. 44. It can be inferred from the passage that ...

42. Which of the following is TRUE according to (A) chronic bad breath can be cured by a
the passage? dentist
(B) most people are concerned with the
(A) People who consume liquor regularly treatment of bad breath
are likely to suffer from halitosis. (C) protein and bacteria in the mouth are
(B) People who suffer from halitosis do not the causes of bad breath
go to health club. (D) the production of sulfurous gases is
(C) People know very well that carried out in the mouth through two
mouthwashes cure halitosis. stages
(D) Mouthwashes contain sugar and (E) the bitter and sour taste in the mouth
alcohol. are caused by common medicines
(E) The easily obtained mouthwash makes
80 million people use it.

bimbelocean.com
74
UTUL UGM
PAKET 1

45. Which of the following is TRUE according 48. The word "amazing' in line 18 is closest
to the passage? meaning to which of the following?

(A) Halitosis is caused by bad (A) well-known


digestive system. (B) outstanding
(B) Some common medicine will create (C) striking
odorless gases. (D) surprising
(C) The bacteria in the mouth are tasteless. (E) unusual
(D) Mouthwashes give effective result in
the problem of halitosis. 49. W hich of the following foods or beverages
(E) Halitosis can be detected from are the least to cause halitosis?
the emission of sulfurous gases.
(A) mint, milk and meat
46. The word "nature" in line 16 is closest (B) coffee, candy and cheese
in meaning to which of the following? (C) chocolate, ice cream and egg
(D) liquor, cake and syrup
(A) characteristic (E) water, carrot and tomato
(B) existence
(C) quality 50. The word "exacerbate" in line 21 is closest in
(D) state meaning to which of the following?
(E) kind
(A) exaggerate
47. Which of the following is TRUE according (B) evoke
to the passage? (C) worsen
(D) cause
(A) The primary need of people going to (E) irritate
health club is to improve their social
life.
(B) The treatment of halitosis with
medicine containing oxygen is
effective and has no side effects.
(C) The depression caused by halitosis
can be lessened by taking
medication for high blood pressure.
(D) It is easy to use mouthwashes
because the products can be found in
any store.
(E) Mouthwashes have a long lasting
effect in the treat

II. Read the following text and find the appropriate word or phrase to fill in each of the
blanks.
You may have wondered why the use of cellular telephones is not permitted in the cabin of
the plane. There are very good reasons for this. Modern aircrafts depend greatly on radio waves
(51) ______many functions, including communication with the control tower, navigation, and
even control of the atmosphere within the cabin. Radio wave (52) from cell phones seriously
disrupts these functions.
How bad is this disruption? Between March 1996 and February 2002, the British Civil
Aviation
Authority recorded no less than 35 air safety incidents related to the use of cell phones.

bimbelocean.com
UTUL UGM 75
PAKET 1
NASA has analyzed 118 cases related to the use of personal electronic devices in aircraft.
The NASA (53) in June 2001, concluded that 25 of these cases were stroggly correlated
with the use of cell phones, and 16 of these were critical cases.
You may not realize that (54) it is on standby, your phone is still emitting
electromagnetic signals. These signals serve to notify your cell phone network that the phone is
active and can be contacted. These signals (55) _ when the transmitter at a base
terminal station (BTS) communicates with your phone to send you a call or a text message
(SMS).
In fact, once the plane has taken off and (56)_ altitude, your cell phone will not work.
First, the distance between the BTS and the aircraft is too great. Second, the plane moves (57)
before the phone is detected and registered by a cell in the network, it has already left that
cell. (58) you cannot be contacted, if you leave your phone on "active" it continues to emit
electromagnetic signals that could seriously disrupt various types of flight control equipment.
That is why, although no nation has yet passed a law that would impose prison sentences
on passengers who insist on using their phones while in the aircraft, you (59) turn off your
phone as soon as you enter the cabin. If your cell phone is in your hand-carry luggage, please
check and make sure the phone is off.
Isn't it better to be careful, (60) the lives of hundreds of passengers, including yourself?

Extracted from Garuda, December


2005

51. A. to carry on 56. A. approached cruising


B. to carry out B. is approaching cruised
C. to be carried away C. is cruising approaching
D. to carry D. cruised approaching
E. to be carrying E. is approaching cruising

52. A. interfering 57. A. so fast that


B. interfered B. not so fast
C. interfere C. very fast
D. interferes D. faster
E. interference E. fast

53. A. released report 58. A. Because


B. report release B. But
C. report released C. But, even though
D. which released report D. However
E. releasing report E. Nevertheless

54. A. even 59. A. can


B. even when B. could
C. even though C. should
D. if even D. must
E. when even E. ought to

55. A. will be strong 60. A. rather than risking


B. must be strong B. rather risk
C. are strong C. risking
D. get stronger D. to risk
E. are stronger E. risk

bimbelocean.com
76
UTUL UGM
PAKET 2

PAKET TES KEMAMPUAN DASAR UMUM

2
WAKTU : 100 MENIT
JUMLAH SOAL : 60

MATEMATIKA DASAR
3 2𝑥+3𝑦+2
1.
5(√3+√2)(√3−√2)
= …. 5. JIka 𝑥 dan 𝑦 memenuhi =2
𝑥+𝑦
2√2−√3
3𝑥−𝑦+1
(A) √3 − √2 dan = 6, maka 𝑥 − 𝑦 = ….
4𝑥+5𝑦
(B) 3√3 − 2√2 (A) 6
(C) 2√2 − 3√3 (B) 5
(D) 3√2 − 2√3 (C) 4
(E) 4√2 − 3√3 (D) −4
(E) −5
2. Jika 3 log 8 = 𝑥 dan 3 log 25 = 𝑦, maka
3 6. Jika fungsi 𝑓(𝑥) = 𝑎𝑥 2 + 𝑏𝑥 + 𝑐
log 15√16 = ….
mencapai minimum di 𝑥 = 0 dan grafik
(A) 9𝑥 + 8𝑦 + 18
9𝑥+8𝑦+18 fungsi 𝑓 melalui titik (0,2) dan (1,8)
(B) 18 maka nilai 𝑎 + 𝑏 + 2𝑐 = ….
(C) 8𝑥 + 9𝑦 + 18 (A) 6
8𝑥+9𝑦+18
(D) 18
(B) 8
(E)
2𝑥+3𝑦+5 (C) 10
7
(D) 12
(E) 16
3. Penyelesaian persamaan 32𝑥+2 + 8 ∙
3𝑥 − 1 = 0 terletak pada interval ….
1
7. Diberikan 𝑥1 dan 𝑥2 merupakan akar
(A) [− 2 , 0] dari persamaan 𝑥 2 − 𝑝𝑥 + (𝑝 + 2) = 0.
(B) [−2,0] Nilai 𝑥1 2 + 𝑥2 2 minimum bila nilai 𝑝
1 1
(C) [− 2 , 2] sama dengan ….
1
(A) −1
(D) [2 , 1] (B) 0
(E) [1,2] 1
(C) 2
(D) 1
4. Persamaan garis yang melalui titik
(E) 2
potong garis 2𝑥 + 2𝑦 − 4 = 0 dan 𝑥 −
2𝑦 − 5 = 0 dan tegak lurus pada garis
8. Nilai maksimum dari 𝑧 = 4𝑥 + 9𝑦
12𝑥 + 6𝑦 − 3 = 0 adalah 𝑥 + 𝑏𝑦 + 𝑐 =
dengan syarat 𝑥 + 2𝑦 ≤ 12,2𝑥 + 𝑦 ≤
0. Nilai 𝑏 + 𝑐 adalah ….
12, 𝑥 ≥ 0, 𝑦 ≥ 0 adalah ….
(A) −7
1
(A) 24
(B) −3 2 (B) 42
1
(C) 1 2 (C) 48
(D) 13 (D) 52
(E) 5 (E) 54

bimbelocean.com
UTUL UGM 77
PAKET 2
1
(C) − 2
9.
(D) 1
(E) 2

14. Empat buah bilangan merupakan suku


berurutan suatu deret aritmetika. Hasil
kali kedua suku tengahnya sama
dengan 135 dan hasil kali kedua suku
Diketahui ∆𝐴𝐵𝐶 siku-siku di 𝐵,
4
pinggirnya sama dengan 63. Jumlah
cos 𝛼 = 5 dan tan 𝛽 = 1. Jika 𝐴𝐷 = 𝑎, kedua suku tengah tersebut adalah
maka 𝐴𝐶 = …. ….
(A) 4𝑎 (A) −35 atau 35
1
(B) 4 3 𝑎 (B) −27 atau 27
2 (C) −24 atau 24
(C) 4 3 𝑎
(D) −21 atau 21
(D) 5𝑎 (E) −15 atau 15
1
(E) 5 3 𝑎
15. Hasil penjualan suatu took serba ada
√𝑥 2 +5−3 diperlihatkan dalam lingkaran di
10. Nilai lim adalah ….
𝑥→2 𝑥 2 −2𝑥 samping ini. Jika diketahui hasil
(A) 0 penjualan minyak lebih besar
1
(B) Rp1.260.000,- dibandingkan hasil
3
1 penjualan beras maka hasil penjualan
(C)
2 rokok adalah ….
3
(D) (A) Rp1.260.000, −
4
(E) ∞ (B) Rp1.380.000, −
(C) Rp1.800.000, −
11. Fungsi 𝑦 = 2𝑥 + 3 √𝑥 2
3
mencapai (D) Rp1.890.000, −
maksimum untuk 𝑥 bernilai …. (E) Rp1.900.000, −
(A) 2
(B) 1 16. Jika 𝐴 dan 𝐵 dua kejadian dengan
(C) 0 𝑃(𝐵𝐶 ) = 0,45, 𝑃(𝐴 ∩ 𝐵) = 0,45 dan
( ) 𝐶
(D) −1 𝑃 𝐴 ∪ 𝐵 = 0,85 maka 𝑃(𝐴 ) sama
(E) −2 dengan ….
(A) 0,15
12. Jika nilai maksimum fungsi 𝑓(𝑥) = 𝑥 + (B) 0,25
(C) 0,45
√𝑎 − 3𝑥 adalah 1, maka 𝑎 = ….
3 (D) 0,55
(A) − 4
(E) 0,75
1
(B) − 4
(C) 0 −5 2 ],
17. Apabila 𝐴=[ 𝐴𝑇
1 2 −1
(D) 2 menyatakan transpose dari 𝐴 dan 𝐴−1
3
(E) menyatakan invers dari 𝐴, maka 𝐴𝑇 +
4
𝐴−1 = ….
3 7 −1 −2
13. Jika 𝑥 − 1, 𝑥 − 2 , 𝑥 − 4 adalah tiga (A) [ ]
−2 −5
suku pertama suatu deret geometri −1 2
(B) [ ]
maka jumlah tak hingga deret tersebut 2 −5
1 −2
adalah …. (C) [ ]
−2 5
(A) −2
(B) −1

bimbelocean.com
78
UTUL UGM
PAKET 2
−6 0 19. Akar-akar persamaan 𝑥 2 − (𝑎 + 3)𝑥 +
(D) [ ]
0 6 4𝑎 = 0 adalah 𝛼 dan 𝛽.
−6 0
(E) [ ] Nilai minimum dari 𝛼 2 + 𝛽 2 + 4𝛼𝛽
0 −6
dicapai untuk 𝑎 =….
−1 1 (A) −7
−1 2 0
18. Jika ( ) ( 1 0) + (B) −2
3 −1 2
2 1 (C) 2
𝑝 𝑞 1 0
( )=( ), maka 𝑝 + 𝑞 + 𝑟 = (D) 3
𝑟 𝑠 0 1
𝑠 =…. (E) 7
(A) −5 𝑥
log 𝑎 log(4𝑎 − 14)
(B) −4 20. Jika dan ( )=
log(𝑏 − 4) 1
(C) 3
log 𝑏 1
(D) 4 ( ), maka 𝑥 = ….
log 𝑎 1
(E) 5
(A) 1
(B) 4
(C) 6
(D) 10
(E) 106

BAHASA INDONESIA

Bacalah dengan cermat bacaan di bawah ini, kemudian jawablah pertanyaan nomor 21 sampai
dengan 23.

Orang Indonesia yang gemar melancong ke luar negeri mungkin lebih kenal Singapura atau
Eropa daripada daerah dan budaya negeri sendiri. Penelitian Departemen Kebudayaan dan
Pariwisata tahun 2004 mengungkapkan bahwa mereka yang bepergian ke mancanegara rata-rata
baru pernah mengunjungi tiga dari 33 provinsi di Tanah Air.
Selain wisatawan asing, turis lokal sebetulnya dapat menjadi pasar pariwisata. Persoalannya
adalah bahwa faktor yang menarik bagi mereka untuk berwisata di negeri sendiri sangat lemah.
Objek wisata di luar Bali, misalnya, rata-rata kurang terawat karena keterbatasan dana. Menjangkau
lokasinya pun tak mudah karena sarana transportasi terbatas. Berbagai retribusi memberatkan
pengusaha wisata untuk meningkatkan mutu pelayanan. Dukungan pemerintah daerah pada usaha
kepariwisataan pun masih kurang.
Pada tahun 2000, melalui 13 pintu keberangkatan, tercatat 2,2 juta orang Indonesia yang
berkunjung ke negeri asing. Empat tahun kemudian melonjak menjadi 3,9 juta. Hingga November
2005, sudah 3,7 juta orang melancong ke negara lain.
Menurut Sekjen Asosiasi Biro Perjalanan dan Wisata (ASITA), objek wisata di negeri orang
menyajikan atraksi menarik pada saat pergantian tahun. Bulan Juni dan Juli, masa libur sekolah, juga
waktu favorit warga Indonesia untuk berlibur ke luar negeri. Kelompok pelancong ke luar negeri
terbesar ini adalah penduduk berumur 25 hingga 34 tahun, kelompok yang berada di puncak
produktivitas. Mereka pada umumnya berupa pasangan muda, pengantin baru, keluarga dengan
anak usia di bawah lima tahun, dan dari kelas ekonomi menengah ke atas. Beban finansial yang
belum begitu berat memungkinkan mereka dapat menyisihkan biaya berlibur.

21. Pada paragraf kedua terdapat kalimat 22. Topik yang dibicarakan dalam teks di
yang tidak baku, yaitu kalimat atas adalah
(A) kesatu. (A) kecenderungan orang berlibur ke
(B) kedua. luar negeri.
(C) ketiga. (B) tujuan orang berlibur ke luar negeri.
(D) keempat. (C) daya tarik wisata ke mancanegara.
(E) kelima. (D) berkunjung ke negara tetangga.
(E) pelancong ke mancanegara.

bimbelocean.com
UTUL UGM 79
PAKET 2
23. Berikut ini adalah pernyataan yang tidak 25. Pelayanan kesehatan di berbagai tempat
sesuai dengan isi teks di atas. di tanah air belum beronentasi kepada
(A) Tempat wisata di Bali lebih terawat pasien. Hal ini ditandai oleh maraknya
daripada tempat lainnya. pernberian obat antibiotik yang tidak
(B) Bagian terbesar orang Indonesia sesuai dengan kondisi klinis. Pemakaian
yang berlibur ke luar negeri adalah antibiotik yang melebihi kebutuhan bisa
kelompok usia produktif. mengakibatkan resistansi dalam tubuh
(C) Jumlah orang Indonesia yang pasien. Kuman penyakit dalam tubuh
beriibur ke luar negeri meningkat menjadi kebal. Oleh karena itu, ....
dari tahun ke tahun. Kalimat yang tepat untuk melengkapi
(D) Umumnya orang lebih suka paragraf di atas adalah
mengunjungi tempat wisata di dalam (A) pertu ditumbuhkan sikap kritis
negeri sebelum mengunjungi tempat masyarakat terhadap layanan
(E) wisata di luar negeri. kesehatan.
(F) Pada tahun 2004 jumlah orang (B) pasien harus selalu mematuhi
Indonesia yang berkunjung ke luar semua instruksi dan dokter.
negeri mengalami kenaikan sebesar (C) pasien harus pasrah pada layanan
1,7 juta dibandingkan dengan tahun kesehatan yang diterima.
2000. (D) pasien harus diberi antibiotik di atas
kebutuhan supaya daya tahan tubuh
24. Mahasiswa idealis yang memiliki visi dan lebih tinggi.
misi serta imajinasi bagus itu tahun (E) setiap penyakit harus diobati dengan
depan akan mengikuti program antibiotik.
pertukaran mahasiswa Indonesia-Jepang
yang merupakan realisasi kerja sama 26. Penulisan kata serapan yang benar
bidang pendidikan dan pengajaran. terdapat dalam kalimat
Pernyataan yang benar berkaitan dengan (A) Setiap wanita WNI yang menikah
kalimat di atas adalah dengan pria WNA harus siap
(A) Yang menduduki fungsi subjek menanggung konsekwensi bahwa
dalam kalimat di atas adalah status kewarganegaraan anaknya
mahasiswa. akan mengikuti suaminya.
(B) Kalimat di atas dapat diubah menjadi (B) Kita pertu segera mengkonkritkan
Program pertukaran mahasiswa gagasannya itu.
Indonesia-Jepang yang merupakan (C) Penduduk Pasifik yang kebanyakan
realisasi kerja sama bidang tinggal di pulau karang adalah
pendidikan dan pengajaran tahun "kelompok yang paling beresiko”.
depan akan diikuti oleh mahasiswa (D) Kami mengusulkan agar salah satu
idealis yang memiliki visi dan misi di antara kita bersedia mengkoordinir
serta imajinasi bagus itu. pelaksanaan kegiatan ini.
(C) Kalimat di atas sama idenya dengan (E) Dia berhasil memopulerkan jenis
kalimat Tahun depan program olahraga berhahaya itu di
pertukaran mahasiswa Indonesia- negaranya.
Jepang akan diikutinya.
(D) Kalimat di atas tidak dapat diubah 27. Penulisan bilangan dalam kalimat berikut
menjadi kalimat pasif. tepat, KECUALI
(E) Dengan arti atau maksud yang (A) Penulisan bilangan tiga dua pertiga
sama, kalimat di atas dapat diubah adalah 3 2/3.
menjadi Mahasiswa itu tahun depan (B) Pada halaman 1.024 tertera nomor
akan mengikuti program pertukaran teleponnya 760130.
mahasiswa Indonesia-Jepang (C) Tamu undangan yang hadir dalam
karena memiliki visi dan misi serta perhelatan itu diperkirakan mencapai
imajinasi bagus. 500 orang.

bimbelocean.com
80
UTUL UGM
PAKET 2

(D) Ulang tahun yang ke-20 adalah (D) D Mereka juga memprotes kwalitas
peristiwa penting bagi orang Jepang. beras yang mereka terima.
(E) la telah membaca dua bab dan lima (E) Rapat itu gagal karena yang hadir
bab buku itu. tidak memenuhi korum.

28. Kalimat berikut tidak menggunakan tanda 31. Kalimat Koran Masuk Desa merupakan
baca yang tepat, KECUALI langkah yang strategis guna
(A) Telah dikemukakan bahwa, bahan meningkatkan kecerdasan masyarakat
bakar merupakan energi yang memiliki pola yang sama dengan
sangat penting dalam kehidupan (A) Pemerintah mengajukan beberapa
masyarakat. usul guna membangun desa-desa
(B) Meskipun belum sempurna, uraian tertinggal.
yang disampaikannya sangat (B) Pemerintah meningkatkan
menarik. kesejahteraan masyarakat dengan
(C) Investasi yang kita berikan ini, tidak proyek-proyek unggulan.
seluruhnya berupa uang. (C) Usaha pemerintah berupa
(D) Mutu pelayanan harus selalu pemberian subsidi kepada rakyat
ditingkatkan, supaya tidak kecil harus didukung.
mengecewakan pelanggan. (D) Usaha itu adalah strategi yang tepat
(E) Kerusakan lingkungan yang parah di untuk membangun kesadaran
Dataran Tinggi Dieng, boleh jadi rakyat.
akan menimpa kawasan lereng (E) Pemerintah berusaha meningkatkan
Gunung Ungaran. kesejahteraan rakyat dengan
berbagai upaya.
29. Penulisan kata yang bercetak miring
dalamkalimat berikut yang sesuai dengan 32. Kalimat yang mencerminkan penalaran
EYD adalah yang baik adalah
(A) Hasil analisa peneliti itu sangat (A) Karena kampanye yang melewati
bermanfaat bagi masyarakat dan ruas jalan itu terjebak macet
perkembangan ilmu pengetahuan. sehingga pertikaian antar simpatisan
(B) Dalam sistim pendidikan yang baik, tidak dapat dihindarkan.
proses pendidikan dapat (B) Dia yang tertua dari ketiga
diselenggarakan di dalam maupun di saudaranya yang kini sukses
luar kelas. menjadi pengusaha.
(C) Konduite merupakan salah satu (C) Harga lotokopi dengan mesin Xerox
syarat pegawai untuk naik pangkat. yang mengalami kenaikan drastis
(D) Pendapatnya sangat ekstrim akhirakhir ini.
sehingga para peserta rapat tidak (D) Kontrak jasa bertujuan
menyetujuinya. meningkatkan produksi ladang-
(E) Tehnik penulisan ilmiah harus dapat ladang minyak di wilayah Nimr-Karim
dipahami oleh para mahasiswa di dengan biaya yang efektif.
perguruan tinggi. (E) Mahasiswa yang sedang berunjuk
rasa tersebut yang mana terpaksa
30. Penulisan kata serapan yang betul menjadi pressure force yang harus
terdapat dalam kalimat diperhitungkan.
(A) Di dunia olahraga yang dipentingkan
adalah sportivitas. 33. Setidak-tidaknya ada tiga hal yang dapat
(B) Sikap yang kaku dari seorang dilakukan oleh sebuah perusahaan untuk
pemimpin menimbulkan watak meningkatkan kualitas dan disiplin
arogan dan otoritair. pegawainya. Pertama, perusahaan
(C) Majelis juga mengusulkan, agar menyediakan sarana kerja yang
kalangan eksekutip dan legislatip memadai. Kedua, dilakukan pemberian
peka terhadap aspirasi rakyat. contoh oleh atasan. Ketiga. perusahaan

bimbelocean.com
UTUL UGM 81
PAKET 2
menciptakan suasana kerja yang 35. Kalimat yang menggunakan ragam baku
menyenangkan. adalah
Empat kalimat di atas dapat diringkas (A) Dan hasil penelitian membuktikan
menjadi satu kalimat tanpa mengubah ide bahwa epilepsi bukan penyakit
dasarnya. Kalimat yang tepat menular.
mencerminkan ide yang dikandung empat (B) Mengenai rancangan undang-
kalimat di atas adalah undang itu belum disetujui anggota
(A) Kualitas dan disiplin pegawai DPR.
perusahaan dapat ditingkatkan (C) Hari ini Indonesia akan
dengan penyediaan sarana kerja memberangkatkan tiga belas
yang memadai, pemberian contoh mahasiswa untuk mengikuti
oleh atasan dan penciptaan suasana olimpiade matematika di Perancis.
kerja yang menyenangkan. (D) Menurut penelitian Depbudpar 2005
(B) Kualitas dan disiplin pegawai mengungkapkan orang Indonesia
perusahaan dapat ditingkatkan yang berlibur keluar negeri rata-rata
dengan tiga hal, yakni disediakan mengeluarkan dana 860 dollar AS
sarana kerja, diberikan contoh yang dalam setiap kunjungan.
baik, dan perusahaan menciptakan (E) Karya sastra melukiskan hakikat
suasana kerja yang menyenangkan. yang nyata dari objek sebagai titik
(C) Peningkatan kualitas dan disiplin tolak, kemudian dengan perlahan
pegawai perusahaan dapat bergerak ke arah imajinasi dan
dilakukan dengan menyediakan perasaan.
sarana kerja yang memadai,
pemberian contoh dari atasan, dan 36. Deretan kata yang mengemukakan ide
diciptakannya suasana kerja yang yang utuh adalah
menyenangkan. (A) Pengaruh perubahan sistem seleksi
(D) Untuk meningkatkan kualitas dan mahasiswa baru, khususnya di
disiplin pegawai, perusahaan UGM.
sebaiknya menyediakan sarana, (B) Penegakan keadilan yang harus
memberikan contoh, dan didukung oleh segenap elemen
menciptakan kesenangan. bangsa.
(E) Agar kualitas dan disiplin pegawai (C) Memudarnya semangat gotong
perusahaan meningkat, kita harus royong yang perlu diperhatikan.
menyediakan sarana, memberikan (D) Tingginya usia harapan hidup
contoh, dan suasana kerja pun merupakan salah satu indikator
tercipta. makin baiknya tingkat kesehatan
masyarakat.
34. Setahun berikutnya, dengan merangkul (E) Kerja sama Indonesia-Timur Tengah
Jaeckel, Lea Rosh menerbitkan sebuah yang tengah digalakkan oleh kedua
inisiatif 'Berlin dalam Perspektif untuk belah pihak.
menggolkan ide pembuatan memorial
untuk para keturunan Yahudi yang 37. Perayaan Imlek, misalnya, diakui sebagai
dibunuh secara massal di Eropa. hari libur fakultatif. Hari libur fakultatif
Inti kalimat tersebut meliputi unsur-unsur dalam kalimat tersebut maksudnya
(A) Lea Rosh, merangkul, Jackel. adalah
(B) Lea Rosh, menggolkan, ide. (A) hari libur yang berlaku untuk
(C) Lea Rosh, menerbitkan, inisiatif. kalangan terbatas.
(D) setahun, Lea Rosh, menggolkan, (B) hari libur yang dianjurkan.
ide. (C) hari libur yang belum diakui.
(E) keturunan Yahudi, dibunuh, di (D) hari libur yang belum resmi.
Eropa. (E) hari libur yang tidak diwajibkan.

bimbelocean.com
82
UTUL UGM
PAKET 2

38. Gaya untuk menghaluskan makna


terdapat dalam kalimat 40. Pembangunan perkebunan belum
(A) Laut biru membentang seluas mata berhasil meningkatkan pendapatan petani
memandang. kebun pada tingkat yang optimal. Proses
(B) Tubuhnya kurus kering tinggal kulit pembentukan kata dengan me – kan
pembalut tulang saja. pada kata meningkatkan dalam kalimat
(C) Semenjak anaknya meninggal, tersebut mempunyai makna yang sama
ingatannya agak terganggu. dengan proses pembentukan kata dalam
(D) Selama berminggu-minggu ia tidak kalimat
pernah menampakkan batang (A) Pembangunan itu dinilai belum
hidungnya. menguntungkan.
(E) Kesedihannya ditumpahkan pada (B) Metode itu akan dicobaterapkan
rumput yang bergoyang. untuk mengembangkan usaha
perkebunan itu.
39. Walaupun hasil pertanian berupa (C) Rancang bangun itu diarahkan
buahbuahan dan sayur-sayuran itu cukup kepada sentra produksi dengan
fluktuatif, produksinya masih dapat menggunakan salah satu instrumen
ditingkatkan dengan pendekatan yang ada.
teknologi. Kata fluktuatif dalam kalimat ini (D) Untuk menjalankan misinya, peran
berarti kantor cabang merupakan andalan
(A) bersifat naik turun. yang penting.
(B) bersifat turun terus. (E) Sejumlah nasabah telah
(C) bersifat naik terus. memanfaatkan dana kredit program
(D) bersifat naik lalu turun terus. ini.
(E) bersifat turun lalu naik terus.

BAHASA INGGRIS

/. Read the text carefully; then choose the best alternative to answer each of the questions.

Taking advantage from a high birth rate and the enormous potential of stem cell research,
India's biotechnology firms are coaxing more parents to bank blood from their newly born's
umbilical cord from which the blood cells are frozen and to be revived if and when there is a
scientific breakthrough. Blood which is extracted from the baby's umbilical cord and placenta
discarded after birth are loaded
5 with stem cells. They are master cells from which the body's immune and blood system
originate and which can develop into cells of any organ. Doctors harvest the blood cells once
the umbilical cord is clamped and cut, using a collection kit. The blood is then packaged in a
special shipping material provided. The company officials then ship the package to the
laboratory where the stem cells are processed and tested for various diseases.
10 These cells can help cure more than 75 serious ailments, according to Nalini Khrisnan, a
medical advisor of LifeCell, a firm which deals in preserving cord blood. She further says that
collecting and preserving the baby's cord blood stem cells is a security blanket for the baby
itself and its immediate family members. It is effective in the treatment of leukemia, anaemia,
inherited disorders and several other deficiencies of the immune system. Even lifestyle diseases
such as diabetes, liver disorders and
15 heart ailments and infectious diseases such as hepatitis, HIV and malaria can also be treated
with stem cells.
The two Indian companies which offer cord blood cell banking—LileCell and Reliance Life
Sciences—have a combined repository of more than 4,000 units, with Reliance dominating with
3,000 units. They are targeting to get to 15,000 units by the end of year 2006. LifeCell is going to
open two
20 banking centers outside India and to clock 10 billion rupees (USS 222 million) within the
next five years. Parents must pay 60,000 rupees (USS 1,333) for the process of cord blood
banking.

bimbelocean.com
UTUL UGM 83
PAKET 2
Prasad Mangupidi, marketing vice-president of LifeCell says the company followed "ethical
standards" despite controversies surrounding the stem cell research. In December, South
Korean investigators said that the apparent landmark stem cell research by cloning expert
Hwang W oo-suk
25 had been faked, turning the one-time national hero into a disgraced fabricator. People are not
worried with the controversy, though. Although the research at present is shrouded in a bit of
controversy, no one knows what will happen in the future. It is the "best gift" parents can give to
their children, said Roopa Devi, a doctor, who banked her son's cord blood cells at LifeCell.

Adapted from Jakarta Post, Jan. 4,


2006

41. What do biotechnology firms in India the placenta and umbilical cord of
successfully do ? newborn babies
They successfully ... (E) the firms can process the blood cells
wherever the samples are taken
(A) make a breakthrough in producing
healthy stem cells from newborn babies 44. It can be inferred that a baby's cord blood
(B) convince parents to keep their newborn cells ...
baby's blood cells in their firms
promising that they can be reused in the (A) are life-saving for all human kind
future (B) are not useful for other than the baby
(C) persuade parents to preserve their itself and its family
newly bom baby's placenta and umbilical (C) are still a controversy over who will
cord to be used for medical research benefit from it
(D) made to believe that their babies' cord (D) are useful for anyone who can afford
blood cells have curative values them
(E) make parents believe that the newly (E) are a medical breakthrough
bom baby's blood is loaded with stem cells
useful for their firms to make a 45. These cells" in line 10 refers to ...
breakthrough in medical treatment
(A) cells of any organ
42. The word "discarded" in line 4 is closest in (B) immune and blood system cells
meaning to (C) blood cells
(D) stem cells
(A) thrown away (E) all of the above
(B) taken from
(C) collected from 46. How do these firms treat the newly bom
(D) pulled out baby's cord blood cells?
(E) put into
(A) They extract the baby's blood.
43. In paragraph 1, we can infer that ... (B) They freeze the umbilical cord and
placenta and then ship them.
(A) newly born babies are a source of (C) They clamp and cut the umbilical
medical breakthrough cord and then ship them.
(B) the umbilical cord of newly born babies (D) They bank the baby's blood cells.
can be made into immune and blood (E) They freeze the baby's blood cells taken
systems from its umbilical cord.
(C) C. impaired human organs can be
repaired by stem cells derived from
fresh umbilical cord and placenta
(D) human immune and blood systems
originate from stem cells derived from

bimbelocean.com
84
UTUL UGM
PAKET 2

47. Stem cells cannot possibly cure ... 49. The word "disorders" in line 14 can best be
replaced by ...
(A) blood cancer
(B) nutritional deficiency (A) abnormalities
(C) leukemia (B) troubles
(D) haemoglobin deficiency (C) disturbances
(E) epilepsy (D) difficulties
(E) irregularities
48. What is/are possibly the cause(s) of
controversy over the research of stem cells? 50. We can infer from the article that stem cell
research ...
(A) The cost of the procedure is very high. (A) has resulted in many new
(B) The effectiveness of the results is still medical treatments
questionable. (B) searched for possible alternatives in
(C) Fabrications might occur. medical treatment, but failed
(D) Commercialism may (C) has created new hope among people
happen. thai most diseases may have cures
(E) All of the above (D) will be stopped in time due to its
controversial nature
(E) may cause more problems

II. Read the following text and find the appropriate word or phrase to fill in each of the blanks.

The Internet, World Wide Web or simply the Web started life as a means of
communication for serious computer techies. The "ARAP" in ARAPnet stands for Advanced
Research Projects Agency, a branch of the American military (51)_ top secret systems
and weapons during the Cold W aR (52)_ attached the "net" bit as an afterthought and in
1969 work began on the grandfather of today's internet.
(53) a set of interlinked computer systems that could withstand a nuclear explosion,
ARAPnet protected the flow of information between military (54) by creating a network of
geographically separated computers that exchanged data via a newly developed protocol called
NCP or Network Control Protocol. This Protocol would be superseded by TCP/IP in later years.
The web at this stage, (55) _ , was no place for today's Internet surfer. After all the
browser we take for granted today hadn't even been thought about. It wasn't until 1993 when an
Englishman, Tim Berners-Lee, decided that the volumes of information (56) to be
formulated in such a way that no matter where in the world the data had come from or on what
system it had (57) he could still read it with ease. Thus the Hyper Text Markup Language or
HTML was born.
It may have been Bemers-Lee who originally developed HTML (58) it was a young
student by
the name of Marc Andreessen who made (59) today with a little start up company and a
product called Mosaic. Suddenly anybody was throwing together web pages and putting them
on the Internet (60) . Now the home computer really could be put to use with the help of the
browsers, navigating you around the world while still sitting at your desk happily sipping Horiicks.

Adapted from "How the Internet was Born" by Philip Hall, Hamsafer (January-February 2001)

51. (C) developed the


(A) that developed (D) in the development of
(B) that the development of (E) by developing

bimbelocean.com
UTUL UGM 85
PAKET 2
52. (C) which he was receiving
(A) It (D) about which he had received
(B) They (E) received he had
(C) The projects
(D) The systems 57.
(E) Techies (A) existed
(B) sent
53. (C) remained
(A) As initially designed (D) received
(B) As an initial design of (E) originated
(C) Designed initially as
(D) As initiated designs of 58.
(E) As of initially designed (A) although
(B) however
54. (C) but
(A) basements (D) whereas
(B) basics (E) despite
(C) basically
(D) basis 59.
(E) bases (A) it what is
(B) what it is
55. (C) what is it
(A) in fact (D) it is what
(B) as a result (E) it what
(C) in addition
(D) however 60.
(E) furthermore (A) for reading all
(B) for all to read
(C) reading for all
56. (D) all to read
(A) that he had received (E) to read all
(B) he was receiving had

bimbelocean.com
86
UTUL UGM
PAKET 3

PAKET TES KEMAMPUAN DASAR UMUM

3
WAKTU : 100 MENIT
JUMLAH SOAL : 60

MATEMATIKA DASAR
13
(B) 3y – 4x + 2 = 0
6 3
( √𝑥 2 )( √𝑥 2 √𝑥+1)
(C) 3y + 4x – 13 = 0
13
1. 6 =⋯ (D) 3y + 4x – 2 = 0
𝑥 √𝑥+1
(A) 𝑥 √𝑥 + 1 (E) 3y – 4x + 10 = 0
(B) x
(C) 1 6. Jika garis g melalui titik P(–2, 1) dan
𝑡
(D) 6 2 memotong parabola y = x2 – 4x + 3 di titik
√𝑥
𝑥
Q(x, y) dan R(4, 3) maka y – 5x = ...
(E) (A) − 3
1
√𝑥+1
1
(B) − 9
2. Nilai semua x yang memenuhi 𝑎 log 2 𝑥 ≥
1
8 + 2 𝑎 log 𝑥, dengan bilangan a> 1, (C) 9
adalah ... (D)
1
(A) 𝑎2 ≤ 𝑥 ≤ 𝑎4 3
2
(B) 𝑥 ≤ 𝑎2 atau 𝑥 ≥ 𝑎4 (E) 3
1
(C) 𝑥 ≤ 𝑎4 atau 𝑥 ≥ 𝑎2
1 7. Jika kedua akar persamaan x2 – px + p =
(D) 𝑥 ≤ 𝑎2 atau 𝑥 ≥ 𝑎4 0 bernilai positif, maka jumlah kuadrat
(E) 𝑥 ≤ −2 atau 𝑥 ≥ 4 akar-akar itu mempunyai ekstrem ...
(A) minimum –1
4 8𝑥
3. Bila 5 (23𝑥−1 ) + 10 = 2, maka x = ... (B) maksimum –1
3 (C) minimum 8
(A) − (D) maksimum 8
2
2
(B) − (E) minimum 0
3
(C) 1
2 8. Nilai minimum dari z = 6x + 9y yang
(D) 3
memenuhi syarat 4x + y 20, x + y 20, x
3
(E) + y 10, x 0, dan y 0 adalah ...
2
(A) 40
4. Agar ketiga garis 3x + 2y + 4 = 0, x – 3y + (B) 50
5 = 0, dan 2x + (m + 1)y – 1 = 0 (C) 60
berpotongan di satu titik maka nilai m (D) 80
haruslah ... (E) 120
(A) –3
(B) 2 9. Agar fungsi f(x, y) = ax + 4y dengan
(C) 3 kendala x + y 12, x + 2y 16, x 0, y 0
(D) 4 mencapai minimum hanya di titik (8, 4),
(E) 6 maka nilai konstanta a yang memenuhi
adalah ...
5. Persamaan garis yang melalui titik potong (A) 2 <a< 4
garis-garis 6x – 10y – 7 = 0 dan 3x + 4y – (B) 4 <a< 6
8 = 0 dan tegak lurus dengan garis ke-2 (C) 4 <a< 8
adalah ... (D) –4 <a< –2
(A) 3y – 4x + 13 = 0 (E) –2 <a< –4

bimbelocean.com
UTUL UGM 87
PAKET 3
sin 48°+sin 12° (B) 56
10. Nilai dari cos 78°+cos 42° adalah ...
1 (C) 61
(A) (D) 63
2
(B) 1 (E) 65
(C) √3
(D) cos 18° 17. Tetangga baru yang belum anda kenal
(E) tan 18° katanya mempunyai 2 anak. Anda tahu
salah satunya adalah laki-laki. Peluang
𝑥 √𝑥−𝑝√𝑝 kedua anak tetangga baru anda
11. lim =⋯
𝑥→𝑝 √𝑥−√𝑝 semuanya laki-laki adalah ...
1
(A) 𝑝√𝑝 (A) 5
(B) 3p 1
(B)
(C) p 4
1
(D) 3√𝑝 (C) 3
1
(E) √𝑝 (D) 2
2
(E)
12. Semua nilai x agar fungsi 𝑓 (𝑥 ) = 3

𝑥√𝑥 2 + 4 naik adalah ... 18. Diketahui sistem persamaan linear


(A) −√2 < 𝑥 < √2 3x – 5y = m
(B) −2 < 𝑥 < 2 2x + 4y = n
(C) 𝑥 < −2 atau 𝑥 > 2 𝑏
Jika 𝑦 = 22, maka b = ...
(D) 𝑥 < −√2 atau 𝑥 > √2
(A) 2m – 3n
(E) −∞ < 𝑥 < ∞
(B) 2m + 3n
𝑑𝑦 (C) –3m + 2n
13. Jika y = 3 sin 2x – 2 cos 3x, maka =⋯ (D) 3m + 2n
𝑑𝑥
(A) 6 cos 2x + 6 sin 3x (E) –2m + 3n
(B) –6 cos 2x – 6 sin 3x
(C) 6 cos 2x – 6 sin 3x 19. Jika dua garis yang memenuhi
(D) 3 cos 2x + 2 sin 3x persamaan matriks
(E) 3 cos 2x – 2 sin 3x 𝑎 2 𝑥 16
( )( ) = ( )
1 𝑏 𝑦 −18
14. Jika Sn adalah jumlah n suku suatu deret sejajar, maka nilai dari ab = ...
geometri yang rasionya r maka (A) –4
𝑆4𝑛 (B) –2
=⋯ (C) 1
2𝑆2𝑛
(A) 𝑟 2𝑛 (D) 2
1 (E) 4
(B) (𝑟 2𝑛 − 1)
2
1
(C) + 𝑟 2𝑛 20. Nilai x yang memenuhi persamaan
2 3 4
1 log 𝑦 log 𝑧 2 1
(D) 2 (𝑟 2𝑛 + 1) ( 𝑥 ) = ( 16 )
log 𝑦 −2 log 𝑧 −2
(E) 𝑟 2𝑛 + 1 adalah ...
(A) √2
15. Suatu deret aritmatika mempunyai beda 2
dan jumlah 20 suku pertamanya 240. (B) √3
Jumlah tujuh suku pertamanya adalah ... (C) 4
(A) –5 (D) 9
(B) –6 (E) 81
(C) –7
(D) –8
(E) –9

16. Tiga kelas A, B, dan C berturut-turut


terdiri dari 15 siswa, 10 siswa, dan 25
siswa. Rata-rata nilai gabungan dari
ketiga kelas adalah 58,6. Jika rata-rata
nilai kelas A dan C berturut-turut 62 dan
60, maka rata-rata nilai kelas B adalah ...
(A) 50

bimbelocean.com
88
UTUL UGM
PAKET 3

BAHASA INDONESIA

21. Investasi perlu didorong ke luar Jawa. kencang di ruang konferensi, suara
Selain kekayaan sumber daya alamnya, tembakan dan kekerasan acap kali
investasi di luar Jawa juga lebih menarik mewarnai jalan-jalan di kawasan itu.
karena daya beli rata-rata masyarakat di Persoalan konflik Palestina dan Israel
luar Jawa sebenarnya lebih tinggi bukan persoalan elite belaka. Persoalan
dibandingkan dengan daya beli rata-rata ini telah menyebar dan meluas menjadi
penduduk Pulau Jawa. Namun, persoalan masyarakat Timur Tengah
keterbatasan infrastruktur di luar Pulau secara umum.
Jawa masih menjadi penghambat Berdasarkan isi bacaan di atas,
pengembangan investasi di sejumlah pernyataan berikut ini benar, kecuali ….
sektor. (A) Konflik Palestina dan Israel bukan
Dalam bacaan di atas tersirat bahwa …. hanya persoalan kedua bangsa itu.
(A) selama ini investasi lebih terpusat di (B) Dalam penyelesaian konflik
Pulau Jawa. Palestina dan Israel antara hasil
(B) belum ada investasi di luar Pulau perundingan dengan kenyataan di
Jawa. lapangan sering tidak sejalan.
(C) investasi di Pulau Jawa sudah tidak (C) Sampai saat ini belum ada
menguntungkan lagi. perundingan yang dapat
(D) investasi di luar Pulau Jawa akan menyelesaikan konflik Palestina dan
mendatangkan keuntungan besar. Israel.
(E) investasi di luar Pulau Jawa (D) Hanya perundingan para elite kedua
terhambat oleh keterbatasan negara yang dapat menyelesaikan
infrastruktur. konflik Palestina dan Israel.
(E) Masyarakat di Timur Tengah belum
22. Selain masih minim dan kurang memiliki pandangan yang sama
terurusnya fasilitas, persoalan toilet dalam menyelesaikan konflik
umum yang kotor di kota berakar pada Palestina dan Israel.
masalah budaya masyarakat. Dalam
budaya masyarakat Indonesia yang 24. Pada tahun 2008 pertumbuhan ekonomi
agraris, masyarakat pada zaman dahulu Amerika Serikat diperkirakan melambat
pernah punya kebiasaan buang hajat di menjadi 2% atau sedikit menurun
alam terbuka, seperti sungai, empang, dibandingkan dengan tahun sebelumnya.
hutan, atau semak belukar di belakang Menuruf IMF, sekitar 1% penurunan
rumah, tanpa harus membersihkannya. pertumbuhan ekonomi di Amerika Serikat
Kotoran dibiarkan saja karena bisa akan menurunkan pertumbuhan ekonomi
langsung diurai organisme dalam tanah, di Asia sekitar 0,5% - 1%. Dampak resesi
tumbuhan, atau air. Di kota orang harus global yang berasal dari resesi di Amerika
buang hajat di toilet, tetapi kebiasaan Serikat akan mempengaruhi proyeksi
lama masih dibawa. Buktinya, masih perekonomian di negara-negara Asia,
banyak pengunjung toilet umum pergi terutama Indonesia. Ekonomi global
begitu saja tanpa menyiram atau diperkirakan mengalami penurunan
membersihkan peturasannya. pertumbuhan sebesar 0,4 %, dari 5,2 %
Topik paragraf di atas adalah …. pada tahun 2007 menjadi 4,8 % pada
(A) persoalan toilet umum. tahun 2008. Negara-negara di Asia
(B) budaya masyarakat Indonesia. Tenggara diperkirakan mengalami
(C) organisme dalam tanah. tekanan paling parah akibat melambatnya
(D) kebiasaan lama orang dalam buang ekonomi yang terjadi di Amerika.
hajat. Berdasarkan bacaan di atas, pernyataan
(E) pengunjung toilet umum. berikut ini yang salah adalah ….
(A) Penurunan pertumbuhan ekonomi di
23. Satu pelajaran penting yang dapat Amerika Serikat berdampak pada
diambil dari pelbagai macam upaya penurunan pertumbuhan ekonomi
penyelesaian konflik Palestina dan Israel global.
adalah bahwa konferensi atau (B) Pada tahun 2008 pertumbuhan
perundingan apa pun tidak mampu ekonomi Amerika Serikat
mengubah warna jalan-jalan di Timur diperkirakan lebih rendah daripada
Tengah. Tatkala perdamaian menderu pertumbuhan ekonomi global.

bimbelocean.com
UTUL UGM 89
PAKET 3
(C) Pada tahun 2008 Amerika Serikat persilangan dari itik alabio Kalimantan
diperkirakan akan mengalami Timur dengan itik Mojosari Jawa Timur.
penurunan pertumbuhan ekonomi Kalimat yang tepat untuk melengkapi titik-
sebesar 1%. titik dalam bacaan di atas adalah ….
(D) Sebagai dampak penurunan (A) Para peternak itik di wilayah itu
pertumbuhan ekonomi Amerika membuat penemuan baru.
Serikat, pertumbuhan ekonomi di (B) Demikianlah yang terjadi pada para
Asia tidak akan turun lebih besar peternak itik di wilayah itu.
daripada besarnya angka penurunan (C) Dua jenis itik disilangkan oleh para
pertum-buhan ekonomi Amerika peternak itik di wilayah itu untuk
Serikat. mendapatkan varietas itik baru.
(E) Jika pertumbuhan ekonomi Amerika (D) Di wilayah itu para peternak itik
Serikat turun 2%, sebagai menunjukkan kreativitasnya.
dampaknya pertumbuhan ekonomi (E) Itulah sebabnya para peternak itik di
negara Asia akan turun sebesar 1% wilayah itu menunjukkan kreativi-
- 2%. tasnya.

25. Kehidupan orang Malind, salah satu suku 27. Asam folat selama ini dikenal luas
di Papua yang masih mengandalkan sebagai suplemen wajib pada masa
perburuan dan penokokan sagu untuk prakehamilan dan kehamilan ..., timbul
mencari makanan, sangat dipengaruhi anggapan bahwa zat ini hanya
alam. Selama ribuan tahun kekayaan dibutuhkan oleh para calon ibu. ..., baik
hutan Ulayat Malind membuat hidup pria maupun wanita sama-sama
mereka berkelimpahan. Sagu di mana- membutuhkan asupan harian asam folat.
mana, saham (kanguru kecil) dan rusa ..., kebutuhan harian asam folat pria
berlarian, burungburung berkicau di dewasa sama besarnya dengan
sekitar kampung adat, dan ikan pun kebutuhan wanita dewasa.
melimpah. Namun, sekitar dua puluh Kata yang tepat untuk mengisi tiga
tahun yang lalu, sebuah sentakan budaya bagian kosong dalam bacaan di atas
terjadi ketika lebih dan 48.000 hektare berturut-turut adalah ….
hutan ulayat dibuka menjadi sawah bagi (A) lalu, oleh karena itu, sedangkan.
para transmigran. (B) dan, walaupun, adapun.
Dari bacaan di atas dapat disimpulkan (C) sehingga, padahal, bahkan.
bahwa …. (D) meskipun, akan tetapi, jadi.
(A) kehidupan suku Malind telah (E) karena, sebenarnya, dengan
berubah dari sistem meramu dan demikian.
berburu ke bercocok tanam.
(B) pembukaan hutan Ulayat menjadi 28. Jumlah ini meningkat tajam dari tahun
sawah telah mempengaruhi sebelumnya yang hanya tiga kasus. (2)
kehidupan suku Malind. Menurut Direktur LBH Palembang, Eti
(C) kesejahteraan suku Malind semakin Gustina, pada tahun 2007 terjadi 39
meningkat sejak dibukanya hutan kasus penyerobotan tanah milik warga di
Ulayat menjadi sawah. Provinsi Sumatera Selatan. (3) Demikian
(D) sejak dibukanya hutan ulayat salah satu kesimpulan refleksi akhir tahun
menjadi sawah, suku Malind tidak LBH Palembang. (4) Konflik ini di
lagi dapat berburu. antaranya terjadi di Kabupaten Musi
(E) suku Malind belajar bercocok tanam Banyuasin dan Ogan Komering Ilir. (5)
di sawah dari para transmigran yang Lembaga Bantuan Hukum Palembang
datang ke daerah itu. mencatat peningkatan ketegangan akibat
kasus penyerobotan tanah milik warga
26. Terbatasnya lahan dan modal usaha di oleh sejumlah perusahaan perkebunan
tengah semakin tingginya tuntutan tahun 2007.
kebutuhan hidup acap kali justru Untuk menjadi sebuah paragraf yang
mendorong kreativitas. Tidak pernah runtut, kalimat-kalimat di atas harus
puas dengan hasil kerja yang sama disusun menjadi ….
bertahun-tahun, terobosanterobosan baru (A) 1-2-4-3-5
pun dilakukan untuk memperoleh hasil (B) 2-4-3-1-5
yang lebih baik. .... Mereka berhasil (C) 5-1-3-4-2
mengembangkan budi daya ternak itik (D) 2-3-4-1-5
unggul organik yang merupakan (E) 5-4-3-2-1

bimbelocean.com
90
UTUL UGM
PAKET 3

(A) Sesuai dengan kesepakatan,


29. Untuk membangun tanggul penahan pertunjukan itu akan diadakan di
banjir, pemerintah akan merelokasi warga Jawa dan Sumatra.
yang tinggal di bantaran sungai. (B) Masa yang akan datang kenaikan
Yang dimaksud dengan bantaran dalam yang tidak terkendali dari harga
kalimat di atas adalah … kebutuhan sehari-hari tidak boleh
(A) bagian hilir. terjadi lagi.
(B) bagian hulu. (C) Ramalan BMG semakin kita
(C) bagian sisi kanan dan kiri. perhatikan, terutama setelah muncul
(D) bagian yang tidak dialiri air. berbagai dampak tak terduga dari
(E) bagian lereng tebing. perubahan iklim.
(D) Di negara-negara bermusim empat,
30. Kata yang mengalami perubahan makna ramalan cuaca dan geofisika sudah
karena asosiasi terdapat dalam kaiimat lama menjadi bagian dari kehidupan
… sehari-hari.
(A) Persentase sarjana yang belum (E) Untuk keperluan pelestarian lingku-
mendapatkan pekerjaan semakin ngan diperlukan tanggung jawab
bertambah. kolektif dan antisipatif dari para
(B) Para pegiat perempuan semakin pengusaha yang bergerak di bidang
berani memperjuangkan hak- pengusahaan hutan.
haknya.
(C) Karena kesantunannya, oleh 33. Kalimat Tugas utama pemerintah adalah
temannya dia disebut sebagai memperhatikan kepentingan masyarakat
pendeta. kelas bawah mempunyai pola yang sama
(D) Sesekali dia melontarkan kritikan dengan kalimat ….
pedas terhadap kebijakan atasannya (A) Tingginya angka inflasi akan
yang dianggapnya keliru. berpengaruh terhadap masyarakat
(E) Pada waktu melaksanakan tugas kelas bawah.
jurnalistiknya, dia selalu berusaha (B) Oligarki (kekuasaan di tangan
menolak pemberian amplop dalam segelintir orang) dan plutokrasi
setiap peristiwa yang diliputnya. (peme-rintahan oleh sekelompok
orang kaya) menjadi penumpang
31. Kalimat Kegiatan ekoturisme untuk gelap sistem demokrasi.
mengkonservasi alam sering tidak sejalan (C) Tidak satu pun partai politik di
dengan kebiasaan masyarakat setempat Indonesia terbebas dari sindrom
memiliki pola yang sama dengan kalimat oligarki dan plutokrasi.
…. (D) Sistem kepartaian yang sederhana
(A) Banyak pegiat ekoturisme mengakui dapat mengerem pertumbuhan
bahwa kesalahan yang sering terjadi oligarki dan plutokrasi.
dalam mengembangkan ekoturisme (E) Partai politik dapat menempatkan
adalah pengemasan yang semata- kadernya untuk mengisi jabatan
mata mengikuti selera masyarakat. publik sesuai dengan kapasitas dan
(B) Pembuatan patung kayu komodo kapabilitasnya.
pasti akan menghabiskan
pepohonan yang sekarang sudah 34. Kalimat yang baku adalah ….
jarang. (A) Untuk mengatasi banjir Bengawan
(C) Larangan pengambilan kayu waru Solo, penanaman pohon di sekitar
laut di Pulau Komodo Waduk Gajah Mungkur dan
dilatarbelakangi oleh terbatasnya sepanjang bengawan tidak dapat
populasi vegetasi di sana. ditawar lagi.
(D) Haji Zainun terus membuat patung (B) Sedangkan penanganan jangka
komodo di sela-sela kesibukannya pendek dengan memperbaiki tanggul
sebagai nelayan. sepanjang bengawan yang rusak.
(E) Keterampilan membuat patung (C) Juga dengan membuat bendungan
komodo merupakan sumber mata kecil di puluhan titik sepanjang
pen-caharian baru warga Pulau bengawan.
Komodo. (D) Di sepanjang bengawan
menemukan lebih dari 20 tempat
32. Kalimat yang tidak baku adalah …. yang potensial untuk dibangun
waduk.

bimbelocean.com
UTUL UGM 91
PAKET 3
(E) Jika di tempat tersebut dibangun
waduk kecil-kecil, akan bermanfaat 38. Penurunan cukai impor beras menjadi
mengurangi banjir. Rp.450,- per kilogram per 1 Januari 2008
dimaksudkan agar Bulog dapat
35. Kalimat berikut baku, kecuali … mengendalikan harga beras.
(A) Pesan terpenting dari sejarah adalah Kalimat tersebut akan benar ejaannya
pesan tentang kesalahan. apabila …
(B) Pesan itu kami belum terima sampai (A) Rp.450, ditulis Rp 450,00.
sekarang (B) per kilogram ditulis perkilogram.
(C) Media massa ramai mengulas (C) per 1 Januari 2008 ditulis per-1
temuan rekening liar. Januari 2008.
(D) Dibutuhkan aturan yang jelas untuk (D) Impor ditulis Import.
melaksanakan pilkada ulang. (E) sebelum kata agar ditambahkan
(E) Indonesia kehilangan julukannya tanda koma (,).
sebagai zamrud khatulistiwa.
39. Kalimat yang penulisannya mengikuti
36. Tidak dapat disangkal bahwa EYD adalah ….
kebanyakan orang pada saat ini lebih (A) Sepuluh tahun yang lalu Keluarga
memilih menggunakan kamera yang ada Satrio tinggal di jalan RE
pada telepon genggam daripada secara Martadinata, No.5.
khusus membeli kamera biasa karena (B) Sebagian besar dari kita
kamera analog dianggap kurang praktis. beranggapan bahwa kebudayaan
Inti kalimat tersebut adalah …. Timur lebih halus jika dibandingkan
(A) Disangkal. dengan kebudayaan Barat.
(B) Orang memilih kamera telepon (C) la mengakui ke dalaman wawasan
genggam. tokoh masyarakat kampung Naga
(C) Orang menggunakan kamera. itu.
(D) Orang membeli kamera. (D) Pengusaha besar itu menghadiahi
(E) Kamera analog tidak praktis. atlitatlit berprestasi internasional
dengan bea siswa untuk belajar
37. Kalimat Dalam pergaulan sehari-hari keluar negeri.
sering terlihat ada orang yang tidak dapat (E) Beberapa tokoh anti rasisme dari
menerima saran orang lain memiliki pola beberapa negara sepakat untuk
yang sama dengan kalimat …. menandai tangani suatu
(A) Dengan terbata-bata dikisahkannya kesepahaman.
peristiwa menyedihkan yang
menimpa dirinya itu. 40. Untuk kurangi pemanasan global maka
(B) Di sekitar kita ada orang yang masyarakat diminta untuk meminimalisir
mendorong dirinya untuk membaca pemakaian kendaraan bermotor.
evaluasi dan menghadiahi dirinya Kalimat di atas akan menjadi kalimat
sendiri jika hasilnya bagus. baku apabila diperbaiki dengan cara
(C) Karena hampir setiap individu berikut, kecuali ….
berinisiatif untuk berkembang dan (A) mengganti kata untuk pada awal
berubah, tidak perlu kalimat dengan dalam.
dikumandangkan gerakan (B) mengubah kata kurangi menjadi
perubahan. mengurangi.
(D) Agar dapat terus bekerja di (C) menambahkan tanda koma (,)
perusahaan itu, diikutinya berbagai sesudah kata global.
kursus keterampilan. (D) menghilangkan kata maka.
(E) Dalam lingkungan yang terbatas (E) mengubah kata meminimalisir
disampaikannya pesan yang agak menjadi meminimalkan.
rahasia itu.

bimbelocean.com
92
UTUL UGM
PAKET 3

BAHASA INGGRIS

I. The text below has incomplete sentences. Choose the one word or phrase from each
number that best completes the sentence.

The cultural values of a society determine the social norms of the society. The social norms are the group-
shared rules of behavior. These norms are the (41) ways of behavior in the society based on the
shared values - the purposes and goals - of that society. For example, because we place some value on
physical modesty, we have the social norm of wearing clothes in public. (42) members of the society,
we must live by the rules which that society imposes on us. W hen a person doesn't behave according to these
norms, he or she is considered a deviant. Someone who walks down the main street of an American city
without clothes on (43) a deviant, since he or she is not following the standard rules of behavior.
Some norms apply to all persons in a particular situation. For example, in a certain culture all persons are
expected to be quiet in a library, obey the traffic signals, eat with knife, fork, and spoon, (44) during the
playing of the National Anthem, and pay their bills, These are just a few of the many norms that every member
of the society (45) expected to follow.
Other norms, however, (46) to the individual. They may vary from one person to another.
These norms are related to the individual's special role. A role is the behavior expected of a person because
of his or her position in the social structure or group. Roles are actually specific norms related to the particular
position of the individual. They tell people how they are to behave in their particular positionin the society or
group. For example, an onlooker and a doctor (47) different roles to play if both were at the scene of
a serious car crash (48) several persons were critically injured. The role of the doctor would demand thit
he or she (49) first aid to try to save the lives of the injured. The role of the onlooker might simply (50)
the person to stay out of the way.
Source: Thomas, W.L. & Anderson, R.J. 1977. Sociology: The Study of Human Relationships. New York:
Harcourt Brace Jovanovich.

41. 46.
(A) expecting (A) are applied
(B) expected (B) applies
(C) expectation (C) applied
(D) expect (D) apply
(E) expectancy (E) applying

42. 47.
(A) To accept (A) would have
(B) Accepting (B) will have
(C) To be accepted (C) would have had
(D) Accepts (D) have
(E) Be accepted (E) had

43. 48.
(A) is regarded b (A) which
(B) are regarded by (B) by which
(C) is regarded as (C) at which
(D) are regarded as (D) from which
(E) is regarded for (E) in which

44. 49.
(A) standing (A) give
(B) to stand (B) will give
(C) stand (C) to give
(D) stands (D) has given
(E) stood (E) giving

45. 50.
(A) is (A) requiring
(B) was (B) to require
(C) are (C) requires
(D) were (D) required
(E) be (E) require

bimbelocean.com
UTUL UGM 93
PAKET 3
II. Study the passage and choose the best answers to the questions that follow.
Your biggest enemy is yourself." This idea is being explored by teachers as they and their
students deal with the issue of bullying.
The hardest thing is to control our own emotions because some teachers, like myself, are used to the
stern approach in disciplining students," vice principal of SMAN 103 state high school in East Jakarta,
Budi Muntoro, told The Jakarta Post on .Wednesday.
Most people do not realize that at some stage, adults - parents and teachers - unconsciously
teach bullying to children, especially in the way they use violence in education."About 50 percent of
teachers might be guilty of bullying at some point," said Budi. SMAN 103 is one of three public high
schools in Jakarta taking part in a pilot project on
bullying prevention.
Teachers and principals from the school participated in a workshop on bullying hosted by the
Semai Jiwa Amini Foundation in September. The workshop was attended by 57 SMAN 103 teachers
"Previously, I thought bullying was only physical violence. But now I realize that it also involves
psychological aspects," said Budi who teaches geography.
"So, the violence is not just physical but it can also be in the form of verbal and non-verbal
expressions," said the 43 year-old teacher.
Budi, who has been teaching for 23 years, now tries to watch his words. "I never thought before
that calling students by their nicknames was bullying," he said. Budi, describing himself as a former "killer"
teacher, said he used to regard slapping and pinching as common in disciplining students. "Now, we give
priority to counseling rather than just punishing problematic students," Budi said, referring to
consultation hours provided once a week for students. SMAN 103 currently-has four teachers who have
counseling hours, with five intern teachers assisting them.
Those teachers rarely helped students with counseling before, but lately they provide more time on
a weekly basis for the activity, which is a good thing," said 15-year-old Sari, one of 840 students at
SMAN 103. Asked about the benefit of the counseling, second year student Sari acknowledged she was
not used to sharing her private thoughts with others, including teachers, but the service was helpful for
some of her troubled friends. Budi said he has been trying for years to change his rather militaristic
approach in the classroom. "Now, I don\want to have obedient students just because they are afraid of
me." However, he said some students tended to do as they pleased if they were not punished harshly.
"I'm still looking for a solution, but of course I don't want to go back to my old ways of teaching."

Source: The Jakarta Post, November 28, 2007

51. The passage mainly discusses …. 53. According to the passage, which of the
following best describes the word
(A) the use of stern approach in "bullying" in line 8?
disciplining students in classrooms.
(B) the need to give harsh punishments (A) nuisance
to problematic students at schools. (B) unfairness
(C) violence that possibly teaches bullying (C) intimidation
to children. (D) verbal abuse
(D) the teachers' difficulty in (E) insults
controlling their emotions in
classrooms. 54. In line 13, the word "it" refers to ….
(E) the use of softer approach in dealing
with problematic students. (A) violence
(B) psychological aspects
52. Which of the following is closest in (C) bullying
meaning to "stern" in line 4? (D) physical violence
(E) geography
(A) uncompromising
(B) strict
(C) frightening
(D) lenient
(E) ruthless

bimbelocean.com
94
UTUL UGM
PAKET 3

55. The passage indicates that ….


58. Which of the following has the same
(A) the majority of teachers in high meaning as the word "acknowledged" in
school might have bullied their line 25?
students.
(B) Ball high school teachers participated (A) denied
in the workshop in September 2007. (B) allowed
(C) not all teachers knew the kinds (C) knew
of violence bullying might involve. (D) recognized
(D) the workshop on bullying will be (E) admitted
hosted by the Semai Jiwa Amini
Foundation.
(E) the stem approach in 59. Which of the following is NOT
disciplining students makes teachers considered as bullying?
able to control emotions.
(A) Calling someone "shorty."
56. In line 19, the word "regard" could best (B) Hitting someone's head
be replaced by (C) Punishing severely.
(D) Shouting disapproval.
(A) consider (E) Giving a hard blow.
(B) ignore
(C) doubt 60. It can be inferred from the passage that
(D) conclude Budi
(E) presume
(A) realizes that using militaristic
57. The pronoun "them" in line 22 refers to approach is definitely needed in
which of the following? education.
(B) is a senior teacher in one of the
(A) Problematic students who juniorhigh schools.
need counseling. (C) has better understanding about
(B) Five intern teachers who assist what bullying is
other teachers. (D) is a "killer" teacher who considers
(C) Parents whose children get that slapping and pinching are
punishment. common.
(D) Teachers who have counseling (E) believes that bullying is merely
hours. physical violence.
(E) Some of the 840 students at SMAN
103.

bimbelocean.com
UTUL UGM 95
PAKET 4

PAKET TES KEMAMPUAN DASAR UMUM

4
WAKTU : 100 MENIT
JUMLAH
MATEMATIKA: DASAR
SOAL 60

MATEMATIKA DASAR

1. Jika x1 dan x2 adalah penyelesaian


persamaan (D) 1
𝑥 2 −3 1−𝑥 (E) 2
4 8 3
( ) ( ) =
9 27 2
maka (𝑥1 − 𝑥2 )2 5. Dua kg jeruk dan tiga kg apel harganya
(A)
9 Rp45.000,-. Lima kg jeruk dan dua kg
4
25
apel harganya Rp 52.000,-. Harga satu
(B) 4 kg jeruk dan satu kg apel sama dengan ...
41
(C) (A) Rp6.000,-
4
25 (B) Rp9.000,-
(D) 2 (C) Rp11.000,-
(E) 25 (D) Rp17.000,-
(E) Rp20.000,-
2. Jika 2x = a dan 2y = bdengan x, y> 0,
2𝑥+3𝑦
maka =⋯ 6. Jika garis (a + b)x + 2by = 2 dan garis ax
𝑥+2𝑦
3 – (b – 3a)y = –4 berpotongan di (1, –1),
(A) 5 maka a + b = ...
5
(B) 3
(A) –2
(C) 1 + 𝑎𝑏
log 𝑎𝑏2 (B) –1
(D) 1 + 𝑎𝑏
log 𝑎2 𝑏 (C) 0
𝑎𝑏2 (D) 1
(E) 1 + log 𝑎𝑏
(E) 2
3. Diketahui x1 dan x2 akar-akar persamaan
1 1 4√𝑥 1
6x2 – 5x + 2m – 5 = 0. Jika + 𝑥 = 5, 7. Pertaksamaan ≤ mempunyai
𝑥1 𝑥 2 +3 √𝑥
2
maka nilai m adalah ... penyelesaian ...
(A) –1 (A) 1 ≤ 𝑥 ≤ 3
(B) 0 (B) 1 ≤ 𝑥 ≤ √3 atau 𝑥 ≥ 3
(C) 1 (C) 𝑥 ≤ 1 atau 𝑥 ≥ 3
(D) 2 (D) 0 < 𝑥 ≤ 1 atau 𝑥 ≥ 3
(E) 3 (E) 0 ≤ 𝑥 ≤ 1 atau 𝑥 ≥ 3

8. Nilai maksimum untuk z = 6x + 3y – 2


4. Jika persamaan x – 2ax – 3a + 4a + 1 =
2 2 yang memenuhi sistem pertaksamaan
0 mempunyai akar kembar, maka akar x + 2y 4
tersebut adalah ... x – y 2
(A) –1 x + y 1
1
(B) − 2 x 0, y 0
(C)
1 adalah ...
2 (A) 4

bimbelocean.com
96
UTUL UGM
PAKET 4
1
(B) 10 (D) 2
(C) 13
(E) 1
(D) 16
(E) 19
14. Nilai x yang memenuhi sin x – cos x> 0, 0
x 2 adalah ...
9. Dalam suatu deret aritmatika, jika U3 + U7 𝜋
(A) 0 ≤ 𝑥 ≤ 2
= 56 dan U6 + U10 = 86, maka suku ke-2
𝜋 3𝜋
deret tersebut adalah ... (B) ≤𝑥≤
2 2
(A) 8 𝜋 5𝜋
(C) <𝑥<
(B) 10 4 4
(D) 𝜋 < 𝑥 < 2𝜋
(C) 12 3𝜋 3𝜋
(D) 13 (E) <𝑥<
4 2
(E) 15
15. Jika sebuah dadu dilempar dua kali,
10. Jika barisan geometri y + 1, 2y – 2, 7y – maka peluang untuk mendapatkan jumlah
1, ... mempunyai rasio positif, maka suku angka kurang dari lima adalah ...
ke-4 barisan tersebut adalah ... 2
(A) 3
(A) 108 4
4 (B)
(B) 9
3 5
4 (C)
(C) − 3 18
1
(D)
(D) –108 6
1
(E) –324 (E) 12

𝑎−𝑏 −𝑏 −1 𝑎 1 16. Nilai rata-rata tes matematika suatu kelas


11. Jika ( ) =( ),
0 1 −𝑎 + 2𝑏 1 yang terdiri dari 42 siswa adalah 6,3
maka ab = ...
dengan jangkauan 4. Jika satu nilai
(A) 2
terendah dan satu nilai tertinggi tidak
(B) 1
1
diikutsertakan, maka rata-ratanya
(C) − 2 menjadi 6,25. Nilai terendah untuk tes
(D) –1 tersebut adalah ...
(E) –2 (A) 5
(B) 5,03
12. Jika A matriks berordo 2  2 sehingga (C) 5,3
1 −1 2 4 (D) 5,05
𝐴 ( ) = ( ) dan 𝐴 ( ) = ( ), maka A2
−1 5 1 7 (E) 5,5
= ...
1 2 5𝑥
(A) ( ) 17. Diketahu f(x) = 2x – 1 dan 𝑔(𝑥 ) = 𝑥+1.
4 −1
9 0 Jika h adalah fungsi sehingga (g∘h)(x) = x
(B) ( )
0 9 – 2, maka (h∘f)(x) = ...
9 0
(C) ( ) 2𝑥−3
0 7 (A) 2𝑥+8
7 0 2𝑥−3
(D) ( ) (B)
0 9 −2𝑥+6
7 0 (C)
2𝑥−3
(E) ( ) 2𝑥−8
0 7 2𝑥−3
(D) −2𝑥+8
√2𝑝𝑞 2𝑥−3
13. Jika sin A = √2𝑝𝑞, dan tan A = , (E) −2𝑥−8
𝑝−𝑞
2 2
maka p + q = ...
(A) –1
(B) 0
1
(C) 4

bimbelocean.com
UTUL UGM 97
PAKET 4

18. Jika 𝑓(𝑥 ) = 𝑥√1 − 𝑥 maka nilai a yang (A) f mencapaimaksimumrelatif di x = –1


memenuhi f’(a) = 1 adalah ... (B) f mencapai minimum relatif di x = 1
(A) 0 (C) f mencapaimaksimumrelatif di x = –3
8 dan x = 1
(B) 9
8 (D) f mencapaimaksimumrelatif di x = –3
(C) 0 dan 9 dan x = 2
8
(D) 0 dan − (E) f mencapai minimum relatif di x = –3
9
8 8 dan x = 2
(E) − 9 dan 9
20. Jika x1 dan x2 memenuhi persamaan
19. Jika grafik di bawah merupakan grafik 2𝑥 − 3 3 1 3
| |=| |
fungsi y = f’(x), maka ... 𝑥 𝑥−2 4 6
maka x1.x2 = ...
(A) –12
(B) –6
(C) 0
(D) 6
(E) 12

BAHASA INDONESIA

Bacalah bacaan berikut dengan cermat, Tradisionalisme, mulai dari cara makan,
kemudian jawablah pertanyaan nomor 1 cara mandi, cara berinteraksi, hingga
sampai dengan 3. pandangan hidup, betul-belul masih melekat
dan menjadi identitas warga pedalaman
Begitu matahari terbit di Batu Kambar, Pegunungan Meratus. Lambat Iaun
Desa Hinas Kiri, Kecamatan Barang Alai keterisolasian pedalaman memang bisa
Timur, Kabupaten Hulu Sungai Tengah, ditembus, tetapi hingga kini kehidupan alami
Kalimantan Selatan, ratusan warga ke luar masih melekat. Baik di Hulu Sungai Selatan
rumah. Ada yang Iangsung pergi ke maupun di Hulu Sungai Tengah yang
pahumaan (Iadang), ada yang pergi ke pasar, mempunyai komunitas adat terbesar di
ada pula yang berangkat ke sekolah. Warga di Kalimantan Selatan suasananya sama-sama
pedalaman Pegunungan Meratus memang masih tradisional. Tradisionalisme di satu sisi
terbiasa bangun pagi karena mereka juga menjadi kebanggaan, tetapi di sisi Iain
menyisakan pertanyaan akan capaian sebuah
kesejahteraan masyarakat.
sudah terbiasa tidur sore akibat tidak adanya
aliran Iistrik. Karena itu, aktivitas pagi di 21. Judul yang tepat untuk bacaan di atas
Pegunungan Meratus betul-betul menggeliat adalah
dan dinamis. Hari itu kebetulan hari pasar (A) Kisah Kehidupan dari Batu Kambar,
tradisional yang hanya ada sekali dalam Desa Hinas Kiri.
sepekan. Masyarakat dari berbagai balai adat (B) Kehidupan Tradisional Masyarakat
Dayak Meratus, yang jarak tempuhnya hingga Pegunungan Meratus.
18 jam dengan jalan kaki, berduyun-duyun ke (C) ldentitas Warga Pedalaman
pasar untuk membeli keperluan hidup Pegunungan Meratus.
"modern", mulai dari bahan pangan, sandang,
hingga papan.

bimbelocean.com
98
UTUL UGM
PAKET 4

(D) Tradisionalisme Meniadi (A) kerisauan para petani dan perajin


Kebanggaan bagi Masyarakat makanan oleh-oleh di Purwokeno
Pegunungan Meratus. karena sulitnya pupuk dan minyak
(E) Selama 18 jam Berjalan Menuju tanah.
Pasar untuk Keperluan Hidup (B) “musim panen" bagi industri kecil
"Modern". makanan oleh-oleh.
(C) kerisauan Bambang Riyanto,
22. lnti kalimat keenam dalam paragraf seorang perajin jenang dan tempe
pertama bacaan di atas adalah keripik asal Purwokerto.
(A) Masyarakat berjalan ke pasar (D) kelangkaan minyak tanah di
selama 18 jam. Purwokerto.
(B) Masyarakat membeli keperluan (E) suka duka perajin jenang dan tempe
hidup "modern”. keripik asal Purwuokerto.
(C) Masyarakat berduyun-duyun ke
pasar. 25. Jangan dikira hanya ltalia yang
(D) Masyarakat Dayak Meratus terdiri mempunyai menara miring. Indonesia
atas berbagai balai adat. pun mempunyai bangunan bersejarah
(E) Masyarakat Dayak Meratus berjalan “berbau miring". Istana Tampaksiring,
kaki ke pasar. Bali, memang tidak miring sepeni Pisa,
tetapi letaknya di desa yang sejak zaman
23. Dalam bacaan di atas terdapat bahaula melagenda sebagai tanah
pernyataan implisit berikut, kecuali miring. Kala tampaksiring dicomot dari
(A) Warga Pedalaman Pegunungan bahasa setempat, tampak dan siring yang
Meratus tidak mengenal kendaraan artinya ”leIapak miring". Pembangunan
bermotor Israna Tampaksiring tak bisa dipisahkan
(B) Warga Pedalaman Pegunungan dari kiprah mantan Presiden RI, Bung
Meratus belum menggunakan listrik. Karno (BK). Konon, pemilihan tempat,
(C) Tradisionalisme di satu sisi adalah proses pembangunan, hingga finishing
kesedihan atas keterbelakangan. Istana Tampaksiring, semua melibatkan
(D) Pasar tradisional hanya ada satu kali BK. Ia sudah mengenal tanah
dalam seminggu. berpemandangan indah ini jauh sebelum
(E) Masyarakat Pegunungan Meratus disulap menjadi istana. Sebelum 1955
belum menikmati kebutuhan (istana dibangun 1957) BK sudah sering
sekunder dan tersier. istirahat di bekas tempat tetirah Raja
Gianyar yang dikeramatkan penduduk
24. Pada saat ini, kerisauan ternyata bukan setempat.
hanya milik para petani yang kesulitan Inti paragraf di atas adalah
mendapatkan pupuk. Bambang Riyanto, (A) lstana Tampaksiring yang
seorang perajin jenang dan tempe keripik merupakan bekas tempat tetirah
asal Purwokerto, bahkan harus pontang- Raya Gianyar sering digunakan oleh
panting mencari minyak tanah agar BK untuk istirahat.
usahanya itu tetap berlangsung. (B) Seperti Italia, Indonesia juga
Kerisauan jelas membayang ketika akan memiliki bangunan yang didirikan di
memasuki musim Iiburan akhir rahun lalu. atas tanah berkontur miring.
“Musim panen” bagi industri kecil (C) lstana Tampaksiring dibangun di
makanan oleh-oleh ini tentu tidak akan desa bertanah miring sehingga
dibiarkan Iewat begitu saja. Untuk bangunan ini juga “berbau miring".
mendapatkan minyak tanah yang cukup (D) Raja Gianyar membangun lstana
untuk produksi, ia harus mengerahkan Tampaksiring pada tahun 1957
seluruh anggota keluarganya, bahkan dengan melibatkan BK.
para tetangganya, untuk ikut antre (E) lstana Tampaksiring yang dibangun
membeli minyak tanah. di desa bertanah miring dan
Pikiran pokok paragraf di atas adalah

bimbelocean.com
UTUL UGM 99
PAKET 4
berpemandangan indah tidak bisa Jawa, Iaut bagian timur Pulau Sumatera.
dipisahkan dari BK. hingga memasuki muara dan hulu Sungai
Batanghari.
26. Adat Badui yang ketat tercermin dalam Dari bacaan di atas dapat disimpulkan
pepatah "Yang panjang tidak boleh bahwa
dipotong, yang pendek tidak boleh (A) Pengiriman arca Amoghapasa ke
ditambah". Hal ini sengaja direkankan Kerajaan Melayu menempuh
agar tradisi lama yang sesungguhnya perjalanan Jauh.
tetap dapat dipertahankan. Namun, tidak (B) Penyusuran itu untuk mengenang
berarti masyarakat Badui tidak menerima kebesaran Ekspedisi Pamalayu pada
sesuatu yang baru, atau sama sekali tahun 1286, yakni eskpedisi
menutup sesuatu dari dunia luar. Di sana diplomatik Kerajaan Singasari ke
sini terlihat orang Badui membuat Kerajaan Melayu.
pelbagai percobaan dengan mengubah (C) Ekspedisi PamaIayu tahun 1286
kombinasi atau variasi unsur-unsur adat menunjukkan bahwa Kerajaan
walaupun tetap dalam batas-batas yang Singasari memiliki kekuatan maritim
ditetapkan oleh tetua adat. yang kuat.
Inti paragraf di atas adalah (D) Napak tilas Ekspedisi Pamalayu itu
(A) Adat Badui menekankan kemurnian dilakukan dalam rangka
tradisi lama sehingga sedapat mengembangkan pariwisata
mungkin mereka menolak berkontak sepanjang rute pelayaran yang
dengan dunia luar. diewati utusan Raja Kertanegara ke
(B) Adat Badui berkembang dengan Kerajaan Melayu.
inovasi unsur-unsur adat dan (E) Napak tilas Ekspedisi Pamalayu itu
membuka diri terhadap campur dibiayai oleh Dinas Kepurbakalaan,
tangan dari dunia luar. Budaya, dan Pariwisata Jambi.
(C) Adat Badui mempertahankan tradisi
dan mengembangkannya dengan 28. Upacara wiwit termasuk salah satu tradisi
pelbagai percobaan atas variasi penting dalam dunia pertanian
unsur-unsur adat. masyarakat Jawa, khususnya dunia
(D) Adat Badui telah mengubah tradisi pertanian yang berkaitan dengan budi
lama dengan tradisi baru sehmgga daya tanaman pokok (padi). Hingga kini,
menjadi lebih maju dan modern. upacara tersebut masih bisa dijumpai di
(E) Adat Badui mengikuti batas-batas pelosok pedesaan walaupun semakin
yang ditetapkan oleh tetua adat yang sedikit yang melestarikannya. Upacara
menekankan kesederhanaan. wiwit semakin Iangka dijumpai, seiring
dengan munculnya banyak perubahan
27. Beberapa bulan Ialu Dinas nilai-nilai budaya dalam kehidupan sosial
Kepurbakalaan, Budaya, dan Parwisata komunitas petani, semakin sempitnya
Jambi memprakarsai napak tilas Iahan pertanian mereka, dan adanya
Ekspedisi Pamalayu. Ekspedisi perubahan sistem produksi pertanian di
penyusuran Sungai Batanghari ini Jawa.
bertujuan mencatat segala potensi yang Dari bacaan di atas dapat disimpulkan
ada di sepanjang rute untuk perencanaan bahwa
dan pengembangan pariwisata daerah (A) Langkanya upacara wiwit karena
tersebut Penyusuran itu mengingatkan sistem pertanian di Jawa mulai
kita pada Ekspedisi Pamalayu pada berubah.
tahun 1286, yakni ekspedisi diplomatik (B) Langkanya upacara wiwit karena
Kerajaan Singasari ke Kerajaan Melayu. banyaknya perubahan niai-nilai
Ekspedisi atas prakarsa Raja budaya.
Kertanegara itu disertai pengiriman arca (C) Upacara wiwit masih bisa dijumpai di
Amoghapasa. Menurut catatan sejarah, pelosok pedesaan di Jawa walaupun
pengiriman arca itu menempuh rute Laut

bimbelocean.com
100
UTUL UGM
PAKET 4

semakin sedikit yang sensus penduduk. Ditinjau dari segi


melestarikannya. ukurannya, komputer yang dipergunakan
(D) Upacara wiwit yang kini makin itu cukup bervariasi, mulai dari komputer
langka merupakan tradisi panting mini, misalnya jam tangan digital,
dalam budi daya tanaman padi di kalkulator dan telepon seluler; komputer
Jawa. mikro, misalnya laptop dan PC (personal
(E) Upacara wiwit termasuk salah satu computer); hingga komputer makro,
tradisi penting dalam dunia pertanian misalnya server dalam unit-unit LAN
masyarakat Jawa. (Local Area Network) dan mainframe
yang biasa dipergunakan di bank-bank
29. Wakil Presiden Bank of China, Zhu Min, atau perusahaan-perusahaan besar
menegaskan bahwa belanja konsumsi di lainnya.
Amerika Serikat, negara dengan ekonomi Paragraf di atas dapat diringkas menjadi
terbesar di dunia akan turun 10% tahun (A) Komputer menjadi bagian penting
ini akibat lesunya ekonomi global. dalam era globalisasi.
Konsumsi Amerika dalam beberapa (B) Komputer yang dipakai dalam
tahun ini diperkirakan menyusut tajam. kehidupan sehari-hari memiliki
Penurunan belanja negeri Obama itu ukuran bervariasi.
diprediksi akan Iebih besar Iagi dalam (C) Komputer digunakan dalam berbagai
tiga atau empat tahun mendatang. bentuk dan bidang kehidupan.
Padahal, Amerika dan Jepang (D) Komputer menjadi perangkat penting
merupakan pasar ekspor terbesar produk dalam mempermudah pekerjaan
Cina. Akibatnya, krisis di Amerika manusia.
tersebut membuat pertumbuhan ekonomi (E) Komputer digunakan di bank-bank
Cina mulai melambat beberapa bulan ini. dan perusahaan-perusahaan besar.
Imbas krisis itu membuat banyak
perusahaan di Negeri Tirai Bambu 31. Kalimat yang efektif adalah
menjadi goyah. (A) Data perilaku manajerial yang
Berdasarkan isi bacaan di atas, dikumpulkan oleh peneliti dan
pernyataan berikut ini benar, kecuali tenaga teknisi dari lokasi penelitian
(A) Cina banyak tergantung diperoleh dengan cara
perdagangannya pada Amerika mewawancarai dan menyebarkan
Serikat. kuesioner kepada informan.
(B) Penurunan belanja Amerika sebesar (B) Data perilaku manajenal yang
10% akan diikuti lagi dengan dikumpulkan oleh peneliti dan
penurunan konsumsi negara itu di dibantu tenaga teknisi dari Iokasi
tahun-tahun berikutnya. penelitian diperoleh dengan metode
(C) Belanja konsumsi sebesar-besarnya wawancara dan kuesioner kepada
merupakan satu-satunya jalan untuk informan.
memperbaiki ekonomi Amerika. (C) Data perilaku manajerial yang
(D) Banyak perusahaan di Cina menjadi dikumpulkan oleh peneliti dengan
tidak stabil setelah pembelian asisten tenaga teknisi dari lokasi
produknya berkurang di Amerika. penelitian diperoleh dengan metode
(E) Zhu Min berpendapat bahwa krisis di wawancara dan kuesioner kepada
Amerika membuat pertumbuhan informan.
ekonomi Cina tidak begitu signifikan (D) Data perilaku manajerial yang
beberapa bulan ini. dikumpulkan peneliti dan dibantu
tenaga teknisi dengan datang
30. Dalam era globalisasi ini komputer telah Iangsung ke lokasi penelitian
dipergunakan dalam banyak bidang diperoleh dengan metode
kehidupan, misalnya dalam bidang wawancara dan kuesioner kepada
kesehatan, pertahanan dan keamanan, intorman.
termonuklir, perbankan, transportasi, dan

bimbelocean.com
UTUL UGM 101
PAKET 4
(E) Data perilaku manajerial yang (A) Dengan teknik tersebut para
dikumpulkan oleh peneliti dan astronom dapat memperkirakan
tenaga teknisi diperoleh lewat ukuran planet meskipun belum
wawancara dan kuesioner. diketahui massanya, namun
diperkirakan antara 5,7 hingga 11
32. Kalimat berikut tidak baku, kecuali massa bumi.
(A) Dengan adanya “liberalisasi” sistem (B) Landasan pacu Bandara Ahmad
pemilihan semakin membuat profit Yani Semarang tergenang
DPR mendatang sulit diprediksi, dan sepanjang 500 meter dan harus
tidak mempunyai jaminan kinerja menggunakan 23 pompa air untuk
mereka semakin membaik. menyurutkan genangan.
(B) Menurut diagnosa dokter, pasien itu (C) Bandara menunda semua
positif mengidap penyakit flu burung. penerbangan sejak Minggu pagi
(C) Dalam Bab I penelitian ini hingga pukul 21.00 karena kondisi
membicarakan tentang latar landasan pacu masih buruk.
belakang, masalah, dan landasan (D) Komisi pemilihan umum berencana
teori. mengatur penetapan caion terpilih
(D) Jadi, di negara kita tidak dikenal yang berpihak pada calon
diskriminasi terhadap warga negara perempuan, jika partai politik meraih
yang menyandang cacat. tiga kursi DPR/DPHD, salah satunya
(E) Untuk merenovasi museum itu mesti diberikan kepada calon
memerlukan dana yang tidak sedikit. perempuan yang perolehan
suaranya terbanyak.
33. Yang merupakan kalimat baku adalah (E) Berdasarkan pada data di Dinas
(A) Buku ini terdapat istilah-istilah Bina Marga Pemkot Surabaya, maka
kekerabatan pada orang Jawa dan titik-titik genangan itu menyebar
Batak dalam susunan masyarakat hampir di seluruh wilayah Surabaya
yang ditulis berdasarkan ilmu mulai sisi paiing barat, timur, hingga
antropologi. ke selatan.
(B) Dari beberapa pokok persoalan yang
diberikan untuk rnembandingkan dua 35. Beberapa waktu Ialu berita tentang susu
atau lebih dialek, antara lain dalam bermelamin, biskuit beracun, dan bahan
bidang fonetik atau semantik. kosmetik yang mengandung bahan
(C) Adapun yang akan saya uraikan di merkuri dan hidrokinon dengan cepat
sini ialah kebersihan dan kesehatan, tersebar melalui media massa sehingga
saya terdorong untuk meresahkan pedagang dan konsumen
mengemukakannya, karena sering yang memiliki, menyimpan,
dilalaikan orang dan sesungguhnya memperjualbelikan, atau
kebersihan dan kesehatan itu perlu, menggunakannya.
sebab dengan bersih tentu akan lnti kalimat di atas adalah
menjadi sehat. (A) Berita meresahkan.
(D) Karena nilai yang didapatkan lebih (B) Berita tersebar.
besar dari penolakan, maka (C) Bahan beracun tersebar.
hipotesis nihil ditolak. (D) Pedagang dan konsumen resah.
(E) Jika data yang diolah tidak (E) Pedagang dan konsumen
memenuhi syarat, baik dari segi menggunakan.
kuantitas maupun kualitasnya, hasil
penelitian di lapangan itu tidak dapat 36. Dalam konferensi internasional
dipertanggungjawabkan secara kebudayaan Melayu itu tidak diputuskan
ilmiah. tempat penyelenggaraan konferensi
berikutnya.
34. Kalimat berikut tidak baku, kecuali Kalimat yang memiliki pola yang sama
dengan pola kalimat di atas adalah

bimbelocean.com
102
UTUL UGM
PAKET 4

(A) Obama, Presiden Amerika yang baru (C) Pemerintah daerah tidak boleh
terpilih itu, mengawali kariernya dari hanya berpangku tangan ketika
bawah. melihat rakyatnya menderita karena
(B) Seperti telah kita ketahui, banjir.
perekonomian Amerika Serikat (D) Sulit berdiskusi pada orang yang
sangat sangat berpengaruh. keras kepala.
(C) Berdasarkan pengarahan pimpinan, (E) Pemimpin bertangan besi itu
pembangunan gedung baru itu dapat kemarin meninggal dunia.
dilakukan secara bertahap.
(D) Kita sadari bahwa bencana alam 39. lbu Ria adalah klien saya. Masih gesit,
dapat datang kapan saja tanpa kita masih baca puisi bersama-sama teman
duga. seumurnya. Sekitar dua minggu
(E) Telah kami bicarakan dalam rapat menjelang Natal saya bertemu dia di
terdahulu bahwa kenaikan gaji kantornya. Dia bercerita bahwa anaknya
pegawai baru bisa dilaksanakan yang akan membuat garage sale
bulan depan. mengusulkan melego meja makan dan
meja di ruang tamu rumahnya.
37. Kata jual yang bermakna literal terdapat Yang dimaksud dengan garage sale
dalam kalimat adalah
(A) Sejarah membuktikan bahwa (A) obralan.
sebagian pribumi menjual (B) pameran makanan.
bangsanya sendiri pada tangan (C) pasta di gudang.
penjajah (D) ulang tahun.
(B) Kau tidak akan mendapat jodoh (E) reuni.
kalau terlalu jual mahal
(C) Bagi sebagian anak muda, blog 40. Ekspor timah dari Indonesia yang
adalah media untuk jual tampang. merupakan penghasil timah terbesar
(D) Para penipu berkedok hadiah kedua di dunia setelah Cina semakin
biasanya menjual nama perusahaan berkurang setelah pemerintah menutup
tertentu. pertambangan ilegal di Bangka dan
(E) Oknum paramedis di RS swasta itu Belitung. Upaya ini menjadikan timah
terlibat aksi jual beli gigi palsu sebagai salah salu komoditas yang
berperforma terbaik di London Metal
38. Semua pimpinan partai kini sedang Exchange pada tahun-tahun terakhir.
memeras otak untuk memenangkan Kata upaya dalam teks ini sangat tepat
pemilu. bila diganti dengan
Gabungan kata yang semakna dengan (A) usaha.
memeras otak adalah (B) ikhtiar.
(A) Penyanyi itu memamerkan suara (C) kebijakan.
emasnya. (D) kebijaksanaan.
(B) Buruh tani itu setiap hari (E) tindakan.
membanting tulang.
BAHASA INGGRIS

I. Read the following passage carefully. Answer the questions that follow, and choose the best
answer to each question.

In today's fast-paced, cyber-linked world, it would be hard to imagine working without the internet. Most
of us would probably feel pretty desperate. Well, how about trying to live a day without water, a real every-day
nightmare for some people. I know how it feels.
After suffering from one-day emergency shutdown of my local water supply, which prevented me from
5 getting a single drop of water from the tap, let alone showering or flushing the toilet, I started to think how ironic it

bimbelocean.com
UTUL UGM 103
PAKET 4
is that on a planet where water covers 71 percent of its surface, I suddenly had no.
But when I learned that just over 2 percent of the total volume of water in the global cycle is fresh water,
and that most of that is locked up as polar ice caps and in glaciers, I became horrified.
Yes, horrified at the thought of more than 6 billion people sharing this small amount of water that is
10 available. It made me wonder : is there anything I can do to use this precious resource more wisely?
That question leads me to one of my favorite lines from the movie Evan Almighty." How do we change
the world? By doing one act of random kindness at a time'. It's a message that applies to all aspects of life, including
in conserving water.
15 So, to begin with, I started to list some of my water-consuming activities. And from various sources l
found out that taking a bath is the biggest water consumer (about 45 percent of the total daily consumption),
followed by toilet flushing (25 percent) and kitchen activities (15 percent).
A survey conducted in 2006 by the Directorate of Water Development of the Public Works Ministry
showed that the average daily use of fresh water in Indonesia was 144 liters per person. Based on that figure,
20 bathing, flushing and kitchen activities represent 65, 36, and 21.5 liters respectively or 122.5 liters in total.
Assuming the Indonesian population represents around 230 million people, all having equal access to 122.5 liters per
day, can you imagine what happens to that 2% of global fresh water? You do the math!
Those numbers have given me a stronger urge to be more efficient with my personal consumption of fresh
water. So, I started off with my bathroom. First, I readjusted the showerhead to a tow-flow version and took shorter
25 showers, less than 5 minutes long, since that could already reduce my water use approximately by half.
Second, I tried a simple trick suggested by a friend to save water in the toilet tank, by placing a one-liter
plastic bottle filled with water in the tank. This way, I save one liter of fresh water every time I flush the toilet. If the
toilet is used 10 times a day, I have already saved 10 liters of fresh water.
And since I am not really a kitchen type of person, I believe my water spending in that department
30 wouldn't exceed 10 liters per day.
So in total, I could save around 56.6 liters per day from bathroom and kitchen activities. Not bad for a
start, as there are still more things that can be done to efficiently use water.
'Before the Water Runs Dry," The Jakarta Post, Weekender Magazine, May, 2008, 23.

41. What is the main idea of the passage? 42. The pronoun "it" in line 5 refers to
(A) A nightmare for some people in (A) alter suffering from one-day
Indonesia when living a day without emergency shutdown of my local
water water supply
(B) The importance of people's (B) which prevented me from getting a
awareness in using the precious single drop of water from the tap
resource wisely (C) that on a planet where water covers
(C) The need to start saving the use of 71 percent of its surface, I suddenly
water in the toilet tank and bathroom had no
(D) The stronger urge to make a list of (D) that just over 2 percent of the total
water-consuming activities at home. volume of water in the global cycle is
(E) The need to make a personal fresh water
consumption of fresh water more (E) that most of that is locked up as polar
efficient ice caps and in glaciers, I became
horrified

bimbelocean.com
104
UTUL UGM
PAKET 4

47. The pronoun “this” in line 27 refers lo


43. Which of the following is closest in which of the following?
meaning to 'locked up' in line 8? (A) A simple trick suggested by a friend to
(A) restricted save water in the toilet tank
(B) kept (B) My water spending in that department
(C) limited wouldn't exceed 10 liters per day
(D) bordered (C) By placing a one-liter plastic bottle
(E) secured filled with water in the tank
(D) I save one liter of fresh water every
44. According to the passage, which of the time I flush the toilet
following best describes the word “urge” in line (E) And since I am not really a kitchen
23? type of person
(A) wish
(B) longing 48. Which of the following has the same
(C) desire meaning as the word "exceed' in line 30?
(D) craving (A) transcend
(E) aspiration (B) rise above
(C) do better than
45. The authors purpose in this passage is to (D) overtake
(A) explain how difficult it can be lo live a (E) go beyond
day without fresh water.
(B) present the data about the total 49. According to the passage, all of the
volume of fresh water on the planet following are true about saving fresh water
(C) make the reader aware of using water EXCEPT
more wisely for personal use. (A) washing fruits under running water.
(D) cite statistics about the average daily (B) taking a shower rather than using a
use of fresh water in lndonesia, bathtub.
(E) warn that six billion people share the (C) brushing teeth with a glass of water.
small amount of fresh water. (D) using a new type of toilet flush.
(E) Washing dishes in a soaking sink.
46. Where in the passage does the author
mention how the author has been inspired lo 50. The tone of this passage could best be
save fresh water? described as
(A) lines 1-2 (A) concerned
(B) lines 4-6 (B) calm
(C) lines 9-10 (C) emotional
(D) lines11-13 (D) informative
(E) lines 18-20 (E) exaggerated

bimbelocean.com
UTUL UGM 105
PAKET 4
II. The text below has incomplete sentences. Choose the one word or phrase from each number that best
completes the sentence.

Depending on your point of view, the business of bargaining can be: a method of solidifying
your position in local economic community; a pleasant pastime; a technique for saving money; a
social vehicle for requesting and receiving favours; an example of unevolved and unsophisticated
economics.
Judge gently! Throughout Asia for many, (51) ________ the price of goods and services is
an art form as complicated as a dance, as expressive as a line painting or poem, as affirming as the
practice of religion. Bargaining (52) ______ by the practitioners is definitely a method for establishing
and solidifying one's status in economic community-an item may have a variety of prices, (53)
_______ in the eye of the merchant the correct price for each of his customers. To offer everyone the
same price removes the opportunity to ‘do a favor for my special customer’ (54) ________ the
opportunity to blatantly overcharge an unsuspecting buyer!
(55) ______ in Asia who do not smile when a preferred price is offered to them as a
'courtesy'. Equally flattering is to be the recipient of an extra fruit, an extra spray of flowers, an
additional potted plant as a favor, a small gift.
As one bargains and prices are bandied back and forth, there is ample time (56)_______
the progress of children, or share some local gossip. Of course one must feel a sense of conquest at
the amount of money saved, but most people would admit (57) _____ not the real purpose of
bargaining.
The typical westerner-in-a-hurry is confused and vexed at (58) ______business quickly.
Many foreigners will spend much time looking for the 'fixed price' stores and embrace the goods and
services (59) _______ like familiar friends.'Unevolved and lacking sophistication' is the general snarl
of the westerner who looks limply about for the lamiliar green screen of the checkout computerized
cash register (60) _____ a smiling man digging in twelve pockets for the bits of change.
Adapted from Draine Hall, Culture Shock: A guide to Customs and Etiquette: Indonesia, 63-65.

bimbelocean.com
106
UTUL UGM
PAKET 4

51. (A) over bargain .57 (A) this is


(B) over bargaining (B) it is
(C) to bargain over (C) that is
(D) bargaining over (D) there are
(E) bargain over (E) they are

52. (A) as understood 58. (A) resisting to do


(B) is understood (B) resisting to doing
(C) understood (C) having resisted to doing
(D) it is understood (D) the resistance to doing
(E) understanding (E) the resistance to do

53. (A) each reflection 59. (A) here


(B) (B) the reflection of each (B) then
(C) (C) reflecting each (C) where
(D) each reflects (D) there
(E) each reflecting (E) look

54. (A) and equally 60.. (A) encounters only and


. (B) equally and (B) (B) only encounters and
(C) equal and (C) (C) encounters and only
(D) and equals (D) (D) only and encounters
(E) equally (E) and only encounters

55. (A) Very few of them


(B) Few are those
(C) Those are few
(D) A few of them
(E) A Few are there

56. (A) asking about


(B) asking for
(C) to ask about
(D) to ask for
(E) which ask about

bimbelocean.com
UTUL UGM 107
PAKET 5

PAKET TES KEMAMPUAN DASAR UMUM

5
WAKTU : 100 MENIT
JUMLAH SOAL : 60

MATEMATIKA DASAR

1. Jika 2𝑥 = 2 − √3, maka 2+√3


log 4𝑥 = ⋯ (E) (0, 10)
(A) –2
(B) −
1 5. Salah satu nilai x yang memenuhi sistem
2
persamaan xy + y2 = 0 dan x – 2y = 3
(C) 1
1
adalah ...
(D) (A) –1
2
(E) 2 (B) 0
(C) 1
𝑥+𝑦 𝑥−𝑦
2. Jika log 2 = 𝑎 dan log 8 = 𝑏, (D) 2
4
dengan 0 <x<y, maka log(𝑥 2 − 𝑦 2 ) = (E) 4

𝑎+3𝑏 𝑥 𝑦 5
(A) 6. Jika x dan y memenuhi + 𝑥 = 2 dan x –
𝑎𝑏 𝑦
𝑎+𝑏
(B) 3y = 1 maka 5x + 5y = ...
2𝑎𝑏
𝑎+𝑏 (A) –15 atau –3
(C) 3
4𝑎𝑏
3𝑎+𝑏
(B) –3 atau− 5
(D) 2𝑎𝑏 (C) –3 atau 15
3𝑎+𝑏 3
(E) (D) 3 atau5
4𝑎𝑏
(E) 3 atau 15
𝑥 2 +𝑎𝑥 𝑚+2
3. Jika akar-akar persamaan =
𝑏𝑥−2 𝑚−2
berlawanan dan ab maka nilai m adalah 7. Himpunan penyelesaian dari √2𝑥 + 2 −
... √6𝑥 − 8 ≥ 0 adalah ...
𝑎+𝑏 (A) {𝑥|𝑥 ≥ −1}
(A) 𝑎−𝑏 4
2(𝑎+𝑏) (B) {𝑥|𝑥 ≥ 3}
(B) 𝑎−𝑏 5
(C) 𝑎 + 𝑏 (C) {𝑥|𝑥 ≤ 2}
2(𝑏+𝑎) 5
(D) (D) {𝑥|𝑥 ≥ }
𝑏−𝑎 2
𝑏+𝑎 4 5
(E) (E) {𝑥| 3 ≤ 𝑥 ≤ 2}
𝑏−𝑎

4. Grafik fungsi kuadrat y = f(x) mempunyai 8. Nilai minimum f(x, y) = 3 + 4x – 5y untuk x


titik puncak (–1, 8) dan memotong sumbu dan y yang memenuhi
X di (x1, 0) dan (x2, 0). Jika x1x2 = –3, maka –x + y 1
grafik tersebut memotong sumbu Y di ... x + 2y 5
(A) (0, –10) 2x + y 10
(B) (0, –2) adalah ...
(C) (0, 4) (A) –19
(B) –6
(C) –5
(D) (0, 6) (D) –3

bimbelocean.com
108
UTUL UGM
PAKET 5

(E) 23 (C) 0°x 20°atau 100°<x< 140°


(D) 20°<x< 100°atau 140°<x< 180°
9. Tiga bilangan membentuk barisan (E) 30°<x< 100°atau 140°<x< 180°
geometri dengan rasio positif. Jika
bilangan kedua ditambah 4, diperoleh 14. Dua kotak masing-masing berisi lima bola
barisan aritmatika. Jika bilangan pertama yang diberi nomor 2, 3, 5, 7, dan 8. Dari
adalah 2, maka jumlah ketiga bilangan setiap kotak diambil sebuah bola. Peluang
semula adalah ... terambil sedikitnya satu bola dengan
(A) 20 nomor 3 atau 5 adalah ...
(B) 22 2
(A) 5
(C) 24 3
(D) 26 (B) 5
16
(E) 28 (C) 25
18
(D)
10. Diketahui Un adalah suku ke-n suatu 25
4
barisan aritmatika. Jika untuk setiap (E) 5
bilangan asli n, nilai Un – Un-2 sama
dengan tiga kali suku pertama dan 15. Amin telah mengikuti test matematika
𝑈3 +𝑈11 𝑈1 +𝑈3
= mana U10 = ... sebanyak 8 kali dari 12 kali test yang ada
𝑈9 −𝑈5 3
87 dengan nilai rata-rata 6,5. Jika untuk
(A) 10 seluruh test, Amin ingin mendapat rata-
29
(B) rata nilai minimal 7, maka untuk 4 test
3
(C) 21 yang tersisa, Amin harus mendapatkan
(D) 29 nilai rata-rata minimal ...
(E) 32 (A) 7,9
(B) 8
1 2 (C) 8,1
11. Jika matriks 𝑃 = ( ) dan I matriks
3 2 (D) 8,2
identitas yang berorder sama dengan P (E) 8,5
maka hasil kali akar persamaan det(P – xI)
= 0 adalah ... 1
16. Jika 𝑓 (𝑥 ) = dan (𝑓 ∘ 𝑔)(𝑥) =
√𝑥 2 −2
(A) –6
1
(B) –4 maka g(x + 2) = ...
√𝑥 2 +6𝑥+7
(C) –3 1
(A)
(D) 3 𝑥+3
1
(E) 4 (B) 𝑥−2
(C) x – 2
12. Diketahui segitiga ABC lancip dengan AB (D) x + 3
= 2√2, BC = 2, dan ABC = . Jika sin  = (E) x + 5
1
3
, maka AC = ...
6 2
1 17. Nilai lim (𝑥 2−𝑥−2 − 𝑥−2) sama dengan ...
(A) 3
√3 𝑥→2
(A) –1
(B) √6 2
2 (B) − 3
(C) 3
√3
1
3 (C) − 3
(D) 2
√2
1
1 (D)
(E) 2
√2 3
2
(E) 3
1
13. Nilai x yang memenuhi cos 3x> untuk
2
𝑥2
0°x 180° adalah ... 18. Kurva 𝑦 = 𝑥−1 mencapai maksimum relatif
(A) 0°<x< 20°atau 90°<x< 140° di ...
(B) 0°x< 20°atau 100°<x< 140° (A) (2, 4)

bimbelocean.com
UTUL UGM 109
PAKET 5
(B) (0, 0) (E) 5
4
(C) (2, 3)
9
(D) (3, 2)
20. Diketahui akar-akar persamaan kuadrat
4
(E) (– 2, − 3) ax2 – bx + 1 = 0 adalah p dan 2p, p
bilangan bulat. Jika 1, a, b merupakan 3
19. Garis singgung kurva y = x4 – x2 di titik (1, suku berurutan suatu barisan aritmatika,
0) dan (–1, 0) berpotongan di (a, b). Nilai a maka p = ...
– b = ... (A) 2
(A) 1 (B) 1
(B) 2 (C) –1
(C) 3 (D) –2
(D) 4 (E) –4

BAHASA INDONESIA

21. Hingga saat ini, Amerika Serikat dan Cina sebagai sumber energi bagi pembangkit
masih menduduki peringkat teratas listrik.
penyumbang emisi dunia. Tingginya tingkat (C) Tercatat sebanyak 46% produksi batu
emisi kedua negara tersebut salah satunya bara tua dunia dihasilkan Cina.
diakibatkan oleh tingginya penggunaan batu (D) Amerika Serikat dan Clna merupakan
bara sebagai sumber energi bagi produsen terbesar emisi karbondioksida
pembangkit Iistrik. Amerika Serikat masih karena tingginya penggunaan batu bara
memiliki cadangan batu bara sebesar 264 sebagai pambangkit tenaga lisrik di
miliar ton, dan cukup untuk memenuhi negara tersebut.
kebutuhan energi 225 tahun ke depan (E) Setiap hari sekitar 14.000 ton batu bara
dengan Iaju saat ini. Batu bara yang murah dibakar di pembangkit lisirik Hunter,
dan mudah diperoleh di kawasan Amerika Utah yang memasok lisirik ke wilayah
Serikat menghasilkan setengah daya listrik AS bagian barat.
negara rersebut. Setiap hari sekitar 14.000
ton batu bara dibakar di pembangkit listrik 22. Pemerintah diharapkan segera
Hunter, Utah yang memasok Iistrik ke membentuk panel khusus untuk mengaudit
wilayah AS bagian barat. Cina memproduksi penambangan batu bara di wilayah
Iebih dari 18 miliar ton karbandioksida per Kalimantan. Audit itu diperlukan untuk
tahun. Di Cina, Ieblh dari 80% Iistrik negara mengetahui sebaran penambangan,
dihasilkan oleh pembangkit bertenaga batu kesesuaian lahan, dan kepatuhan pemilik
bara. Tercatat sebenyak 45% produksi batu kuasa pertambangan pada standar
bara tua dunia dihasilkan Cina. Bahkan, Iingkungan. Sejauh ini pemerintah daerah
pada awal tahun 2008, dua buah dan pusat dinilai saling tunggu untuk
pembangkit listrik tenaga batu bara mengambll Iangkah yang harus dilakukan
dioperasikan setiap minggunya untuk guna mengatasi masalah penambangan
mengimbangi pesatnya Iaju pertumbuhan yang merambah hutan konservasi di
ekonomi Cina. Kalimantan. Pemerintah daerah menuding
lnti paragraf di atas adalah pemerintah pusat seenaknya
(A) Hingga saat ini, Amerika Serikat dan mengeluarkan izin dan menetapkan
Cina masih menduduki peringkat wilayah hutan, sementara pemerintah
teratas negara penyumbang emisi pusat menuduh pemda tidak terkendali
dunia. memberi kuasa pertambangan.
(B) Tingginya tingkat emisi di AS dan Cina Yang tersebut di bawah ini dikemukakan
salah satunya diakibatkan oleh dalam bacaan di atas, kecuali
tingginya penggunaan batu bara (A) Penambangan batu bara meresahkan.

bimbelocean.com
110
UTUL UGM
PAKET 5
(B) Penambangan mengabaikan standar (E) Sel berada di daerah getah bening,
lingkungan. darah, dan jaringan periferal, seperti
(C) Sebaran penambangan tidak kulit dan paru-paru.
transparan.
(D) Penambangan batu bara dibatasi. 24. Upaya meniadakan nyamuk penyebab
(E) Pemerintah pusat bebas mengeluarkan demam berdarah dengue (DBD) hanya
izin. satu, yaitu menyuluh masyarakat. Bukan
air comberan atau sampah, melainkan air
23. Pemberian vaksin Iewat goresan di kulit jernih tergenanglah yang merupakan
ternyata memberlkan respons imun yang tempat nyamuk aedes bersamng. Oleh
Iebih kuat dibandingkan dengan vaksin karena itu, segala yang berpotensi menjadi
yang disuntikkan. Para peneliti juga sarang perindukan nyamuk wajib
menemukan banwa vaksin Iewat goresan disingkirkan. Kebijakan mencegah bukan
tubuh akan 100 kali Iebih cepat membentuk dengan menyemprot (fogging), melainkan
zat kekebalan. Pemberian vaksin dengan membunuh nyamuk dengan Iarvasida,
cara menggores kulit tersebut pertama kali mengingat umur nyamuk dewasa tidak
dikenal pada dua abad lalu, yakni saat Iebih panjang daripada jentiknya. Nyamuk
pemberian vaksin pencegah cacar dewasa baru disemprot kalau sudah ada
(smallpox). Namun, kini sebagian besar yang terjangkit.
metode imunisasi modern diberikan Iewat lnti paragraf di atas adalah
injeksi. Dalam beberapa tahap penelitian, (A) Pemerintah harus memberi
para ahli dari Brigham and Women's penyuluhan kepada masyarakat.
Hospital, Amenka Serikat, juga (B) Penanganan wabah demam berdarah
menemukan bahwa cara pemberian vaksin dengue (DBD) selama ini keliru.
yang dilakukan dengan menggores (C) Masyarakat harus diberi tahu cara
tersebut akan membuat memori sel T, sel mencegah DBD secara benar.
yang menjaga sistem imun untuk melawan (D) Hal yang penting dilakukan dalam
virus penyebab penyakit, meningkat. Sel T penanganan DBD adalah
diduga Iebih panting daripada sel imun, Sel menghilangkan jentik.
ini berada di daerah kelenjar getah bening, (E) Fogging yang selama ini dilakukan
darah, dan jaringan periferal, seperti kulit tidak bisa berhasil mencegah DBD.
dan paru-paru.
Berdasarkan Isi bacaan di atas, pernyataan 25. Seni, apa pun bentuknya, adalah media
berikut ini benar, kecuali ekspresi yang secara estetik membangun
(A) Para peneliti juga menemukan bahwa watak dan karakter manusia Indonesia
vaksin lewat goresan tubuh akan 100 dalam setiap wujudnya. Tak jarang,
kali Iebih cepat membentuk kekebalan. identitas “manusia Indanesia’ ditemukan
(B) Brigham and Women’s Hospital adalah bukan dari mata pelajaran budi pekerti,
tempat para periset meyakini kewarganegaraan, atau wawasan
temuannya bahwa sel T ternyata Iebih nusantara, melainkan dari sebuah Iakon
panting daripada sel imun. yang dipentaskan Bengkel Teater, Teater
(C) Metode imunisasi modern yang kini Koma, atau Teater Gandrik. “Kepribadian
Iebih populer dan paling efektif Indonesia” juga ditemukan pada novel-
dilakukan adalah pemberian vaksin novel Andrea Hirata, Umar Kayam,
melalui cara injeksi. Mochtar Lubis, atau sajak-sajak Chairil
(D) Cara pemberian vaksin yang dilakukan Anwar, W.S. Rendra, Taufik Ismail, dan
dengan menggores tersebut akan sederet nama lain di negeri ini. Dengan
membuat sistem memori sel T, sel yang demikian, sudah selayaknya negara
menjaga sistem imun untuk melawan berperan dalam menumbuhkan kesadaran
virus penyebab penyakit, meningkat. estetik guna menghargai produk-produk
seni yang lahir di negeri ini.
Inti paragraf di atas adalah

bimbelocean.com
UTUL UGM 111
PAKET 5
(A) Indonesia mempunyai sejumlah nama yang menyebabkan perempuan Nabire
besar di bidang kesenian yang layak rentan kekerasan dalam Iingkup domestik.
dibanggakan. Setelah berkeluarga, tugas perempuan
(B) Seni yang sudah terbukti membentuk kian berat. Selain melahirkan dan
kepribadian bangsa perlu mendapat mengurus anak, mereka juga harus
perhatian pemerintah. mengelola kebun. Di pedalaman dengan
(C) ldentitas Indonesia bisa ditemui pada mudah dijumpai seorang ibu seraya
pentas-pentas teater dan karya sastra. membopong anak di pundaknya juga
(D) Karya seni adalah pembentuk membawa tas rajut berisi ubi serta
kepribadian yang lebih baik memanggul sekop atau kapak.
dibandingkan dengan mata pelajaran di lnti paragraf di atas adalah
sekolah. (A) Perempuan di Papua mempunyai
(E) Mata pelajaran di sekolah tidak berhasil tugas mengurus anak dan mencari
membentuk kepribadian bangsa makan.
Indonesia. (B) Dalam rumah tangga, kedudukan
perempuan di Nabire di bawah laki-iaki
26. Masa lalu peradaban bukan hanya dan rentan terhadap kekerasan.
peristiwa dan pengalaman yang Iewat. (C) Laki-laki di Nabire leluasa berbuat apa
Masa lalu merupakan bagian integral yang saja terhadap istrinya karena memsa
bertali-temali dengan masa sekarang dan sudah memberi mas kawin.
masa depan. Masa lalu, sekarang, dan (D) Perempuan di Nabire mempunyai
masa nanti tidak bersifat linear. Dalam tiga peran ganda sekaligus beban ganda.
entitas itu terletak harapan dan (E) Posisi setara antara Iaki-Iaki dan
keputusasaan, keberhasilan dan perempuan dalam masyarakat Nabire
kegagalan. Belajar dari masa lalu tidak suiit ditemukan.
hanya bisa ditempuh Iewat warisan
pemikiran dan tradisi tokoh bangsa, tetapi 28. Kadar kolesterol tinggi banyak dialaml oleh
juga melalui warisan yang berupa benda orang berbagai usia dan risikonya terus
mati, yang ada di atas permukaan tanah meningkat seiring dengan bertambahnya
atau yang ada di kolong tanah. Kita bangga usia. Kolesterol tinggi ini umumnya
dangan Muora Jambi atau Borobudur yang disebabkan oleh gaya hidup tidak sehat,
termasuk kategori warisan dunia, sekadar seperti makan makanan tidak sehat yang
memungut dua contoh. berlemak jenuh dan berkolesterol tinggi.
Inti paragraf di atas adalah Merokok, kurang olahraga, dan stres pun
(A) Masa lalu merupakan sejarah dari masa memicu naiknya kadar kolesterol. Selain
kini. flu, faktor genetik atau keturunan juga patut
(B) Zaman selalu berkembang. diwaspadai, seperti orang dengan
(C) Kita harus belajar dari masa lalu jika hipertensi, obesitas, diabetes melitus, serta
ingin berhasil. mempunyai riwayat keluarga sekandung,
(D) Masa lalu merupakan hal yang penting karena potensial menjadi pengidap
untuk dipelajari. hipenensi dan penyakit jantung koroner.
(E) Benda mati di permukaan bumi Paragraf di atas dapat diringkas menjadi
merupakan warisan yang berharga. (A) Kolesterol adalah penyakit yang umum
diderita.
27. Adat yang mengharuskan pria membayar (B) Kolesterol meningkat seiring dengan
dan melunasi mas kawin tanpa disadari bertambahnya usia.
menempatkan perempuan Nabire, Papua, (C) Kolesterol disebabkan oleh gaya hidup
sebagai komoditas. Apalagi yang tidak sehat.
menentukan besarnya mas kawin bukan (D) Kolesterol merupakan penyakit
perempuan, meiainkan paman. Sejak awal turunan.
posisi setara antara perempuan dan laki- (E) Kolesterol menyebabkan obesitas.
laki daiam perkawinan tidak terjadi. Itulah

bimbelocean.com
112
UTUL UGM
PAKET 5
29. Bisnis menuntut inovasi. Inovasi tidak nama” tampaknya tidak berlaku bagi
selalu merupakan penemuan baru atau masyarakat dan perlu direnungkan
teknologi baru, tetapl dapat juga aplikasi kembali kebenarannya.
baru dari hal-hal yang sudah dianggap Paragraf ini membahas
kuno. Bisnis mengandung kebutuhan yang (A) pemberian nama diri.
terus-menerus untuk menjajaki sekeliling (B) pertimbangan penamaan diri.
kita, menguji setiap ujung Iemah yang (C) nama diri sebagai identitas.
mungkin ditajamkan sebagai sebuah (D) pandangan Shakespeare
tombak bagi sebuah pusat Iaba baru. (E) kerumitan penamaan diri.
Sementara itu, sekolah bisnis biasanya
hanya mengajarkan hal-hal yang telah 32. Hingga kini, kurangnya tenaga medis dan
terjadi. kesenjangan fasilitas kesehatan, terutama
Ide pokok paragraf di atas adalah di pedesaan, masih terjadi. ..., pemerintah
(A) teknologi baru dalam bisnis. perlu memberikan dorongan pada para
(B) perlunya sekolah dalam calon dokter dan paramedis agar bersedia
pengembangan bisnis. ditempatkan di berbagai puskesmas atau
(C) penerapan hal yang baru dari sesuatu rumah sakit di daerah terpencil. ...,
yang sudah ada. pemerintah perlu memberikan perhatian
(D) perlunya inovasi untuk meraih laba den penghargaan yang Iayak pada tenaga
dalarn bisnls. medis yang mengabdi di daerah tetinggal.
(E) perlunya pembaruan dalam bisnis. ..., diharapkan, ... masyarakat desa ... kota
sama-sama menikmati pelayanan
30. Anda ingin berwisata alam pedesaan kesehatan.
sambil menikmati hidangan istimewa, Kata yang tepat untuk melengkapi titik-titik
memandang hamparan sawah Iuas dalam paragraf di atas berturut-turut
terbentang dan deretan pohon kelapa adalah
yang membingkai tepian desa, mendengar (A) karena, sehingga, adapun, baik,
suara gemericik air dan menyaksikan ikan- ataupun.
ikan yang berenang babas di kolam alam, (B) oleh karana itu, untuk itu, dengan
sambil menikmati masakan udang galah demikian, baik, maupun.
yang Iezat? (C) di samping itu, sehingga, oleh sebab
Inti kalimat di atas adalah itu, baik.
(A) Anda ingin pergi ke desa? (D) selain itu, sehingga, maka, baik,
(B) Anda ingin menikmati pemandangan ataupun.
alam? (E) sedangkan, sehingga, adapun, baik,
(C) Anda ingin menikmati udang galah atau.
yang lezat?
(D) Anda ingin berwisata? 33. Walikota Yogyakarta sudah memberikan
(E) Anda ingin berwisata alam dan berbagai kelonggaran kepada para
berwisata kuliner? pedagang kaki lima untuk berjualan di
trotoar Jalan Malioboro atau tempat-tempat
31. Pemberian nama diri terhadap seseorang lain selama bulan puasa.
rupanya tidak sesederhana yang disangka Kalimat yang sama strukturnya dengan
arang. Pada umumnya, penamaan kalimat di atas adalah
disesualkan dengan pertimbangan- (A) Gubernur DKI menertibkan para
pertimbangan tertentu, seperti menjadi gelandangan dan pengemis yang
penanda identitas, sebagai sarana berumah di bawah jembatan.
penghormatan, penanda urutan, penanda (B) Rektor Ul memberikan beasiswa
jenis kelamin, penanda keakraban, kepada mahasiswa berprestasi dari
kerahasiaan, gengsi, dan sebagainya. berbagai fakultas.
Dalam hal ini, ungkapan Shakespeare (C) Presiden memberi arahan kepada
yang berbunyi “apalah artinya sebuah para menteri untuk tetap bekerja

bimbelocean.com
UTUL UGM 113
PAKET 5
keras tanpa terganggu isu-isu (B) Perusahaan asing menanamkan
pemakzulan. modalnya.
(D) Para mahasiswa menjadi (C) Perusahaan asing mendapatkan
pendamping enak-anak jalanan di keuntungan.
lembah Kali Code. (D) Kelapa sawit laku keras di lnggris, AS,
(E) Menteri Sosial RI menggelontorkan dan Belgia.
dana usaha kepada pemuda-pemuda (E) Perkebunan kelapa sawit menarik
lulusan SMK untuk berwiraswasta modal asing.
dalam berbagai bidang sesuai dengan
minatnya. 36. Tertawa dapat menghambat aliran kortisol,
hormon stres yang dapat menyebabkan
34. Kalimat yang benar adalah peningkatan tekanan darah, penggumpalan
(A) Untuk mengeksplorasi dan darah, dan penurunan sistem kekebalan
mengeksploitasi minyak dan gas tubuh.
bumi yang mana merupakan sumber lnti kalimat di atas adalah
devisa negara diperlukan tenaga ahli (A) Tertawa menurunkan tekanan darah.
di bidang geologi dan perminyakan. (B) Tertawa menyebabkan
(B) Untuk mengeksplorasi dan penggumpalan darah.
mengeksploitasl minyak dan gas (C) Tertawa meningkatkan hormon stres.
bumi, mengingat sebagai sumber (D) Tertawa meningkatkan sistem
devisa negara memerlukan tenaga kekebalan tubuh.
ahli yang terampil di bidang geologi (E) Tertawa menghambat aliran kortisol.
dan perminyakan.
(C) Untuk mengeksplorasi dan 37. Untuk menjaga keberlangsungan sektor
mengeksploitasi minyak dan gas pertanian, pemerintah perlu menciptakan
bumi karena sebagai sumber devisa suatu keadaan yang mendorong kaum
negara diperlukan tenaga ahli yang muda tertarik bekerja di sektor ini.
terampil di bidang geologi dan Pola kalimat di atas sama dengan pola
perminyakan. kalimat
(D) Untuk mengeksplorasi dan (A) Dalam memakai alat-alat teknologi
mengeksploitasi minyak dan gas komunikasi yang canggih, kita harus
bumi sebagai sumber devisa negara memperhatikan akibat yang
memerlukan tenaga ahli yang berpengaruh negatif.
terampil di bidang geologi dan (B) Kenaikan harga jual produk pertanian
perminyakan. dapat mendorong anak-anak muda
(E) Untuk mengeksplorasi dan terjun ke sektor ini.
mengeksploitasi minyak dan gas (C) Kurangnya minat kaum muda bergelut
bumi yang merupakan sumber devisa dengan cangkul dan tanah tampaknya
negara diperlukan tenaga ahli yang sudah menggejala di Pulau Jawa.
terampil di bidang geologi dan (D) Dalam usahanya memenangkan
perminyakan. perkara, kadang-kadang para
pengacara tidak lagi
35. Karena terbukti manguntungkan, sekarang mempertimbangkan kebenaran, tetapi
banyak perusahaan asing berbondong- mementingkan siapa yang
bondong menanamkan modalnya di membayar.
perkebunan kelapa sawit, seperti HCMA (E) Sepanjang dua belas ronde Tyson
dari lnggris, Uni Royal dari AS, dan SIPEF terus-menerus memukuli lawannya,
dari Belgia. tetapi tidak juga bisa meng-KO.
lnti kalimat di atas adalah
(A) Berkebun kelapa sawit 38. Makna asosiasi dapat ditemukan dalam
menguntungkan. kalimat-kalimat berikut, kecuali

bimbelocean.com
114
UTUL UGM
PAKET 5
(A) Mereka sering mengolok-oloknya
karena berasal dari Ragunan. 40. Penulisan kalimat berikut tidak sesuai
(B) Beberapa saksi mata mengatakan dengan kaidah EYD, kecuaii
bahwa pada hari-hari tertentu di (A) Dunia kita sekarang memang jauh
sekitar pohon itu sering terlihat putih- lebih komplek dibandingkan dengan
putih melintas dengan cepat. masa yang lalu karena, tehnologi yang
(C) Mulut gang itu telah ditutup pagar makin canggih telah membantu orang
kawat berduri agar para perusuh tidak lebih efisien dan cerdas, juga lebih
dapat masuk ke perkampungan. terbuka.
(D) Sejak 1976 warung Pak Kumis (B) Kini di era perkembangan teknologi
memang telah digandrungi orang. informasi, kite dengan mudah bisa
(E) Dengan khidmat semua peserta diteropong dan menempong pendapat,
upacara memberi hormat kepada pemikiran, emosi, kedalaman etika,
sang Merah Putih. bahkan tatakrama kila lewat status di
facebook, YM, juga forum diskusi di
39. Pemakaian kalau secara tepat dapat berbagai situs di internet.
dijumpai dalam kalimat (C) Gus dur telah berpulang, kata
(A) Rencana pemberian insentif untuk rohaniawan itu, “Tetapi ajarannya
anggota dewan selama sidang Pansus tentang demokrasi dan toleransi
Century dapat ditinjau ulang kalau beragama masih perlu dikembangkan
dipandang perlu. dalam masyarakat kita yang multi
(B) Banyak SMK di Yogyakarta tldak kultural.
menyangka kalau anggaran uji (D) Meskipun sering kontroversial,
kompetensi tidak turun tahun ini pengaruh Gus Dur terbukti masih
sehingga pencairan dana untuk sangat kuat dan diikuli oleh ribuan
melaksanakannya sebagian besar santrinya.
dibebankan kepada siswa. (E) Presiden SBY mengatakan, dalam
(C) Banyak orang tidak menyangka kalau penyusunan draft teks baru yang
bank yang relatif tidak terlalu besar itu mengakomodasi masukan para kepala
diselamatkan dengan bail-out yang negara pada penemuan Konferensi
jumlahnya tidak rasional. Perubahan lklim PBB harus
(D) Sering tidak disadari kalau uploading dicantumkan keseimbangan kemitraan
foto-foto seronok para remaja di yang adil antara negara maju dan
Facebook dapat mengundang berkembang dalam pengelolaan hutan
prasangka negatif.
(E) Para pembalak tidak mau tahu kalau
aktivitas yang dilakukannya itu bisa
berakibat fatal herhadap ekosistem
beberapa tahun kemudian.

bimbelocean.com
UTUL UGM 115
PAKET 5
BAHASA INGGRIS

I. Study the passage below and choose the best answers to the questions that follow.
India is often the chosen land for those searching for emotional serenity or a cure for
physical ills. At one famed Indian health institute, patients have to tough it out naturally.
Think of spiritual travel and India is synonymous with curative sojoums. From the ashrams
nestled in the valley of the Himalayas to yoga retreats on the shore of Kamataka, the
5 subcontinent offers myriad venues of rest and retreat, for the soul and body and for mental
balance.
Think of medical travel and the subcontinent still bodes well. Home to some of the best
doctors and advanced medical facilities, India now attracts thousands of patients a year for those
unable to meet the hefty costs of surgery in the "developed" world.
10 Think of all of the above? And only one place pops into mind. The Jindal Naturecure
Institute (JNI), which spreads across nearly 50 hectares of lawn, lakes and parks on the outskirts
of India’s garden city, Bangalore, offers two of India's most curative tools: naturopathy and yoga
blending spirituality and medicine, it proposes a unique treatment that combines these two
ancient sciences with dietary and other drug-free therapies.
15 The institute promises complete detox of the mind, body and soul. In its ideals, Jindal
naturopathy is miles away from India's other homeopathic remedies or the western world's
much-loved chiropractic. Much like yoga, it believes that the body is made up of five elements -
earth, water, fire, air and sky. Unhappiness and sickness occur only when these elements are
out of balance.
20 The way to bring these elements back into level ground is by flushing out every negative
toxic in your body and bones - not through medicine, but through yogic kriyas and dietary
changes. Jindal offers treatments for diseases from asthma to depression, migraine to arthritis
and many more - without the use of a single pill or needle. 20 Personal testimonials accompany
each cure.
25 "Health is not something that can be purchased in a bottle,” says Charakasamhita of the
institute. "Diseases are a sign of the cleansing and healing effect of nature. If you suppress the
conditions by drugs or by any other means, you are simply laying a foundation for chronic
diseases. Drugging Is like whipping an already exhausted horse.” The basic belief is that our
bodies are our best healers.
30 While the institute offers some of the best massages and spa treatments in the region,
the Jindal way is no stroll in the park. The institute makes for a pretty setting with benches to
view the sunset over a pristine lake and lots of greenery infused with rare species of migratory
birds. But in reality, it is less spa town and more boot camp.
The whole purpose at JNI is therapeutic healing. While applying for admission, you are
sent forms to capture your medical history and conditions and are warned of the rigidness of the
program. Patients are not allowed out of the institute except for supervised town visits on

bimbelocean.com
116
UTUL UGM
PAKET 5
Sundays. That being said, the staff at JNI are amiable and they are very well informed about the
adjustment period needed for patients not used to confinement.
Adapted from Sherry Samtani, "Nature's boot camp,"
http://www.thejakartapost.com/new!/2009/10/04/natureo39s_boot_camp.html:
Accessed February 12, 2010

41. What is NOT true about Jindal Nature cure (D) putting a heavy load on an animal
Institute? (E) diagnosing an animal's disease
(A) It is located far from the hustle and
bustle of a city. 46. The word "stroll" in line 30 is closest in
(B) It combines traditional and modem meaning to which of the following?
treatments. (A) run
(C) It is far from any other hospitals (B) walk unhurriedly
(D) It promotes mental health as well as (C) laze around
physical health. (D) sit
(E) It does not use chemicals to cure (E) rest
diseases.
47. What does the author mean by "But in
42. The word "sojoums” in line 3 can be best reality, it is less spa town and more boot
replaced by camp" (line 27)?
(A) treatments (A) The institute applies a tight rule for the
(B) trips patients.
(C) brief visits (B) The institute Is run by military staff.
(D) vacations . (C) The institute only treats military
(E) breaks officers.
(D) The institute is a relaxing center.
43. The world “it” in line 16 refers to (E) The institute combines a relaxing
(A) the body center and a camping area.
(B) the JNI
(C) one body element 48. What can be Inferred about India from the
(D) yoga passage?
(E) naturopathy (A) Health care in India Is as expensive as
in the US.
44. What does Charakasamhita mean when (B) India's biggest income Is from its
saying "Diseases are a sign of the health care industry.
cleansing and healing effect of nature” (C) Doctors in India are the best doctors in
(lines 25-27)? the world
(A) Your body has its own way to heal (D) There are new big hospitals In India
from the disease. nowadays.
(B) Nature may heal diseases that you (E) Many people go to India to find peace
suffer from. of mind
(C) Nature contributes a lot to the
diseases that you 49. How do doctors at JNI treat the patients?
(D) Diseases can cure by themselves. (A) Using India's traditional medicine.
(E) When you are sick, your body is (B) Doing therapy outdoors.
cleaning itself. (C) Making the patients stay at the
institute for a certain period of time.
45. The word "whipping” in line 27 means (D) Balancing body elements through food
(A) beating an animal repeatedly and meditation.
(B) making an animal regain Its strength
(C) pushing an animal to keep moving

bimbelocean.com
UTUL UGM 117
PAKET 5
(E) Letting the patients eliminate their
unhappiness to achieve a balance of
body element.

50. What could best replace the word


"confinement" in line 31?
(A) restriction
(B) detention
(C) limitation
(D) quarantine
(E) captivity

II. The text below has incomplete sentences. Choose the one word or phrase from each number that
best completes the sentence.
Eating slowly could help you win the battle of the bulge. A study of more than 3000 Japanese
adults (51) ____ found that those who ate their meals quickly were about twice as likely to be obese as
(52) _____ slow-munching counterpart.
(53) ____ and who ate until they were full were three times more likely to be obese. The
research (54) ____ that people tend to consume more calories when they eat quickly, (55) ____ eating
quickly is linked to obesity regardless of how many calories are eaten. Dr. Elizabeth Denney-Wilson,
(56)____, says research suggests people can learn to eat more slowly and (57)______ their internal
cues for fullness. She adds that while it is not known (58)______, behavior such as eating while
distracted and eating fast food on the go may be to blame.
(59)______ and chatting between bites can help slow the rate of eating and allow your body to
send fullness signals (60) ____ you've overeaten.
Taken from "Fast Eaters Get Far, Reader's Digest February 2009, p. 143.
51. 54.
(A) the British Medical Journal was (A) whose previous evidence supports
published in (B) that supports previous evidence
(B) published in the British Medical Journal (C) supports previous evidence
(C) which published it in the British Medical (D) that previous evidence supports
Journal (E) supporting previous evidence
(D) was published In the British Medical
Journal 55.
(E) the British Medical Journal was (A) but also that
published (B) so that also
(C) also that
52. (D) in that also
(A) theirs (E) also for that
(B) they
(C) them 56.
(D) their (A) an obesity expert is from the University
(E) themselves of New South Wales
(B) Is an obesity from the University of
53. New South Wales
(A) People, when they ate quickly (C) an obesity expert from the University of
(B) People were eating quickly New South Wales
(C) People whose quick eating (D) from the University of New South
(D) People ate quickly Wales, an obesity expert is
(E) People who ate quickly (E) an obesity expert from the University of
New South Wales

bimbelocean.com
118
UTUL UGM
PAKET 5
57. 59.
(A) be recognizing (A) To set aside meal times
(B) recognized (B) Setting aside meal times
(C) recognize (C) By setting aside meal times
(D) recognizes· (D) In setting aside meal times
(E) recognizing (E) Having set aside meal times

58. 60.
(A) what drives us to eat quickly (A) after
(B) when it drives us to eat quickly (B) since
(C) that drives us to eat quickly (C) if only
(D) how it drives us to eat quickly (D) before
(E) how drives us to eat quickly (E) although

bimbelocean.com
UTUL UGM 119
PAKET 6

PAKET TES KEMAMPUAN DASAR UMUM

6
WAKTU : 100 MENIT
JUMLAH SOAL : 60

MATEMATIKA DASAR

1.
√18−√12
+ 1+
5
=⋯ maka x + y = ...
√18+√12 √6 (A) –2
(A) √6 (B) –1
(B) 1 − √6 (C) − 12
11

(C) √2 + √3 5
(D) −
(D) 4 − √6 6
3
(E) 5 − 2√6 (E) − 4

2. Nilai 1 – x yang memenuhi persamaan 6. Jika (b + c, b, c) memenuhi sistem


√83−𝑥 =42 1−2𝑥
adalah ... persamaan
(A) –4 3x – y + 2z = –1
(B) –3 –2x + y + 3z = –3
(C) –2 maka b + c = ...
(D) 3 (A) 0
(E) 4 (B) 1
(C) 2
3. Parabola y = –x2 + 2ax + a – 2 dan garis (D) 3
y= ax + a – 2 berpotongan di (x1, y1) dan (E) 4
(x2, y2). Jika x1 + x2 = 2, maka y1 + y2 = ...
(A) 0 7. Jika m> 0, maka himpunan semua
(B) 1 penyelesaian pertidaksamaan
(C) 2 √𝑚2 − 𝑥 2 ≤ 𝑥 adalah ...
(D) 3 𝑚
(A) {𝑥|0 ≤ 𝑥 ≤ 2 }
(E) 4 𝑚
(B) {𝑥| ≤ 𝑥}
√2
4. Jika α + 2β = 5 dan αβ = –2 maka (C) {𝑥|𝑥 ≤ −
𝑚
atau 𝑥 ≥
𝑚
}
√2 √2
persamaaan kuadrat yang akar-akarnya 𝑚 𝑚
𝛼 2𝛽 (D) {𝑥| − 𝑚 ≤ 𝑥 ≤ − atau ≤ 𝑥 ≤ 𝑚}
𝛼+1
dan 2𝛽+1 adalah ... √2 √2
𝑚
7 (E) {𝑥| ≤ 𝑥 ≤ 𝑚}
(A) 𝑥 2 − 2 𝑥 − 1 = 0 √2
7
(B) 𝑥 2 + 2 𝑥 + 3 = 0 3√2−𝑥
7 8. Semua nilai x yang memenuhi <2
2 𝑥−1
(C) 𝑥 + 2 𝑥 − 3
adalah ...
(D) 2𝑥 2 + 3𝑥 + 4 = 0 7
(A) 𝑥 ≤ 4
(E) 2𝑥 2 + 3𝑥 − 4 = 0
7
(B) 1 < 𝑥 ≤ 4
5. Jika x dan y memenuhi sistem 7
(C) 𝑥 < 1 atau 4 ≤ 𝑥 ≤ 2
persamaan 7
2 1 (D) 𝑥 ≤ − 4 atau 1 < 𝑥 ≤ 2
− = 10
𝑥−1 𝑦+2 (E) 𝑥 < 1
3 1
+ = −9
𝑦+2 𝑥−1

bimbelocean.com
120
UTUL UGM
PAKET 6

9. Daerah penyelesaian sistem 10. Jika jumlah empat suku pertama dan
pertidaksamaan linear y ≥ 0, x + y ≤ 2, 3x jumlah tujuh suku pertama suatu barisan
– 2y ≤ 3 dan –2x + 3y ≤ 3 adalah ... aritmatika berturut-turut 30 dan 48 maka
jumlah kelimabelas suku pertama barisan
itu adalah ...
(A) 330
(B) 336
(C) 345
(D) 360
(E) 366

11. Suku ke 3, 5, dan 8 suatu deret aritmatika


3𝑥+1
berturut-turut adalah , 2x + 2, 4x – 7.
2
Jika Un menyatakan suku ke n barisan
tersebut, maka suku ke 2n adalah ...
(A) 5 + 3n
(B) 2 + 6n
(C) 2Un
(D) 3 + 2Un
(E) 3n + Un

2 1 1 1
12. Jika 𝐴 = ( ), 𝐵 = ( ) dan I
−1 0 0 1
matriks identitas, maka AB–1 + BA–1 = ...
1
(A) 𝐼
3
1
(B) 𝐼
2
(C) I
(D) 2I
(E) 3I

3 1
13. Jika matriks 𝑃=( ) dan 𝑄=
4 2
1 0
( ) serta P–1 invers matriks P, maka
−2 3
determinan untuk matriks QP–1 adalah ...
3
(A) 2
(B) 3
(C) 6
19
(D) 2
(E) 19

14. Hasil penjumlahan semua penyelesaian


𝜋 1
sin2 (𝑥 − 6 ) = 2 untuk 0 ≤ x< 2π adalah ...
4
(A) 𝜋
3
(B) 2
8
(C) 𝜋
3
10
(D) 𝜋
3
14
(E) 𝜋
3

bimbelocean.com
UTUL UGM 121
PAKET 6
15. Misalkan ada 2 jalan dari kota A ke kota 18. Sebuah garis menyinggung grafik f(x) =
B, 4 jalan dari kota A ke kota C, 2 jalan x2 + 3x – 1 di titik (2a – 1, b) dan
1
dari kota B ke kota C. Dari kota B dan C menyinggung grafik g(x) = 2x3 + 4x + 1 di
masing-masing ada 3 jalan ke kota D.
titik (a, c). Nilai a + b = ...
Jika seseorang dari kota A pergi ke kota
(A) 3 atau 41
D melalui kota B atau C atau kota B dan
(B) 4 atau 20
C, maka banyaknya cara yang dapat ia
(C) 4 atau 42
tempuh adalah ...
(D) 6 atau 29
(A) 14
(E) 20 atau 42
(B) 18
(C) 36
19. Persamaan kuadrat𝑥 2 − (3 −
(D) 54 2 2
log 𝑚)𝑥 − log 16𝑚 = 0 mempunyai
(E) 144
akar-akar x1 dan x2. Jika x1x22 + x12x2 = –
6 maka 𝑚 log 8 = ...
16. Nilai rata-rata test matematika di suatu 3
kelas adalah 72. Nilai rata-rata siswa (A) –1 atau
2
3
putra adalah 75 dan nilai rata-rata siswa (B) − 4atau 1
putri adalah 70. Jika banyaknya siswa 1
(C) atau 8
putri 6 lebih banyak dari siswa putra, 16
1
maka banyaknya siswa di kelas tersebut (D) 8
atau 4
adalah ... (E) 4 atau 8
(A) 30
(B) 35 1
20. Jika 1 − cot 𝛼 = − 3 maka nilai sin 2α +
(C) 40
cos 2α = ...
(D) 45 17
(E) 50 (A) 25
(B) 1
𝑥 2 −4 6
17. Jika 𝑎 = lim 2− maka nilai 4 – a (C) 5
𝑥→2 √𝑥+2
31
adalah ... (D) 25
(A) –20 (E)
7
5
(B) –12
(C) –4
(D) 12
(E) 20

BAHASA INDONESIA

21. Belakangan ini mafia hukum dan mafia Terdapat kesalahan pada kalimat-kalimat
pajak sering dibahas, juga disorot mafia pembentuk paragraf di atas, yaitu pada
pengadilan, pertambangan, dan (A) tiadanya tanda koma (,) setelah kata
narkotika. Keadaannya terasa absurd ‘pertambangan' pada kalimat kedua
karena perbincangan temang bahaya (B) mestinya tidak ada partikel ‘-nya'
mafia hanya menimbulkan kegaduhan pada kata ‘Keadaannya’ pada
dan kehebohan ketimbang menggalang kalimat kedua
upaya keras untuk mengatasinya. Semua (C) kata ‘tampak' pada kalimat keempat
seperti tampak tidak berdaya. Tidak mestinya diganti dengan kaia
hanya masyarakat yang mengeluh, tetapi ‘nampak'
juga pejabat pemerintah. Apakah negara
kalah menghadapi mafia?

bimbelocean.com
122
UTUL UGM
PAKET 6

(D) kata penghubung ’tetapi’ pada (C) masalah pembangunan perikanan


kalimat kelima mestinya diganti yang kurang mengacu pada
dengan ‘melainkan‘ kenyataan yang terjadi di lapangan.
(E) kata ‘Apakah‘ pada kalimat keenam (D) masalah peran penyuluh perikanan.
mestinya diganti dengan ‘Mengapa’ (E) masalah keseriusan pemerintah
dalam mengatasi problem yang
22. Dalam rnasyarakat Jawa, wayang dihadapi nelayan dan pembudi daya
merupakan seni pertunjukan yang ikan.
mempunyai peran sebagai sarana
edukatif dan refleksi filosofis. Wayang
yang melakonkan cerita berasal dari 24. Sudah cukup lama upaya pemberantasan
India, yaitu dari epos Ramayana dan mafia hukum diwacanakan oleh para
Mahabarata, itu menjadi Seni lokal yang penegak hukum. lronisnya, meskipun
mencerminkan budaya asli karena wacana tersebut bergaung cukup kuat,
perubahan-perubahan yang dilakukan pihak aparat jarang bisa menangkap
terhadap cerita sebelumnya. Hazeau, mafiosonya. Hal itu karena sosok
seorang pakar budaya Jawa mengatakan maiosonya tidak pernah jelas. Dalam
bahwa “The Javanese theatre and the pengalaman banyak negara, karena
javanese plays are technically original mafia memang merupakan organisasi
Javanese not revealing any Indian kejahatan yang tertata rapi, maiosonya
influence". sulit dilacak. Mafioso justru sering tampil
Paragraf itu dapat diringkas sebagai atau menampilkan diri sebagai orang
berikut. terhormat. Bankan, kata mafioso sendiri
(A) wayang adalah kesenian asli Jawa sering diartikan sebagai “manusia
(B) wayang menjadi seni lokal terhormat".
(C) wayang berasal dari India Kalimat yang paling mencerminkan isi
(D) wayang bercerita tentang epos paragraf di atas adalah
Ramayana (A) Meskipun upaya pemberantasan
(E) wayang selalu berubah mafia hukum telah diwacanakan,
karena mafiosonya tidak pernah
23. Dalam kondisi kerentanan sektor jelas, justru sering tampil sebagai
perikanan, porsi pembangunan perikanan orang terhormat, upaya itu gagal
selayaknya didekatkan pada realitas dilakukan.
keseharian nelayan. Keseriusan (B) Karena sosok mafiosonya tidak
pemerintah memperkuat nelayan harus pernah jelas, upaya pemberantasan
bisa dibuktikan dengan konsistensi mafia hukum sering bersifat ironis
penguatan nelayan, pengawasan dan saja, apalagi mafioso sering
pemulihan kualitas lingkungan, serta menampilkan dirinya sebagai orang
ekosistem pesisir. Peran penyuluh terhormat.
perikanan selayaknya dihidupkan lagi (C) Sebagai organisasi kejahatan yang
sebagai garda terdepan arus informasi tertata rapi, mafia hukum tidak
kepada nelayan dan pembudi daya ikan. pernah bisa diberantas.
Panyuluh juga memiliki peran strategis (D) Pihak aparat jarang bisa menangkap
untuk melaporkan perkembangan mafioso karena sosoknya tidak jelas.
lapangan sehingga kebijakan yang (E) Mafioso sering menampilkan diri
diambil pemerintah bisa menyentuh sebagai orang terhormat.
sasaran.
Pokok persoalan yang dibahas dalam
paragraf di atas adalah 25. Di era globalisasi yang penuh persaingan
(A) masalah kerentanan sektor ini setiap orang dituntut untuk hidup lebih
perikanan. dinamis, lebih aktif, dengan mobilitas
(B) masalah keseriusan pemerintah yang tinggi. Pola hidup masyarakat yang
dalam pembangunan perikanan. sibuk ini pun makin berisiko

bimbelocean.com
UTUL UGM 123
PAKET 6
mendatangkan stres dan kelehahan fisik diperkirakan meningkat 6-8 persen dari
maupun psikis yang menyebabkan total ekspor 2012, atau mencapai 2,4 juta
gampang terserang penyakit. Di sisi lain, ton. Ekspor karet tahun 2012 hingga
kondisi lingkungan semakin buruk Oktober adalah sebanyak 2,01 juta ton.
dengan meninngkatnya pencemaran air Hingga akhir 2012 ekspor karet
dan udara. bertambah 400.000 ton.
Ide pokok paragraf itu adalah Pernyataan berikut ini sesuai dengan isi
(A) globalisasl. yang terkandung dalam paragraf di atas.
(B) pola hidup, (A) Pada tahun 2013 diperkirakan harga
(C) persaingan. karet alam akan naik.
(D) kelelahan. (B) Pada tahun 2013 terjadi peningkatan
(E) pencemaran. permintaan terhadap karet.
(C) Ekspor karet pada tahun 2013
26. Hak asasi manusla dalam pengertian meningkat di atas 8 persen.
umum adalah hak-hak dasar yang dimiliki (D) Permintaan ekspor karet terutama
oleh setiap pribadi manusia sebagai dipengaruhi oleh meningkatnya
anugerah Tuhan yang dibawa sejak lahir. produksi kendaraan.
lni berarti bahwa sebagai anugerah dari (E) Ekspor karet sampai akhir tahun
Tuhan kepada makhluknya, hak asasi 2010 mencapai 2,5 juta ton.
tidak dapat dipisahkan dari eksistensi
pribadi manusia itu sendiri. Hak asasi 28. Daging termasuk jenis makanan yang
tidak dapat dicabut oleh suatu kekuasaan punya segi negatif dan positif. Daging
atau oleh sebab-sebab lalnnya, karena mengandung protein dan zat besi yang
jika hal itu terjadi maka manusia penting untuk tubuh, tetapi Iemaknya
kehiangan martabat yang sebenarnya yang tinggi menjadikan daging sebagai
menjadi inti nilai kemanusiaan. WaIau musuh nomor satu bagi kesehatan.
demikian, bukan berarti bahwa Karena itu, asupan daging dalam menu
perwujudan hak asasi manusia dapat harian perlu dibatasi, maksimal sekali
dilaksanakan secara mutlak karena dapat seminggu.
melanggar hak asasi orang lain. lsi paragraf di atas adalah
Memperjuangkan hak sendiri sampai- (A) Segi negatif dan positif daging
sampai mengabaikan hak orang lain, ini (B) Kandungan protein dan zat besi
merupakan tindakan yang tidak pada daging
manusiawi. Kita wajib menyadari bahwa (C) Kandungan lemak daiam daging
hak-hak asasi kita selalu berbatasan cukup tinggi
dengan hak-hak asasi orang lain. (D) Asupan daging cukup dikonsumsi
Di dalam paragraf tersebut terdapat seminggu sekali
kalimat yang dapat dihapuskan tanpa (E) Pembatasan menu hanan daging
mengurangi informasinya, yaitu kalimat
(A) ke-1
(B) ke-2
(C) ke-3 29. Jalanan rusak, berlubang di sana-sini,
(D) ke-4 dan tambahan aspal yang membuat lalu
(E) ke-5 lintas tak nyaman sudah menjadi
pemandangan biasa di negeri ini. Kondisi
27. Harga karet alam di pasar dunia tahun ini jelas tak menguntungkan bagi
2013 tetap bagus. Seiring meningkatnya masyarakat dan dunia usaha. Harga
produksi kendaraan, permintaan terhadap barang menjadi tambah mahal karena
karet meningkat. Menurut Chairman arus lalu lintas komoditas yang seret. Tak
Gabungan Perusahaan Karet Indonesia, Cuma jalan, kualitas infrastruktur lain juga
Asril Sutan Amlr, tingginya permintaan sama seperti listrik masih byar-pet.
mendorong peningkatan laju ekspor Inti paragraf itu adalah
karet. Eksporkaret tahun 2013 (A) sarana tak memadai

bimbelocean.com
124
UTUL UGM
PAKET 6

(B) harga barang mahal seriatopora, sedangkan yang


(C) arus lalu lintas seret pemutihannya moderat dari jenis
(D) listrik byar-pet pocillopora.
(E) Jalanan rusak (C) Dari skenario itu bisa dikembangkan
identifikasi sumber daya yang
30. Akibat erupsi merapi, seluruh anggota tersedia.
keluarganya tidur berhari-hari di (D) Karang yang lambat memulihkan diri
pengungsian. dengan mudah digantikan oleh karang
Bentuk ulang yang bermakna sama yang pemulihannya lebih cepat.
dengan bentuk ulang yang terdapat pada (E) Selalu ada saja pihak-pihak yang
kalimat di atas adalah mengambil sikap ekstrem.
(A) Meskipun telah dibacanya
berhalaman-halaman, dia belum 33. Makna kolusi dapat ditemukan dalam
menemukan juga apa yang kalimat berikut.
dicarinya. (A) Perusahaannya berkembang pesat
(B) Kakek terkekeh-kekeh mendengar karena ia pandai menjaga
ceritaku. hubungannya dengan banyak rekan
(C) Ketika kami beristirahat di lembah itu bisnisnya.
terdengar suara burung bersahut- (B) Budaya “pertemanan” dituding
sahutan. banyak orang telah menyuburkan
(D) Setelah bersalam-salaman kami aktivitas kerja sama tak resmi dalam
makan bersama sambil melanjutkan melancarkan urusan di berbagai
obrolan. bidang birokrasi.
(E) Sebelum mengajar, mereka terlebih (C) Banyak asumsi menyatakan bahwa
dahulu mengikuti diklat selama dua kekayaan yang dipermasalahkan itu
bulan berturut-turut. didapatnya dengan cara yang tidak
dapat dipertanggungjawabkan dari
31. Kalimat berikut adalah kalimat baku banyak pos kegiatan pemerintah.
(A) Cukup banyak partai politik yang (D) Juru kampanye partai harus pandai
masih mengultuskan Bung Besar berdiplomasi untuk menarik massa
sebagai maskot perjuangan mereka. sebanyak-banyaknya agar dapat
(B) Selain mengorganisasikan materi, memenangkan pemilu.
guru harus dapat memposisikan (E) Pendapat-pendapat bernada minor
dirinya sebagai aktor yang baik agar bermunculan manakala seorang
kelas menjadi hidup. presiden mengangkat putrinya dalam
(C) Dibandingkan dengan GM, T jajaran menteri kabinetnya.
menjual lebih banyak mobil di luar
Amerika Serikat. 34. Penggunaan kata yang bermakna leksikal
(D) Arisan tidak hanya dilakukan oleh terdapat dalam kalimat
ibu-ibu di kampung, melainkan juga (A) Kenangan indah semasa dl SMA
para pemulung, pengasong, dan terukir di hatinya.
kondektur bus kota. (B) Masih terukir dengan sangat jelas di
(E) Ada siyalemen menyatakan ingatannya peristiwa letusan Gunung
peredaran narkoba cenderung Merapi itu.
meningkat seiring dengan maraknya (C) Suasana hiruk pikuk di ternpat
bisnis hiburan malam. pengungsian terukir dengan baik
pada lukisannya.
32. Induk kalimat yang berintikan pronomina (D) Masa remaja yang penuh ceria
dan nomina terdapat pada terukir begitu mendalam di halinya.
(A) Sebagai tanda kasih dan terima kasih, (E) Jejak-jejak ombak serta alur air
merekalah yang melayani para ibu. teriihat terukir di atas karang.
(B) Karang yang mengalami pemutihan
total umumnya dari spesies

bimbelocean.com
UTUL UGM 125
PAKET 6
35. Masyarakat masa depan yang berada Kegunungapian Yogyakarta
dalam konteks risk society perlu memperkirakan bahwa bahaya lahar
melakukan kalkulasi risiko. Kalkulasi ini dingin dari material vulkanik hasil
akan membantu untuk mengarahkan erupsi Gunung Merapi akan
masyarakat ke format masyarakat masa berlangsung dalam waktu yang
depan yang diinginkan melalui prediksi lama.
yang disusun dan keputusan yang (C) Sosialisasi peningkatan penggunaan
diambil. produk dalam negeri (P3DN) yang
Makna risk society dalam paragraf di atas dilakukan Menteri Perindustrian MS
adalah Hidayat saat inidinilai hanya masih
(A) masyarakat yang berisiko sebatas inventarisasi permasalahan
(B) risiko bermasyarakat dunia usaha.
(C) risiko kemasyarakatan (D) Pada Selasa dilaporkan bahwa
(D) masyarakat berisiko terjadinya letusan sekunder di kali
(E) risiko yang harus ditanggung Gendol yang menyebabkan
masyarakat munculnya asal dengan ketinggian
sekitar 300 meter.
36. Penulisan huruf kapital yang mengikuti (E) Pada bulan Desember juga akan
EYD terdapat dalam kalimat berikut ada hari besar keagamaan dan
(A) Senin malam banjir lahar kembali Tahun Baru, sehingga penggunaan
terjadi di aliran Kali Putih dan premium akan naik.
sempat menerobos areal dekat
pasar Jumoyo. 38. Kalimat berikut adalah kalimat Baku
(B) Setelah sejak lama diserahkan ke (A) Partai politik yang tidak menaati
DPR RI pemerintah melalui peraturan akan mendapat sanksi
Mendagri akan memberikan tegas.
penjelasan mengenai rancangan (B) UGM memelopori aktivitas itu
Undang-Undang Keistimewaan DIY dengan mengadakan KKN pengamat
di hadapan Komisi ll. pemilu.
(C) Enam partai kecil di Salatiga akan (C) Dealer menyediakan beberapa
menggetarkan Pemilihan Umum kemudahan dalam pembayaran
Kepala Daerah (Pemilukada) kota sehingga konsumen dapat menyicil
Salatiga yang digelar pada 8 Mei angsuran sesuai waktu yang
2013 mendatang. diinginkan.
(D) Sebagaimana diungkapkan oleh (D) Meski tidak diakui, beberapa tahun
Jassin surat dari PSSI yang berisi lalu Golkar pernah menginstruksikan
penegasan bahwa Piala AFF kantor-kantor pemerintah untuk
bukanlah gratifikasi itu telah diterima mencat pagar mereka dengan warna
KPK. kuning.
(E) Dalam usaha menegakkan aturan (E) Dengan adanya status otonomi
dan menghilangkan kesan tidak adil pemerintah pusat tidak lagi
Komisi Penyiaran Indonesia Daerah sepenuhnya mensubsidi daerah.
Jateng akan menertibkan Radio
Siaran Pemerintah Daerah. 39. Mengharap kehadiran Bapak/Ibu pada
acara syukuran pernikahan anak kami
37. Penulisan huruf kapital yang benar yang akan kami selelnggarakan pada
terdapat dalam kalimat Rabu, 2 Februari 2011.
(A) Aspek infrastruktur pula yang Kalimat di atas tidak memenuhi syarat
membuat penerapan pembatasan sebagai sruktur kalimat yang baku karena
premium tidak bisa serentak (A) penggunaan kata kami yang
dilakukan di seluruh Tanah Air. berlebihan
(B) Balai Penyelidikan dan (B) tidak berobjek
Pengembangan Teknologi (C) tidak bersubjek

bimbelocean.com
126
UTUL UGM
PAKET 6

(D) tidak bersubjek dan tidak berobjek (C) Dalam pendaftaran sekolah pun
(E) tidak menggunakan bentuk aktif calon siswa akan diranking dan
pada kata selenggarakan dikelompokkan berdasarkan nilai
ujian nasional.
40. Berikut ini merupakan kalimat majemuk (D) Sekolah yang seharusnya menjadi
bertingkat tempat yang menyenangkan dan
(A) Dari sudut pandang ini, kegiatan membudayakan anak berubah
pembelajaran tidak lagi bersifat menjadi arena persaingan yang tidak
klasikal, tetapi lebih dikembangkan sehat.
ke arah kerja kelompok dan (E) Misi pencerahan pendidikan harus
perseorangan. mencakup perbaikan aspek-aspek
(B) Anak-anak perlu mempersiapkan diri kemlskinan tersebut secara
untuk memasuki era demokratisasi, seimbang dan tidak dapat direduksi
suatu era yang ditandai oleh pada orientasi kemajuan ekonomi
keragaman perilaku, dengan cara semata.
terlibat dan mengalami secara
langsung proses pendemokrasian.

BAHASA INGGRIS

I. Study the passage below and choose the best answers to the questions that follow

1 US pop and soul music legend Stevie Wonder told diplomats from nearly 200 nations on Monday to
slop squabbling over copyright and agree on a pact bringing “hope and light” to blind people around the
globe.
And the singer-musician, himself sightless since just after birth, warned negotiators at the United
5 Nations intellectual property and copyright agency WIPO that he would write a sad song about them if
they didn’t act on his appeal.
“We must declare a state of emergency and end the information deprivation that continues to keep
the visually impaired in the dark”, said Wonder, whose music has won dozens of top awards in his 50-
year career.
10 He told delegates on the opening day of WIPO’s annual assembly that they should agree on an
action plan that would empower the blind and near-blind by side-stepping copy-right rules and giving
them access to books and learning.
And the star wrapped us his 10-minute appeal by singing to his accompaniment famous lines from
many of his best-loved songs including I just Called to Say I Love You and Keep Our Love Alive.
15 His call was endorsed by the World Blind Union, which said that in developing countries less than
one percent of published works were available in formats like Braille or audio. Even in rich countries, the
total was less than 5 percent.
WIPO member states have for years been considering a deal that would overcome cross-border
copy-right rules and finance translation of books into Braile but has run up against strong differences
20 among member states.
Some argue for totally free rights of translation into format accessible to the blind, while others insist
that no loopholes be left for piracy of texts, music, and technology.
Wonder, a UN Messanger of Peace who has sold more than 100 million records since the 1960s,
said the delegates should “put ideological differences aside and come up with a practical solution”.
25 While it was critical in loosening laws to avoid harming authors of great works that “nourish our
hearts, minds and souls,” he said, WIPO members must find a deal that allowed easy import and export
of copyright material for the blind.
We have the greatest minds in the world right here in this room. Please work it out. Or I’ll have to
29 write a song about what you didn’t do’, he declared to laughter and applause from many delegations.

bimbelocean.com
UTUL UGM 127
PAKET 6
Taken from Robert Evans, “Pop icon urges the UN to ring light to the blind.”
The Jakarta Post, Sepetember 22, 2010, p.28.

41. This passage mainly discusses 46. The word “accompaniment” in line 13 is
(A) World Blind Union closest in meaning to....
(B) WIPO’s annual assembly (A) To escort someone to go somewhere
(C) Stevie Wonder’s appeal for the blind (B) To make something more adequate
(D) Deprivation of information of the visually (C) Two types of musical instruments
impaired played at the same time
(E) Stevie Wonder’s song for WIPO members (D) A pianist who plays one part of a piece
of music while someone else sings
42. It can be inferred from the passage that the (E) Music that is played as a background to
word “light” in the phrase “hope and light in line a song
2 refers to....
(A) provision on information 47. Where in the passage does the writer
(B) illumination indicate the reason for the delay in
(C) vision translating books into Braille?
(D) intellectual property (A) Lines 3-5
(E) Braille audio (B) Lines 11-12
(C) Lines 18-20
43. From the term “a state of emergency” in line 7, it (D) Lines 24-26
can be inferred that it refers to a state where.... (E) Lines 28-29
(A) Copy-right rules are side stepped
(B) Cross-border copy-right rules cannot be 48. According to the passage, the World Blind
overcome Union....
(C) WIPO members are unable to come up with (A) Publishes works in Braille or Audio
a solution (B) Advocates access to books and
(D) Diplomats squabble over copy-right learning for the visually impaired
(E) The blnds are kept in the dark (C) Argues that piracy of texts is intolerable
(D) Came up with a practical solution
44. The passage emotions that Stevie Wonder (E) Totally agrees to free rights of
pleaded that the delegates of WIPO’s annual translation
assembly should....
(A) evade copy-right rules 49. Whose minds does Stevie Wonder refer to as
(B) empower the blind the greatest minds (lines 28)?
(C) agree on a pact (A) Authors
(D) declare a state of emergency (B) WIPO members
(E) finance translation of books in Braille (C) UN Messenger of Peace
(D) WIPO delegates
45. It is stated in the passage that as a wrap-up, (E) World Blind Union
Stevie Wonder sang...
(A) Many of his best loved songs 50. According to the passage, the delegates
(B) Two songs laughed and applauded to Stevie Wonders...
(C) For 10 minutes (A) Reference to greatest minds in the room
(D) Famous lines from his best-loved songs (B) Cautionary remark
(E) Lines from I just Called to Say I love You (C) Insistence on finding a deal
and Keep Our Love Alive (D) Statement “nourish our hearts, inds,
and souls”
(E) Request to work it out

bimbelocean.com
128
UTUL UGM
PAKET 6

II. The text below has incomplete sentences. Choose the one word or phrase from each number that
best completes the sentences.

Historians have traced the first toothbrush to the 11 th-century China. They were made of
bristles from Siberian hogs (51)___________ in cold climates to increase their bristles. The bristles
were attached (52)_______ handles made of bamboo or wood.
During the Dark Ages in Europe, dental hygiene consisted of using the tip of a knife as a
toothpick. (53)_________ hog-bristle toothbrushes came to Europe in the 15 th century, many people
found the bristles too stiff, and preferred toothpicks. Other people used horsehair or badger hair
brushes, which were too soft to do much good.
(54)__________, in England by the late 19 th century, mass production of hog-bristle
toothbrushes was underway. (55)________ 1938, in the US, that toothbrushes made of synthetic
nylon were introduced. Again, people found them too stiff.
Finally in the 1950s, softer nylon brushes arrived. As for toothpaste, (56)________ in an
Egyptian tomb dating back 4000 years. Since it contained strong vinegar along with poedered pumice
stone, it actually wore down tooth enamel, (57)_________ decay.
The Romans had similar recipes, with similar results, but one of their ingredients was urine.
The ammonia found in urine may offer protection (58)__________ decay, and ammonia compounds
are still used in some tooth-pastes today.
Up until the 19th century, toothpaste were (59)_________. But they continued to evolve, and
after World War II modern toothpaste (60)_______produced. The introduction of fluoride in 1956
caused a dramatic drop in cavities

51. (A) Which raised 56. (A) It was found in an earliest recorded recipe
(B) Which were raising (B) It was recorded earliest in a recipe found
(C) Which were raised (C) Its earliest recorded recipe was found
(D) Who were raised (D) Found in an earliest recipe, was recorded
(E) Whom they were raising (E) Was found in its earliest recorded recipe

52. (A) by 57. (A) Caused


(B) to (B) Causes
(C) into (C) And causes
(D) with (D) And causing
(E) from (E) Causing

53. (A) when 58. (A) Of


(B) before (B) Over
(C) after (C) From
(D) as (D) Against
(E) since (E) To

54. (A) besides 59. (A) Harming as much as doing good


(B) moreover (B) Doing as much harm as good
(C) for (C) Good as much as harm
(D) since (D) Doing much harm as good
(E) nonetheless (E) Doing so good as much harm

55. (A) It wasn’t until 60. (A) Were


(B) Since (B) Are
(C) After (C) Is
(D) During (D) Was
(E) Until it wasn’t in (E) Had been

bimbelocean.com
UTUL UGM 129
PAKET 7

PAKET TES KEMAMPUAN DASAR UMUM

7
WAKTU : 100 MENIT
JUMLAH SOAL : 60

MATEMATIKA DASAR

1. Jika 4𝑦+3𝑥 = 64 dan 𝑥


log(𝑥 + 12) − (D) 𝑘 < 3
2
𝑥
3 log 4 = −1, maka 𝑥 + 2𝑦 = ⋯ 3
(E) 𝑘 < 2
(A) 86
(B) 34
(C) –5 5. Diberikan sistem persamaan
(D) –14 (𝑎 − 1)𝑥 + (𝑏 − 1 )𝑦 = 0
(E) –34 (𝑏 + 1 )𝑥 + (𝑎 + 1 )𝑦 = 0
dengan ab. Agar penyelesaian sistem
2. Suatu grafik fungsi kuadrat memotong persamaan di atas tidak hanya (x, y) = (0,
sumbu x di A (1, 0) dan B (2, 0). Jika 0) saja, maka nilai a + b = ...
grafik fungsi kuadrat tersebut melalui titik (A) –1 (D) 0
(0, 4) dan puncaknya di titik (p, q), maka (B) –2 (E) 2
p + q = ... (C) 1
(A) 1
3 6. Nilai x yang memenuhi pertidaksamaan
(B) 2 𝑥 2 −2𝑥−3
(C) 2 < 𝑥 + 5 adalah ...
𝑥−2
5
(D) (A) 𝑥 < 2
2 7
(E) 3 (B) 𝑥 > 5
7
(C) <𝑥<2
5
3. Diberikan dua parabola dengan 13
(D) − <𝑥<2
persamaan 𝑓(𝑥 ) = 𝑎𝑥 2 + 𝑏𝑥 + 𝑐 dan 5
2 7
𝑓(𝑥 ) = 𝑝𝑥 + 𝑞𝑥 + 𝑟. Jika f dan g tidak (E) 𝑥 < atau 𝑥 > 2
5
𝑏 𝑞
berpotongan dan = 𝑝, maka jarak
𝑎
terdekat dua parabola tersebut adalah 7. Pada sistem pertidaksamaan 𝑥 − 𝑦 ≤ 0,
selisih dari ... 𝑥 + 𝑦 ≥ 4 dan −5𝑦 + 𝑥 ≥ −20 berlaku
(A) r dan c 2𝑥 + 3𝑦 ≥ 𝑘. Nilai k terbesar adalah ...
𝑏 𝑞 (A) 25 (D) 5
(B) 𝑓 (− 2𝑎) dan 𝑔 (− 2𝑝)
(B) 12 (E) 0
(C) 𝑓 (−𝑏) dan 𝑔(−𝑞 ) (C) 10
𝑏 𝑞
(D) 𝑓 ( ) dan 𝑔 ( )
𝑎 𝑝
𝑏 𝑞 8. Jika tiga bilangan x, y dan z membentuk
(E) 𝑓 (2𝑎) dan 𝑔 (2𝑝) 1 1
barisan geometri, maka − 𝑦−𝑧 = ⋯
𝑥−𝑦
1
4. Jika garis 2𝑥 − 3𝑦 + 5𝑘 − 1 = 0 (A) 𝑥
1
memotong parabola 𝑦 = 𝑥 2 − 2𝑥 + 𝑘 + 1 (B) − 𝑦
di dua titik maka nilai k yang memenuhi 1
(C)
adalah ... 𝑧
3 1
(A) 𝑘 < − 2 (D) 𝑥+𝑧
2 1
(B) 𝑘 < − 3 (E) 𝑥−𝑧
2
(C) 𝑘 > − 3

bimbelocean.com
130
UTUL UGM
PAKET 7
9. Tiga bilangan membentuk barisan muatan kelima truk, maka muatan truk A
aritmatika. Jika jumlah suku ke-1 dan + muatan truk E = ... ton.
suku ke-3 adalah 30 dan jumlah dari (A) 8,17
logaritma suku ke-1, ke-2 dan ke-3 (B) 9
adalah 3 + log 3, maka suku ke-1 barisan (C) 10
tersebut adalah ... (D) 10,25
(A) –5 atau 5 (E) 10,5
(B) 5 atau –10
(C) 5 atau 25 14. Dalam suatu barisan aritmatika, nilai rata-
(D) 10 atau 20 rata dari 4 suku pertama adalah 8 dan
(E) 25 atau 15 nilai rata-rata 9 suku pertama adalah 3.
Jumlah n suku pertama barisan tersebut
10. Nilai semua x sehingga matriks adalah ...
2 (A) –10n + n2
[√𝑥 − 1 1] mempunyai invers adalah ...
𝑥 2 (B) 11n + n2
4 4
(A) 𝑥 ≠ − 3 dan 𝑥 ≠ 3 (C) 12n – n2
4 4
(D) –10n – n2
(B) 𝑥 ≠ −√3 dan 𝑥 ≠ √3 (E) 8n – n2
4 4
(C) √3 < 𝑥 ≤ −1 atau 1 ≤ 𝑥 < √3 15. Diberikan fungsi-fungsi f dan g dengan
4 4
persamaan 𝑓(𝑥 ) = 𝑥 2 , 𝑥 ≤ 0 dan 𝑔(𝑥 ) =
(D) −√3 < 𝑥 ≤ −1 atau 1 < 𝑥 < √3 −√𝑥, 𝑥 ≥ 0. Jika 𝑓 −1 adalah invers dari f,
4 4 maka (𝑓 −1 °𝑔)(𝑥 ) = ⋯
(E) 𝑥 < −√3 atau −√3 < 𝑥 ≤ −1 atau
(A) 0
4 4 (B) –x
1 ≤ 𝑥 < √3 atau 𝑥 > √3
(C) x
11. Jika sudut 
memenuhi cos 2 𝛼 + (D) − 4√𝑥
1
2 sin(𝜋 − 𝛼) = sin2 (𝜋 + 𝛼 ) + 1 2, maka sin (E) 4√𝑥
 = ...
1 16. Jika 𝑓 (𝑥 2 + 3𝑥 + 1) = 2
log(2𝑥 3 − 𝑥 2 +
(A) 2 7), 𝑥 ≥ 0 maka f(5) = ...
1
(B) 2
√2 (A) 1
1 (B) 2
(C) 3
√3
(C) 3
(D) √3 (D) 4
(E) 1 (E) 5

12. Peluang Ali, Budi dan Dian lulus “UAN” 17. Untuk x ≥ 1, nilai maksimum fungsi f(x) =
masing-masing adalah 0,7; 0,8 dan 0,9.
−x 3 + 6x 2 − 9x + 7 adalah ...
Peluang lulus hanya satu orang diantara
(A) 3 (D) 11
tiga orang tersebut adalah ...
(B) 6 (E) 23
(A) 0,082
(C) 7
(B) 0,092
(C) 0,504 3
18. Kurva 𝑦 = 3𝑥 − 𝑥 2 memotong sumbu x di
(D) 0,82
(E) 0,92 titik P. Persamaan garis singung kurva di
titik P adalah ...
13. Diketahui 5 buah truk. Truk A dan B (A) x – 9y – 9 = 0
masing-masing memuat 4 ton. Truk C (B) x – 9y + 9 = 0
dan D masing-masing memuat 6 ton. Jika (C) 9x – y – 9 = 0
truk E memuat 1 ton lebih dari rata-rata (D) 9x – y + 9 = 0
(E) 9x + y – 9 = 0

bimbelocean.com
UTUL UGM 131
PAKET 7
19. Jika  dan  adalah akar-akar persamaan 20. Bentuk sederhana dari
1 1 1 1 1 2
kuadrat 𝑥 2 − (𝑎 + 5)𝑥 + 5𝑎 = 0, maka (𝑥 3 − 𝑥 6 ) (𝑥 2 + 𝑥) (𝑥 2 + 𝑥 3 + 𝑥 3 )
nilai minimum dari α2 + β2 adalah ...
4 1 2
(A) 5 (𝑥 3 − 𝑥) (𝑥 + 𝑥 3 + 𝑥 3 )
(B) 10
dengan x 0 adalah ...
(C) 18 1
(D) 20 (A) 𝑥 −3
1
(E) 25 (B) 𝑥 3
2
(C) 𝑥 3
2
(D) 𝑥 −3
1
(E) 𝑥 2

BAHASA INDONESIA

Bacaan berikut digunakan untuk menjawab 21. lde pokok bacaan di atas adalah
soal nomor 1 sampai dengan nomor 3. (A) kekayaan budaya Lombok.
(B) membaca naskah Iontar sebagai
Etnis Sasak, Lombok, Nusa Tenggara tradisi di Lombok.
Barat, memiliki kekayaan seni budaya yang (C) pembaca lontar yang populer di
beragam. Salah satu di antaranya adalah Lombok.
naskah lontar atau takepan yang masih (D) isi naskah lontar Lombok.
banyak disimpan masyarakat. Naskah Iontar (E) perlunya pelestarian naskah lontar
antara lain berisi ajaran akhlak, cerita, Lombok.
falsafah, upacara adat, keagamaan, dan
solidaritas sosial. Naskah ini beraksara Jawa 22. Inti bacaan di atas adalah
Kuno, Jawa, Bali, dan Sasak atau Jejawan. (A) Membaca naskah lontar menjadi
Keberadaan naskah ini kemudian melahirkan tradisi di Lombok.
tradisi membaca naskah Iontar. (B) Tradisi membaca naskah Iontar
Tradisi membaca naskah Iontar sudah mulai ditinggalkan.
biasanya diselenggarakan pada acara tenentu, (C) Naskah lontar memiliki banyak
seperti khitanan, ngurisang atau potong ajaran.
rambut pada anak balita, dan pernikahan. (D) Membaca naskah Iontar dilakukan
Naskah yang dibacakan pada beragam acara dalam acara tertentu.
tersebut disesuaikan dengan permintaan si (E) Pembaca naskah lontar sudah Ianjut
empunya hajat. Dalam acara ngurisang, usia.
naskah yang dibacakan bisa takepan
lndarjaya, sedangkan dalam acara pernikahan 23. Pernyataan yang berhubungan dengan isi
dapat dibacakan naskah Dewi Rengganis. bacaan di atas adalah
Takepan lndarjaya berisi kisah putri bisu yang (A) Naskah lontar tergerus oleh
kemudian bisa berbicara setelah disapa kemajuan teknologi.
seorang pemuda. Adapun naskah Dewi (B) Ada banyak naskah lontar yang
Rengganis berisi pedoman muda-mudi dalam hilang.
memilih jodoh. (C) Generasi muda sudah tidak tertarik
Salah satu pembaca takepan yang membaca naskah lontar.
relatif populer adalah Nari alias Amak Nurmini (D) Nari adalah salah seorang pembaca
atau Mini. Usianya sudah lanjut, 70 tahun. naskah lontar.
Namun, ia masih sering kali mendapat (E) Ada lomba membaca naskah Iontar.
permintaan warga Lombok untuk membaca
takepan demi meramaikan acara yang mereka
selenggarakan.

bimbelocean.com
132
UTUL UGM
PAKET 7
24. Badan Kesehatan Dunia (WHO) target yang diincar (laut, danau, atau
menyatakan bahwa polusi udara menjadi tempat Iain yang diprediksi terdapat
ancaman kesehatan terbesar di dunia banyak ikan). Setelah dilepaskan,
karena telah menyebabkan tujuh juta perangkat ini dapat Iangsung
kematian pada tahun 2012. Polusi udara mengirimkan informasi melalui
telah menjadi satu dari delapan penyebab smartphone mengenai keberadaan ikan.
kematian di seluruh dunia pada tahun
2012. WHO menegaskan bahwa upaya lde pokok paragraf di atas adalah
pengurangan polusi di dalam dan di Iuar (A) Ditemukannya alat pencari ikan
rumah dapat menyelamatkan jutaan otomatis.
nyawa di masa depan. Kematian akibat (B) Alat pencari ikan dengan teknologi
polusi udara biasanya terjadi dalam sonar.
bentuk penyakit jantung, stroke, penyakit (C) Pelacakan keberadaan ikan.
paru-paru kronis, infeksi saluran udara, (D) Prosedur pemakaian alat pencari
dan kanker paru-paru. ikan.
(E) Alat pencari ikan otomatis siap pakai.
lnti paragraf di atas adalah
(A) Karena polusi udara telah 26. Eisiensi dapat dicapai oleh PT Pelni
menyebabkan tujuh juta kematian dengan menekan biaya operasional
pada 2012, upaya pengurangan secara signifikan. Melalui audit teknologi
polusi udara menjadi sangat penting. dengan memasang alat pengukur kinerja
(B) Polusi udara merupakan salah satu mesin penggerak dan generator di kapal
penyebab kematian di seluruh dunia akan bisa diketahui kebocoran, keausan,
pada tahun 2012. dan kerusakan Iainnya. Efisiensi juga bisa
(C) Menurut WHO, upaya pengurangan dilakukan dengan pengelolaan kapal
polusi di dalam dan di luar rumah yang ramah lingkungan, antara Iain
dapat menyelamatkan jutaan nyawa dengan menggunakan bahan bakar
di masa depan. nabati dan mendaur ulang air Iimbah cair
(D) Menurut WHO, pada tahun 2012 di kapal. Audit energi oleh BPPT pada
kematian yang disebabkan oleh gedung dan pabrik merekomendasikan
polusi udara naik dua kali lipat dari perbaikan sistem operasi yang dapat
perhitungan tahun sebelumnya. menghemat hingga 30% dari total
(E) Menurut WHO, polusi udara menjadi konsumsi energi.
ancaman kesehatan terbesar di
dunia. Pernyataan berikut ini yang tidak
berhubungan dengan isi paragraf di atas
25. Orang-orang yang mempunyai hobi adalah
memancing ikan sudah tidak perlu Iagi (A) Pengelolaan kapal yang ramah
menunggu lama agar sang ikan lingkungan dapat menekan biaya
memakan umpan. Saat ini sudah operasional.
ditemukan alar pencari ikan dengan (B) Alat pengukur kinerja mesin
teknoiogi sonar yang dinamakan Fish penggerak dan generator di kapal
Hunter Sonar. Alat ini dapat melacak dapat mendeteksi kebocoran,
kedalaman, temperatur, dan Iokasi keausan, dan kerusakan lain.
tempat terdapat banyak ikan. Untuk (C) Perbaikan sistem operasi dapat
mendapatkannya, pertama yang harus menghemat hingga 30% total
dilakukan adalah mengunduh aplikasi konsumsi energi.
untuk IOS. Setelah itu, pasang aplikasi (D) Audit energi dilakukan oleh BBPT
yang relah diunduh tersebut. Selanjutnya, pada gedung dan pabrik.
hubungkan sonar dengan koneksi (E) Bahan bakar nabati dan daur ulang
bluetooth ke smartphone. Alat ini siap air Iimbah cair di kapal dilakukan
dipakai dengan care melepaskannya ke oleh PT Pelni.

bimbelocean.com
UTUL UGM 133
PAKET 7
sendiri, yaitu implementasi konkrit
27. Banjir bukan ancaman Iinier. Sebab dalam penyelesaian kasus-kasus
musababnya berkembang mengikuti pelanggaran berat HAM di masa lalu.
perilaku manusia. Menurut catatan Badan (D) Direktur Utama BNI Syariah yakin
Nasional Penanggulangan Bencana, bahwa kondisi perbankan syariah
pada era sebelum 1970-an penyebab masih kondusif meski
banjir di DKI didominasi oleh fakfor alam. pertumbuhannya tak setinggi 2013.
Pada era sesudahnya, faktor manusialah (E) Dalam mengajukan budaya yang
yang paling dominan. Perbuatan manusia akan dinominasikan kita perlu
itu dari merusak daerah resapan air dan menerapkan strateji seperti
daerah aliran sungai sampai membuat membandingkan dengan apa yang
permukiman yang mengakibatkan diajukan negara lain.
penyempitan batang sungai.
30. Pemakaian tanda koma yang tidak tepat
Simpulan paragraf di atas adalah terdapat dalam kalimat
(A) Karena itu, penyebab banjir berbeda- (A) Usaha diet dan olahraga yang
beda. dilakukannya dapat menurunkan
(B) Dengan demikian, manusialah yang berat badan, tetapi setelah itu angka
menjadi penyebab terjadinya banjir. timbangan seolah sulit bergerak
(C) Oleh karena itu, banjir dapat turun Iagi.
ditanggulangi. (B) Sebagai orang yang sibuk, pola
(D) Jadi, ada perubahan cara makan yang tak seimbang
menangani penyebab banjir. sepertinya sudah menjadi kebiasaan.
(E) Sehingga, penanggulangannya (C) Makan dapat menimbulkan perasaan
harus sesuai penyebabnya. nyaman, sehingga banyak orang
memilih makan saat mereka sedang
28. Pemakaian huruf kapital yang tepat stress atau tertekan.
terdapat dalam kalimat (D) Profesor David Herman, peneliti di
(A) Karena sedang ada perbaikan, Universitas Gadjah Mada,
Saudara dimohon memakluminya. melakukan riset itu selama puluhan
(B) Moh. Hatta adalah Wakil Presiden RI tahun.
pertama. (E) Karya seni pada dasarnya,
(C) Daerah ini merupakan penghasil merupakan hasil penafsiran
gula merah atau gula Jawa. kehidupan yang dilakukan oleh para
(D) Kementerian Pendidikan Dan seniman melalui proses kreatif.
Kebudayaan membuka posko
pengaduan uiian nasional. 31. Pemakaian kata ulang yang tepat
(E) Untuk menjaga stabilitas di kawasan terdapat dalam kalimat
Asia tenggara, dibentuklah (A) Dengan beberapa kali membaca-
organisasi yang bernama ASEAN. baca tulisan di papan iklan, secara
langsung anak belajar membaca dan
29. Penulisan unsur serapan yang benar memahami isi bacaan.
terdapat pada kalimat (B) Ada banyak surat-surat kabar terbit
(A) Perbankan syariah hingga kini masih sehubungan dengan kebebasan
mengalami masalah lekuiditas. pers.
(B) Berdasarkan data Autoritas Jasa (C) Mereka berdua saling panggil-
Keuangan, hingga akhir Desember memanggil di bandara.
2013, angka Finance to Deposit (D) Sayur-mayur dijual di pasar
Ration (FDR) Bank Syariah tradisonal yang letaknya tidak jauh
mencapai 121 persen. dari sini.
(C) Yang tak kalah penting adalah (E) Angin puting beliung telah membuat
penegakan substansi HAM itu perkampungan porak-poranda.

bimbelocean.com
134
UTUL UGM
PAKET 7
32. Nama-nama calon ketua perkumpulan ini (D) Interaksi antarkebudayaan dalam
telah tercatat dalam daftar. kelompok masyarakat dijalin tidak
Makna imbuhan ter- seperti dalam kalimat hanya meliputi suku bangsa yang
di atas ada pada kalimat berbeda, melainkan juga
(A) Pengumuman itu tertempel di papan antarperadaban di dunia.
pengumuman. (E) Banyak juga dijumpai seni tari yang
(B) Suara-suara kendaraan di jalan raya memiliki nilai-nilai keagamaan.
terdengar sampai di ruang ini.
(C) Karena mengantuk, beberapa 35. Yang termasuk kalimat baku adalah
anggota rapat tertidur. (A) Habisnya hujan di wilayah hulu
(D) Pengunjung dapat tergelincir jika membuat air hujan tidak dapat
tidak hati-hati. diserap tanah sehingga volume air
(E) Beberapa gunung terlihat dari bukit yang melaju di permukaan
ini. meningkat.
(B) Mempertahankan bahasa daerah
33. Pihaknya sedang menyiapkan strategi juga sangat dibutuhkan peran orang
mengolah sampah di dalam kota tua terhadap anak-anaknya untuk
sehingga Jakarta tidak penuh sampah membiasakan berkomunikasi
lagi. dengan menggunakan bahasa
Kalimat yang polanya sama dengan daerahnya.
kalimat di atas adalah (C) Sudah kerap dikemukakan, banjir
(A) Penanganan sampah tidak hanya datang bersamaan dengan
dilakukan dengan menambah menghilangnya wilayah-wilayah
pengangkut sampah, tetapi juga resapan air berupa hutan lindung
dengan menyediakan area dan hutan tutupan di hulu-hulu
penimbunan. sungai.
(B) Kalangan pengusaha terpanggil (D) Tidak adanya kaitan halo matahari
untuk memberikan bantuan truk dengan prediksi gempa sehingga
sampah sebagai wujud masyarakat tidak perlu khawatir dan
persahabatan dengan Jakarta. mengaitkannya dengan gempa.
(C) Peremajaan truk sampah sangat (E) Dari hari ke hari, untuk atas nama
mendesak karena tidak sedikit truk pertumbuhan ekonomi, untuk atas
sampah DKI yang sudah tidak layak nama pertambahan populasi, dan
operasi. entah untuk atas nama apa lagi, kita
(D) Pemerintah DKI Jakarta akan justru terus menambah dosa-dosa
menambah kontainer penimbunan ekologis kita.
sampah di sekitar rumah warga.
(E) Pemerintah akan menyediakan 36. Senyuman dapat menstimulasi otak
insinerator di berbagai wilayah agar sehingga membuat pikiran lebih positif.
biaya pengolahan sampah dapat Makna kata bercetak miring di atas
ditekan. adalah
(A) merangsang.
34. Kalimat yang tidak baku adalah (B) mengalirkan.
(A) Indonesia dengan penduduk 250 juta (C) mengaktifkan.
jiwa di mana mereka tinggal tersebar (D) menghubungkan.
di pulau-pulau Indonesia. (E) membuat jaringan.
(B) Kasus penebangan liar di daerah itu
masih sering terjadi. 37. Meskipun Indonesia adalah negara dua
(C) Pengadilan akan melaksanakan musim, koleksi permainan baru yang
tugasnya untuk memberi hukuman memang mengikuti negara asal produk
setimpal pada para pelanggar. ini, yakni London, tetap bisa dimainkan
oleh anak-anak di tanah air.

bimbelocean.com
UTUL UGM 135
PAKET 7
Inti kalimat di atas adalah
(A) Indonesia negara dua musim. 39. Makna idiomatis terdapat dalam kalimat
(B) Koleksi permainan mengikuti negara (A) Untuk mempercepat pencapaian
asal. tujuan, kami mengambil jalan pintas
(C) Koleksi permainan dimainkan anak- lewat belakang kampus.
anak. (B) Anak yang sering tinggal ketas itu
(D) Koieksi permainan tetap bisa dijuluki oleh teman-temannya si
dimainkan. kerbau.
(E) London negara asal produk. (C) "Perempuan sendirian keluar malam-
malam tak pantaslah!" teriak lbu.
38. Kalimat yang mengandung kata yang (D) Tokoh Gatotkaca berotor kawat dan
bermakna idiomatis adalah bertulang besi.
(A) lndustri batik belum berkembang (E) Mata kita silau apabila menatap
secepat industri lain. Iangsung si raja siang, matahari.
(B) Potensi ekonomi rakyat ini perlu
dikelola agar tumbuh sejalan dengan 40. Melahirkan semangat Pancasila dapat
target nasional, membangun diartikan sebagai keinginan untuk
ekonomi kreatif berbasis kerakyatan. menegaskan diri kembali pada komitmen
(C) Bertempat di Pusat Kegiatan ber-Pancasila.
Mahasiswa, Mendikbud dan para Padanan yang tepat untuk kata komitmen
wakil mahasiswa mengadakan dalam kalimat di atas adalah
pembicaraan dari hati ke hati. (A) tekad.
(D) Ajang pameran Indocraft di Jakarta (B) perjanjian.
itu menyiratkan prospek cerah (C) keinginan.
pengembangan industri batik. (D) kemauan.
(E) Ajang pameran 300 peserta yang (E) tanggung jawab.
memamerkan beragam produk batik
ini menarik perhatian Iebih dari
15.000 pengunjung selama tiga hari.

BAHASA INGGRIS

I. Study the text and choose the best answer to the questions that follow.

1 While bullying has always been a problem encountered during childhood and
adolescence, we all know it is on the rise. When a child is intentionally and repeatedly mean to
another child, the cycle begins. Bullying occurs on the playgrounds, at lunch in the cafeteria, in
locker rooms, even over the phone. And now that so many children have access to the latest
5 technology, the problem has become even more prevalent.
Enter the newest room-cyber bullying-whereby children and teens deliberately use digital
media to disseminate false, embarrassing, or just plain mean messages or pictures about one
person to others. Cyber-bullying can occur via text, email, Facebook, Twitter, or other social
media outlets. The American Academy of Pediatrics calls cyber-bullying “the most common
10 online risk for all teen.”
Studies have shown that between 25 percent to 45 percent of teens report being bullied
online. Many kids report such treatment having occurred more than once. Cyber-bullying is
affecting all ages, as even young children often have access to technology and the internet.
Children of all ages need to understand that the Internet is not a “safe” place, and that it
15 is a public forum. Even if you delete a message or photo, it actually continues to exist in
cyberspace. Also, many teens mistakenly think they will not “get caught” if the bully someone
online, or that such behavior is “not that big a deal.”
All parents need to discuss internet safety and the problem of cyber-bullying with their
children. This is especially important for tweens and teens, as they spend much of their time
20 online. Just as kids must learn and practice good manners in public, they also need to mind
their manners online. If a child would not say something to another young person’s face, then

bimbelocean.com
136
UTUL UGM
PAKET 7
the same dig should not be emailed or texted. It is really as simple as that.
This is what I call the “front door rule”. Tell your child that if he/she writes an email or text,
or posts something derogatory about another person on Facebook, Twitter, Instagram (or other
25 sites), to think before they push “send.” If they would not want to post the information on their
front door for family members and all the neighbrs to see, they should stop, think and change-or
forget about-the message. It could hurt someone more than they realize and could also be
28 forwarded on to hundreds, thousands, even millions of others.

Adapted from Hubbanrd, S. “Teach your kids the hazards of cyber-bullying”

41. Which of the following best describes the 45 Which of the following best describes the
topic of the text? organization of this text?
(A) Limiting children to the access of the (A) Comparison and contrast
latest technology (B) Cause and effect
(B) Informing the younger generation (C) Description
about online intimidation (D) Problem and solution
(C) Being aware of cyber bullying as the (E) Sequence
most common crime nowadays
(D) Understanding the disadvantages of 46. All of the following are stated in the text
using the internet as a public forum about cyber-bullying, EXCEPT
(E) Needing to learn good manners in
public and online during childhood (A) Cyberbullying is done by a child by
sending bad message on purpose with
42. The word “prevalent” in line 5 could best the use of Internet
be replaced by (B) Cyberbullying has made the American
(A) Urgent Academy of Pediatrics have a deep
(B) Obvious concern in people
(C) Serous (C) Cyberbullying gives negative effects
(D) Pressing on people regardless their ages
(E) Widespread (D) The internet and technology have
become outlets for children to bully
43. The pronoun “it” in line 15 refers to others
(A) The internet (E) The more frequent children make use
(B) A public forum of technology, the higher the number
(C) Cyber-bullying of cyber-bullying will be
(D) A message or photo
(E) Cyberspace 47. Which of the following is NOT true
according to the text?
44. it can be inferred from the text that (A) Not more than 45 percent of teenagers
(A) To avoid cyber-bullying, parents have been bullied online as they have
should limit the children’s use of frequent access to the internet
computer (B) Children should change their
(B) Bullying online by children occurs as perception and realize that cyber-
frequently as that by adults bullying can put them in trouble.
(C) If a child does not want to get hurt, (C) Children should behave themselves
then he/she should not deliberately do not only in public but also online
bad things to others (D) Children could unintentionally spread
(D) Bullying has become the number-one a message to lots of people when
problem urgently needing to be solved posting it oline
during childhood or adolescence (E) Children should deliberately express
(E) Bullying through social media outlets bad remarks neither directly to people
is preferably chosen by the internet nor via texts.
users nowadays.

bimbelocean.com
UTUL UGM 137
PAKET 7
48. The word “derogatory” in line 24 could best (E) A stumble may prevent a fall
be replaced by
(A) Disappointing 50. The author’s purpose in this text is to
(B) Annoying (A) Make people realize that the number
(C) Inconvenient of bullying is increasing
(D) Embarrassing (B) Teach children their messages may
(E) Offensive still be available in cyberspace
(C) Inform parents and children the
49. Which of the following proverbs expresses consequences of bullying on the
the idea in paragraph 6? internet
(A) Look before you leap (D) Educate people that bullying may take
(B) You reap what you saw place in various social media outlets
(C) A stitch in time saves nine (E) Notify adults and children to learn and
(D) The weak can never forgive practice good manners

II. The text below has incomplete sentences. Choose the one word phrase from each number that
best completes the sentences.

A common way of gaining knowledge is through experience, which relies on trial-and-error learning.
One of the authors is reminded of a time when he (51)_________ an Old Order Amish farmer while
their two 5-year-old sons played with a small snapping turtle. As the Amish boy held the turtle in one
hand he would reach out with the other and tap the turtle’s beak with his index finger, jerking it away
as the animal (52)_________ in the air. Several times, the farmer interrupted his conversation to warn
his son that if he continued (53)__________ the turtle, he would be sorry. Suddenly, the boy
(54)__________ and dropped the turtle to the ground. As the father retrieved a handkerchief from his
pocket and attended to the young boy’s bleeding finger, the man looked into his son’s eyes and said
in German (55)__________ into “maybe the turtle has taught you what your father could not.” The
author could not help but feel that his son, who stood in horror (56)________ his Amish friend’s
bleeding finger, had also learned something from the experience.
An old adage (57)_________, “Experience is the best teacher.” If our knowledge is (58)________,
however, then it is limited indeed, for experience is the best teacher only if we cannot also find other
paths to knowledge. Because experience is a very (59)_________ of gaining knowledge, it is
influenced by our social and cultural backgrounds. (60)________ is punctuated by diversity as people
of different racial, ethnic, gender, and socioeconomic backgrounds not only experience vastly different
things in life, but, more important, often interpret similar experience quite differently.

Taken from Thompson, William E. et al. 2008. Society in Focus: An Introduction to Sociology. Boston:
Allyn and Bacon, p.29

bimbelocean.com
138
UTUL UGM
PAKET 7

51. (A) Had interviewed 57. (A) Claims


(B) Is interviewed (B) Claimed
(C) Is interviewing (C) Is claimed
(D) Was interviewed (D) Is claiming
(E) Was interviewing (E) Was claimed

52. (A) Harmless snap 58. (A) To limit the personal experiences
(B) Snapped harmlessly (B) Limiting personal experiences
(C) Snapping harmlessly (C) Limited to personal experiences
(D) Was harmlessly snapped (D) Experiencing personal limitation
(E) Was harmlessly snapping (E) To personal limitation of experience

53. (A) Tease 59. (A) Personal and an individual way


(B) Teased (B) Personality and individuality of way
(C) To tease (C) Personal way and individualistic
(D) Teasing (D) Way’s personal and individuality
(E) To teasing (E) Personal and individualistic way

54. (A) Painful squeal 60. (A) Experiential knowledge


(B) Squeals out in pain (B) Experiencing knowledge
(C) Squealed out in pain (C) Experience of knowledge
(D) Squealing out in pain (D) Knowledge experience
(E) Painfully squealed out (E) Experience from knowledge

55 (A) Roughly translates


(B) Is roughly translated
(C) That is rough translation
(D) What roughly translates
(E) What is roughly translated

56. (A) Looked at


(B) Looking at
(C) Looked for
(D) Looking for
(E) Looking out

bimbelocean.com
UTUL UGM 139
PAKET 8

PAKET TES KEMAMPUAN DASAR UMUM

8
WAKTU : 100 MENIT
JUMLAH SOAL : 60

MATEMATIKA DASAR

1. Dalam suatu barisan aritmetika, 5. Jika garis ℎ menyinggung kurva 𝑦 =


𝜋
perbandingan jumlah 5 suku pertama dan cos 𝑥 − sin 𝑥 di titik yang absisnya 4 , maka
jumlah 10 suku pertama adalah 2: 3. Jika garis ℎ meotong sumbu 𝑦 di titik ….
𝑈𝑛 menyatakan suku ke 𝑛, maka nilai 𝜋
𝑈5 𝑈10 (A) (0, 2 √2)
log (𝑈 − 4 ) = ….
10 𝑈5 𝜋
(B) (0, 4 √2)
(A) log 117 − log 11
(B) log 3 − log 32 (C) (0, 𝜋√2)
(C) log 32 − log 34 (D) (0, √2)
(D) log 7 − log 6 (E) (0,2√2)
(E) log 6 − log 7
6. Diketahui 𝑥𝑦 + 𝑎𝑥 2 + 𝑏𝑥 + 𝑐 = 0. Agar
2 −1
2. Diberikan matriks 𝑃 = ( ) dan 𝑄 = 𝑥 + 𝑦 memiliki nilai maksimum/ nilai
4 3
2𝑟 1 minimum relatif, maka ….
( ) dengan 𝑟 ≠ 0 dan 𝑝 ≠ 0. (A) 𝑏2 − 4𝑎𝑐 > 0
𝑟 𝑝+1
𝑏2
Matriks 𝑃𝑄 tidak mempunyai invers (B) >0
4𝑎𝑐
apabila nilai 𝑝 = …. 𝑏
3 (C) >0
(A) − 2 𝑐−1
𝑐
1 (D) >0
𝑎−1
(B) − 2 𝑎
1
(E) >0
𝑏−1
(C) − 4
1
(D)
2
7. Parabola 𝑦 = 𝑎𝑥 2 + 𝑏𝑥 + 𝑐, 𝑎 > 0
(E)
8 memotong sumbu-𝑥 pada 𝑥 = 𝑝 dan 𝑥 =
7
2𝑝,𝑝 ≠ 0. Nilai 𝑐 − 𝑏 > 0 terpenuhi apabila
1 1 1 1 ….
3. Jika sin 𝜃 = 𝑎 − 𝑏 dan cos 𝜃 = 𝑎 + 𝑏, 3
(A) − < p < 0
2 2 2
dengan 𝑎, 𝑏 ≠ 0, maka 𝑎 + 𝑏 = …. 3
(A) 𝑎2 𝑏2 (B) 𝑝 < − 2 atau p > 0
3 3
𝑎2𝑏2 (C) 𝑝 < − 2 atau p > 2
(B)
2
3
(C)
2 (D) 0 < 𝑝 <
2
𝑎2𝑏2
3
(D)
1 (E) 𝑝 < 0 atau 𝑝 > 2
𝑎2𝑏2
(E) 𝑎𝑏
8. Jika {(𝑥, 𝑦, 𝑧)} adalah himpunan
𝑥−6 penyelesaian sistem persamaan
4. Jika diketahui 𝑓(𝑥 − 3) = 𝑥+3, maka
2𝑥 + 2𝑦 = 6
1
𝑓 −1 (2) =…. {𝑥 − 3𝑧 = −8
𝑥 + 5𝑦 = 11
(A) 1 (D) 6
Maka nilai 𝑥 + 𝑦 + 𝑧 = ….
(B) 3 (E) 12
(A) 4 (D) 8
(C) 6
(B) 5 (E) 10
(C) 6

bimbelocean.com
140
UTUL UGM
PAKET 8
9. Diberikan dua persamaan (C) 2
𝑥 2 + 𝑎𝑥 + 1 = 0 (D) 10
𝑥2 + 𝑥 + 𝑎 = 0 (E) 17
dengan 𝑎 ≠ 1. Agar dua persamaan
tersebut mempunyai akar berseriakt maka 14. Jika persamaan kuadrat 3𝑥 2 + 𝑥 − 3 = 0
nilai 𝑎 adalah …. mempunyai akar-akar 𝛼 dan 𝛽, maka
(A) −3 (D) 2 persamaan kuadrat yang akar-akarnya 2 +
1 1
(B) −2 (E) 3 dan 2 + 𝛽+1 adalah ….
𝛼+1
(C) −1
(A) 𝑥 2 + 𝑥 − 11 = 0
(B) 𝑥2 + 𝑥 − 9 = 0
𝑈 −𝑈2
10. Diketahui matriks 𝐴 = ( 1 ) dan 𝑈𝑛 (C) 𝑥 2 + 9𝑥 − 9 = 0
𝑈4 𝑈3
adalah suku ke-𝑛 barisan geometri. Jika (D) −𝑥 2 − 9𝑥 − 11 = 0
1 1 (E) −5𝑥 2 + 𝑥 − 11 = 0
𝑈1 + 𝑈3 = 𝑝 dan 𝑈2 + 𝑈4 = 𝑞 dengan 𝑝, 𝑞 ≠
0, maka determinan 𝐴 sama dengan …. 15. Himpunan penyelesaian pertidaksamaan
1
(A) ( 𝑥 2 − 9 ) √𝑥 + 2
𝑝2 +𝑞2
≤0
𝑝2 𝑥 + √(𝑥 + 2)2
(B) 𝑝2 +𝑞2
𝑝𝑞 2
adalah ….
(C) (A) {𝑥| − 1 < 𝑥 ≤ 3}
𝑝2 +𝑞2

(D)
1 (B) {𝑥|𝑥 > −1}
𝑝+𝑞
𝑞 (C) {𝑥|𝑥 > −2}
(E) (D) {𝑥| − 2 < 𝑥 ≤ 3}
𝑝+𝑞
(E) {𝑥| − 2 < 𝑥 ≤ −1 atau 𝑥 ≥ 3}
11. Diketahui 𝑓 (𝑥 ) = 𝑚𝑥 + 𝑐 dengan 𝑓 −1 (2) =
−3 dan 𝑓 −1 (8) = 6 dengan 𝑓 −1 16. Jika daerah penyelesaian sistem
menyatakan fungsi invers 𝑓. Nilai pertidaksanaan 𝑥 + 𝑦 ≥ 4, 𝑎𝑥 − 𝑦 ≤
(3+ℎ)𝑓(3)−3𝑓(3+ℎ) 0, −𝑥 + 5𝑦 ≤ 20, 𝑦 ≥ 0 berbentuk bidang
lim = ….
ℎ→0 ℎ
segitiga siku-siku dengan siku-siku pada
(A) 4
titik potong garis 𝑥 + 𝑦 = 4 dan 𝑎𝑥 − 𝑦 = 0
(B) 8
maka maksimum 𝑓 = 3𝑥 + 2𝑦 dengan
(C) 10
kendala sistem pertidaksamaan di atas
(D) 16
adalah ….
(E) 24
(A) 8
(B) 10
12. Jika
1 (C) 15
1 1 1 2
𝑥 = (𝑝 −
2

− 𝑞 ) (𝑝 2 −1
+𝑞 −1
+ 2(𝑝𝑞) )−
2 (D) 17
−2 ( −1 (E) 25
dan 𝑦 = (𝑝 + 𝑞) 𝑝 − 𝑞 −1 ) dengan
𝑥
𝑝, 𝑞 > 0, 𝑝 ≠ 𝑞, maka = ….
𝑦 17. Pada sebuah deret geometri diketahui
(A) (𝑝 + 𝑞 )−1 suku ke-6 adalah 162, dan jumlah
(B) (𝑝 + 𝑞 )−2 logaritma suku ke-2, ke-3, ke-4 dan ke-5
(C) (𝑝 + 𝑞 )2 sama dengan 4 log 2 + 6 log 3. Jika suku
(D) √𝑝 + √𝑞 awal positif, maka suku ke-4 deret
(E) √𝑝 − √𝑞 tersebut adalah ….
(A) 6
3 (B) 9
13. Jika √4 ∙ 23−𝑥 = 2𝑦−3 dan 3
log(2𝑥 + 𝑦) =
5 1
(C) 18
−2 9 log 4 ∙ 32
log 64, maka nilai 𝑥 2 − 𝑦 + (D) 27
1 = …. (E) 54
(A) −2
(B) 1

bimbelocean.com
UTUL UGM 141
PAKET 8
18. Dari 10 siswa terbaik, salah satunya Ayu, berdampingan, maka banyak cara duduk 8
akan dipilih 3 siswa untuk mewakili siswa tersebut adalah ….
sekolah. Peluang Ayu terpilih untuk (A) 360 (D) 1440
mewakili sekolah adalah …. (B) 480 (E) 2880
1 2
(A) (D) (C) 720
10 5
1 1
(B) 5
(E) 2 20. Jika 𝑓 (𝑥 ) = √𝑥 + 1, 𝑥 ≥ −1 dan 𝑔(𝑥 ) =
(C) 𝑥+1
3 , 𝑥 ≠ 0, maka (𝑔 ∘ 𝑓)−1 (2) = ….
(D) 𝑥
10 (A) −1
(B) 0
19. Lima siswa pria dan tiga wanita akan 1
(C)
duduk berdampingan dalam satu baris. 2
Jika disyaratkan kedua ujung ditempati (D) 1
pria dan tidak boleh ada 2 wanita duduk (E) 2

BAHASA INDONESIA

Bacaan berikut digunakan untuk menjawab dibaca ulang, lalu diinterpretasikan untuk
soal nomor 21 sampai dengan nomor 23. menemukan ragam hias busana dan aksesosi
yang mendekati aslinya.
Dalam dua bulan berjalan, setidaknya
ada peristiwa kebudayaan penting yang 21. Pernyataan yang tidak berhubungan
menempatkan candi sebagai sumber “energi”. dengan isi bacaan di atas adalah ...
Pada awal Maret 2014, sekelompak seniman (A) Kolaborasi antarseniman
mementaskan lakon “Shima: Kembalinya Sang menghasilkan sebuah pertunjukan
Legenda”. Dalam pementasan pada 1─2 yang unik.
Maret di Gedung Kesenian Jakarta (GKJ) itu, (B) Pementasan drama tari yang
sutradara Putut Budi Santosa melibatkan dua mempertimbangkan temuan-temuan
arkeolog, yakni Edy Sedyawati dari Inda C. arkeologis pada sebuah candi
Noerhadi. Kemudian, dramaturgi pertunjukan merupakan sesuatu yang relatif baru.
berbentuk drama tari disusun berdasarkan (C) Candi Dieng digunakan sebagai
temuan-temuan arkeologis di Candi Dieng. sumber inspirasi dalam pementasan
Dieng dipakai sebagai acuan riset drama tari "Shima: Kembalinya Sang
mengingat keberadaan Ratu Shima sebagai Legenda".
raja negeri Kalingga hanya ditemukan (D) Pementasan drama tari "Shima:
berdasarkan kabar para pencatat dari negeri Kembalinya Sang Legenda"
Tiongkok. Candi Dieng menurut kajian merupakan hasil kerja sama antara
arkeologis memiliki kesamaan zaman dengan sutradara, seniman panggung, dan
keberadaan Kalingga yang diperkirakan arkeolog.
berasal dari 1.500 tahun yang lalu. (E) Para arkeolog memberikan
Kolaborasi antara sutradara sekaligus sumbangan kepada seniman dalam
koreografer Putut Budi Santosa, penata musik menafsirkan bentuk ragam hias
Joko Winarko, perancang busana Lucky busana dan aksesori berdasarkan
Wijayanti, serta pendesain aksesori Terry W. relief dan bentuk Candi Dieng.
Supit, dan para arkeolog menghasilkan
gambaran pertunjukan yang detail. Mereka 22. Inti bacaan di atas adalah ...
bekerja tidak hanya dalam kerangka mengejar (A) Candi Dieng digunakan sebagai
nilai, tetapi juga memperhatikan bentuk-bentuk sumber inspirasi dalam penyusunan
aksesori, busana, dan alat musik yang drama tari "Shima: Kembalinya Sang
diperkirakan ada di masa Kalingga. Tinggalan- Legenda”.
tinggalan, seperti relief dan bentuk candi,

bimbelocean.com
142
UTUL UGM
PAKET 8
(B) Pementasan drama tari “Shima: (B) Manusia tidak menjaga keberadaan
Kembalinya Sang Legenda" pohon bakau sebagai habitat alami
merupakan peristiwa kebudayaan ikan.
yang penting. (C) Populasi ikan tidak berhubungan erat
(C) Kolaborasi antara seniman dan pakar dengan intensitas penangkapan.
arkeologi menghasilkan gambaran (D) Manusia sebagai subjek terlalu
pertunjukan yang lebih berkualitas. berkuasa atas sumber daya Iaut.
(D) Penempatan candi sebagai sumber (E) Selama ini, laut dijadikan objek
penyusunan pertunjukan drama tari sehingga tidak tercipta harmoni
menghasilkan sebuah pertunjukan ekosistem laut dengan aktivitas
yang menyegarkan. manusia.
(E) Pementasan drama tari "Shima:
Kembalinya Sang Legenda" disusun 25. Dampak buruk mengonsumsi minuman
berdasarkan temuan-temuan soda berlebihan ternyata tidak dapat
arkeologis di Candi Dieng. diremehkan. Gula dan asam pada
minuman bersoda dapat mengikis email
23. Topik bacaan di atas adalah ... gigi. Sementara itu, sodium benzoat yang
(A) Pementasan drama tari "Shima: ditemukan dalam soda dapat mengurangi
Kembalinya Sang Legenda”. persediaan potasium dan bisa memicu
(B) Pemanfaatan candi sebagai salah asma. Banyak mengonsumsi minuman
satu sumber inspirasi dalam ringan juga dapat meningkatkan risiko
penyusunan drama tari. sindrom metabolisme, seperti diabetes
(C) Pementasan drama tari yang disusun dan gangguan hati. Kandungan asam
dengan mempertimbangkan temuan- fosfor yang tinggi dalam soda
temuan arkeologis pada sebuah candi. memengaruhi batu ginjal dan masalah
(D) Pertunjukan drama tari yang ginjal Iainnya.
menandai peristiwa penting dalam Pernyataan yang tidak berhubungan
kebudayaan. dengan isi paragraf di atas adalah
(E) Kolaborasi antara seniman dan pakar (A) Mengonsumsi minuman soda
arkeologi dalam pertunjukan drama berlebihan memiliki dampak buruk.
tari. (B) Email gigi dapat terkikis karena
minuman bersoda.
24. Penurunan populasi ikan di kawasan barat (C) Banyak dampak serius akibat
Indonesia, khususnya Laut Jawa dan mengonsumsi minuman soda berlebih.
Selat Malaka, disebabkan oleh intensitas (D) Minum-minuman soda berlebih dapat
penangkapan yang tidak diimbangi menimbulkan beragam masalah.
dengan pengembangbiakan yang cukup. (E) Minum soda tidak memperburuk
Pohon bakau yang menjadi habitat alami kesehatan.
ikan nyaris punah di wilayah itu sehingga
mengurangi jumlah stok ikan secara alami 26. Pemerintah bisa mengatasi masalah
pula. Di sinilah pentingnya revitalisasi belum optimalnya manfaat pembangunan
pesisir dengan konsep baru, yakni ekonomi dengan melakukan redistribusi
menjaga harmoni ekosistem Iaut dengan pendapatan melalui anggaran dan pajak.
aktivitas manusia. Laut seharusnya Di satu sisi, besarnya subsidi energi,
menjadi subjek yang dibutuhkan manusia misalnya, jelas membuat upaya
sehingga perlu dikelola dengan baik dan redistribusi sia-sia karena penerima
bijaksana. subsidi justru kelompok kaya. Di sisi yang
Simpulan paragraf di atas adalah ... lain, pemerintah harus menaikkan
(A) Penurunan populasi ikan terjadi anggaran infrastruktur yang masih rendah
karena tidak ada usaha revitalisasi untuk membuka kesempatan kerja.
pesisir. Dengan demikian, pemerintah harus
menerapkan pajak progresif yang tinggi

bimbelocean.com
UTUL UGM 143
PAKET 8
untuk kepemilikan properti dan mobil. (C) Hal tersebut membuat imajinasi anak
Upaya ini ibarat sekali merengkuh dua Iebih berkembang dan kreativitas
tujuan tercapai: mengatasi kesenjangan mereka pun terangsang.
sekaligus mengurangi subsidi. (D) Ketersediaan perangkat keras dengan
lde pokok paragraf di atas adalah harga yang terjangkau menjadi
(A) Redistribusi pendapatan melalui pemicu signifikan perkembangan
anggaran dan pajak. industri perangkat Iunak.
(B) Belum optimalnya pembangunan (E) Di dalam negeri, sangat banyak obyek
ekonomi. wisata yang dapat dikunjungi.
(C) Penerapan pajak progresif.
(D) Upaya redistribusi. 29. Penulisan huruf kapital yang semuanya
(E) Langkah mengurangi subsidi. benar terdapat dalam kalimat
(A) Kepala Bidang Pensosbud KJRI
27. Faktor risiko yang berhubungan erat Sydney mengunjungi sekolah di
dengan kanker kolon atau kanker yang Australia guna mengajarkan kesenian
menyerang usus besar adalah gaya hidup. Indonesia, seperti angklung, tari
Tak dapat dipungkiri bahwa kanker kolon Saman, dan tari Bali.
merupakan penyebab kematian kedua di (B) Saya tertarik dengan isi artikel
dunia. Pencegahan kanker kolon dapat "Dampak Minuman Dengan Pemanis
dilakukan dengan cara meningkatkan Buatan dan Pengawet Untuk Tubuh
konsumsi serat makanan, termasuk Kita".
prebiotik dan probiotik (sinbiotik) yang (C) Lebih dari 2,5 juta wanita Indonesia
dikenal masyarakat. menjadi Tenaga Kerja Wanita,
lsi pokok paragraf di atas adalah padahal ancaman kekerasan berada
(A) Kanker kolon merupakan penyebab di depan mata.
kematian kedua di dunia. (D) Sistem Kapitalisme sebagai sistem
(B) Kanker kolon yang diakibatkan oleh yang diterapkan oleh kebanyakan
gaya hidup dapat dicegah dengan Negara di dunia memiliki cara
cara meningkatkan konsumsi serat pandang yang khas dan akan
makanan. memengaruhi kebijakan yang
(C) Susu fermentasi merupakan salah dikeluarkan oleh Pemerintah.
satu produk prebiotik dan probiotik (E) Saat ini sudah sangat luas daratan
(sinbiotik) yang dapat mencegah risiko Pedesaan Cemara Jaya yang tergerus
kanker kolon. akibat abrasi Pantai Cemara.
(D) Meningkatkan konsumsi susu
fermentasi merupakan cara yang 30. Kata bentukan yang tidak tepat terdapat
dapat memperkecil risiko terkena dalam kalimat
kanker kolon. (A) Penambangan liar di Kabupaten
(E) Kanker kolon yang merupakan Grobogan sangat memprihatinkan.
penyebab kematian kedua di dunia (B) Koalisi bukan hal yang mudah untuk
dapat dicegah. dilakukan karena membutuhkan
penyeIarasan visi, misi, kepentingan,
28. Penerapan unsur serapan yang tidak tepat dan kompromi publik.
terdapat dalam kalimat (C) Model koalisi parpol untuk
(A) Desainer-desainer tersebut membuat kemenangan pemilu selama ini adalah
koleksi furnitur yang bahan bakunya koalisi transaksional.
sisa-sisa letusan Gunung Etna di (D) Indeks pembangunan manusia
Sisilia, ltalia. merupakan pengukuran perbandingan
(B) Survei pada 2012 menyebutkan antara harapan hidup, melek huruf,
bahwa 86 persen miliarder sukses pendidikan, dan standar hidup untuk
karena usaha mereka sendiri. semua negara.

bimbelocean.com
144
UTUL UGM
PAKET 8
(E) Pelibatan komunitas masyarakat 33. Yang merupakan kalimat baku adalah
menjadi bagian penting dalam usaha (A) Gunung Rinjani adalah merupakan
memberantas vandalisme hingga gunung yang terletak di Nusa
tuntas. Tenggara Barat.
(B) Banyak pulau-pulau yang menjadi
31. Pemakaian tanda koma yang tepat habitat satwa komodo di pulau
terdapat pada kalimat Komodo.
(A) Pertumbuhan apartemen yang (C) Berdasarkan keterangan direktur
menjamur, diikuti dengan pemukiman perusahaan industri tekstil itu tidak
padat di bawah garis ambang Iuap air memenuhi syarat.
(gala) yang kumuh, merupakan gejala (D) Dengan terungkapnya kasus tindak
pertumbuhan kota yang tak terencana. korupsi ini diharapkan dapat ditangani
(B) Jika Indonesia baru memiliki 15 hasil secepatnya.
kebudayaan yang ditetapkan menjadi (E) Sebagai sebuah objek wisata, Gunung
warisan budaya, Jepang dan Korea Bromo memiliki daya tarik tersendiri.
Selatan, sudah memiliki masing-
masing 43 dan 37 buah jenis 34. Untuk tahun 2014, anggaran negara yang
kebudayaan. mencapai Rp150 triliun sebagian besar
(C) Menurut pakar diet klinis, Emilia E. akan dialokasikan untuk revitalisasi
Achmadi, M. S., jika teh berpemanis industri pertahanan.
buatan diminum satu kali saja, efek Kalimat yang memiliki pola yang sama
negatifnya, tidak akan signifikan. dengan kalimat di atas adalah
(D) "Kita harus bisa membuktikan, bahwa (A) Alat yang sudah tidak berfungsi pun,
tenun memang milik masyarakat, dan sebagaimana dipaparkan Military
memiliki keterkaitan dengan Balance, ada yang masih dimasukkan
kehidupan mereka", jelas anggarannya pemeliharaannya.
Wamendikbud Bidang Kebudayaan. (B) Ada empat belas upaya yang telah
(E) Oleh karena itu, data-data yang dilakukan BNPB untuk menanggulangi
dibawa seperti sejarah, hasil dampak bencana nasional.
penelitian, dan pemetaan, menjadi (C) Presiden meminta Komisi
unsur yang kuat, agar tenun NTT Pemberantasan Korupsi (KPK)
terpilih sebagai warisan budaya dunia. memantau proses pengadaan barang
dan jasa, yang tergolong rawan
32. Kelompok pemuda-pemuda yang korupsi, termasuk di tubuh TNI.
tergabung dalam gerakan "RevoIusi Putih" (D) Pada salah satu industri strategis,
membagi-bagikan 10.000 kotak susu di pemesanan barang dan jasa malah
Bundaran Hotel Indonesia, Jakarta. Tidak dikurangi, bahkan dibatasi.
lupa dalam kesempatan tersebut mereka (E) Berdasarkan kajian ini, perebutan
mengajak tokoh-tokoh superhero "Mas posisi di komisi ekonomi yang
Garuda" dan "Arya Bima" untuk terlibat. menduduki peringkat tertinggi
Anak-anak yang memadati wilayah itu pun beberapa di antaranya direpons
sangat antusias menyaksikan kegiatan- secara berlebihan oleh petinggi partai.
kegiatan itu.
Pada paragraf di atas terdapat 35. Kaum miskin dan menengah yang ada di
penggunaan kata ulang yang tidak tepat, sejumlah negara Asia Tenggara dan
yaitu kata Pasifik Barat paling merasakan dampak
(A) pemuda-pemuda polusi udara pada tahun 2012 dengan 3,3
(B) membagi-bagikan juta kematian akibat polusi di dalam rumah
(C) tokoh-tokoh dan 2,6 juta lainnya akibat polusi di luar
(D) anak-anak rumah.
(E) kegiatan-kegiatan Kalimat tersebut merupakan perluasan
dari kalimat inti

bimbelocean.com
UTUL UGM 145
PAKET 8
(A) Kaum miskin dan menengah (C) Telur perlu diperlakukan dengan baik
merasakan dampak polusi udara. agar supaya awet dan daya
(B) Kaum miskin dan menengah ada di simpannya dapat dipertahankan
sejumlah negara. dalam waktu yang lebih lama.
(C) Kaum miskin dan menengah (D) Untuk menjaga kesegaran dan
mengalami kematian. kualitas isinya, telur memerlukan
(D) Ada 3,3 juta orang yang mengalami teknik penanganan yang tepat.
kematian akibat polusi udara. (E) Secara umum prinsip pengawetan
(E) Polusi udara menyebabkan kematian. telur adalah untuk tujuan agar supaya
mencegah masuknya bakteri
36. Masih belum diakuinya aktivitas ekonomi pembusuk ke dalam telur yang dapat
kreatif oleh perbankan membuat mereka merusak telur.
sulit mendapatkan dukungan finansial.
Minimnya modal itu telah memangkas 39. Diskursus budaya tidak hanya berkaitan
kreativitas mereka. dengan sektor seni dan adat-istiadat,
Makna kata memangkas dari kalimat di tetapi juga dengan perilaku dan cara
atas adalah berpkir.
(A) memotong Padanan makna yang tidak tepat untuk
(B) menurunkan kata diskursus dalam kalimat di atas
(C) membatasi adalah
(D) memagari (A) pertukaran ide
(E) menghilangkan (B) bahasan
(C) wacana
37. Kalimat yang mengandung makna (D) persebaran
idiomatis adalah (E) rasionalitas
(A) Tangan kanannya terkilir sehingga
tidak dapat mengangkat barang- 40. Kata yang mengalami perubahan makna
barang berat. menyempit terdapat pada kalimat
(B) Karena dijadikan kambing hitam, ia (A) Para tokoh mengajak masyarakat
menempuh jalur hukum. untuk menggunakan hak pilihnya
(C) Meja hijau ini sudah lama tidak dicat dengan cerdas dan tanpa tekanan
ulang sehingga warnanya sudah agar kursi di DPR terwakili secara
tampak pudar. benar.
(D) Kepalanya keras, sekeras batu hitam. (B) Keutuhan fungsi UN terkait dengan
(E) Karena suatu penyakit, tangannya pemetaan indeks kompetensi,
selalu dingin. kelulusan seleksi, dan perbaikan
terhadap infrastruktur sekolah.
38. Yang merupakan kalimat baku adalah (C) Para sarjana Iingkungan berkumpul di
(A) Pengawet telur dapat dilakukan Kanada untuk mengikuti konferensi
dengan berbagai cara, antara lain internasional tentang perubahan iklim.
pendinginan; membungkus kering; (D) Kapal-kapal Australia berlayar sejak
proses pelapisan dengan minyak; dan beberapa hari yang lalu mencari kotak
mencelupkan dalam berbagai cairan. hitam pesawat MH370.
(B) Telur jika hanya sekadar dikonsumsi (E) "Maaf, apakah Bapak tahu gedung
dalam keadaan segar saja tidak induk tempat seminar berlangsung?"
mampu disimpan dalam waktu yang
terlalu lama.

bimbelocean.com
146
UTUL UGM
PAKET 8

BAHASA INGGRIS

I. The text below has incomplete sentences. Choose the one word or phrase from each number that
best completes the sentences.

Bullying behavior, broadly described as intimidating or harassing another person through physical or
verbal assaults and insults, can begin at any age, but it seems to be particular problem among
children and adolescents. (41)__________ it is usually a one-on-one behavior, bullying also has
broader social impact. Victims may feel humiliated, and thus (42)__________ in their peer group,
while bullies may feel (43)__________ a position of superiority in the same group. Bullying behavior
can continue into adulthood, although by that time it may be regarded as criminal behavior and result
(44)________ legal action (e.g. chargers of “assault and battery”).
The social stratification (45)_________bullying is often a precursor of adult behavior. Children who
are bullying may continue to intimidate, or try to intimidate, their peers (46)_______ they are adults.
Other adults learn to cope with (47)_________ behavior, either by standing up to it and challenging
the bully or by ignoring the behavior and avoiding the bully. Adult society (48)_______ bullies and
victims in different social group.
(49)__________technique for dealing with bullies is part of (50)_________ task of schools to create a
safe environment for all children and to teach acceptable social behavior.
Adopted from http://connection.ebscohost.com/education/bullying-schools/definition-bullying-schools
Accessed March 16, 2014

41. (C) Is caused by


(A) Yet (D) It is caused by
(B) Despite (E) Which causes
(C) Although
(D) However 46.
(E) In spite of (A) For
(B) Since
42. (C) During
(A) Everyone alienates (D) When
(B) Alienated from everyone (E) While
(C) Alienating from everyone
(D) Everyone feels alienated 47.
(E) From being alienated (A) Such
(B) The
43. (C) This
(A) They have been established (D) Those
(B) That have established them (E) These
(C) They have established that
(D) That has established them 48.
(E) That they has established (A) Often tends to place
(B) Tends to often place
44. (C) Placed often to end
(A) From (D) Often tends and places
(B) Of (E) Often tends placing
(C) Into
(D) In 49.
(E) On (A) In developing
(B) The developed
45. (C) The developing
(A) Causing (D) In the developing of
(B) Caused by (E) The development of

bimbelocean.com
UTUL UGM 147
PAKET 8
50. (C) The larger
(A) Large (D) The large
(B) Larger (E) The largest

II. Study the following text then answer the questions

The impact of electronic commerce in a developing country can be helpful rather than
detrimental. Electronic commerce has the potential to tie developing countries into the rest of the world
so they are no longer considered outsiders. For example, electronic commerce can enable more people
to access products and services that once were not available. Another benefit is that electronic
5 commerce stores are available 24 hours a day, 7 days a week. As the infrastructure for electronic
commerce keeps growing, services that were not offered in the past become available. Many of these
benefits have not been proven yet, but the technology is now available, and developing countries,
however, the constant innovation of software and hardware will hopefully reduce these costs.
Consumer in developing countries can benefit electronic commerce because they can buy
10 product that could only be found in major cosmopolitan cities. Electronic commerce is closing the gap
between the countries that have wide availability. The basic purpose of electronic commerce is to
provide goods and services to consumers who do not live close to the physical location of the product or
service and woud otherwise have a hard time acquiring these products and services.
Society and consumers alike have only begun to enjoy the benefits of electronic commerce.
15 Since new developments are made on a continuous basis, it will eventually affect every individual. Some
of the benefits enjoyed by society and consumers, for example, are ease of transaction, comparability of
products, quick delivery and the ability to make any type of transaction at any given time of day.
Electronic commerce facilities delivery of public and social services, such as; healthcare,
education, and distribution of government social services at a reduced cost, improving the quality of care
20 and living in these communities. For example, health care services can reach patients in rural areas.

Adapted from Lubbe S And J.M.van Heerden,2003.The Economic and Social Impacts of Electronic
Commerce.
London: Idea Group Publishing, p.23.

51. What is the topic of the text? (B) These cost


(A) Government policy on electronic (C) Consumers
commerce. (D) Developing countries
(B) Type of business suitable for (E) Major cosmopolitan cities
electronic commerce
(C) The benefits of electronic commerce 54. Where in the text does the author mention
for developing countries the impact of software and hardware
(D) The gap between countries applying innovation?
and not applying electronic commerce (A) Line 1-2
(E) Minimum requirements of software (B) Line 12-13
and hardware for electronic commerce (C) Line 7-8
(D) Line 14-15
52. The word “detrimental” in line 2 is closest (E) Line 16-17
in meaning to....
(A) Critical 55. It is implied in the text that....
(B) Serious (A) The cost of electronic commerce
(C) Extreme equals to its benefits.
(D) Harmful (B) Electronic commerce breaks the
(E) Desperate barrier of geographical location
(C) Only cosmopolitan cities implement
53. The pronoun they in line 9 refers to.... electronic commerce
(A) Consumers in developing countries

bimbelocean.com
148
UTUL UGM
PAKET 8

(D) Delivery of product relies much on the (C) Provide examples of the use of
types of business transaction electronic commerce
(E) Developing countries experience hard (D) Promote electronic commerce in
time applying electronic commerce developing countries
(E) Show some developing countries
56. Which of the following best describes the applying electronic commerce
organization of this text?
(A) Comparison 59. Which of the following is TRUE according
(B) Spatial order to the text?
(C) Cause and effect (A) Electronic commerce guarantees
(D) Chronological order product quality
(E) General to specific orde. (B) Electronic commerce develops new
technology
57. The text states all of the following (C) Electronic commerce affects rural area
EXCEPT.... development
(A) Electronic commerce makes the (D) Electronic commerce reduces the gap
delivery of public and social services between countries
easier (E) Electronic commercial is subject to
(B) Transaction of electronic commerce regional regulations
can only be made at particular time
(C) Some people still wait to see the 60. It can be inferred from the text that....
benefits of electronic commerce (A) Remote areas need to improve their
(D) Electronic commerce improves quality technology to catch up with
of life businesses
(E) The latest technology is now ready to (B) Electronic commerce is mainly
support electronic commerce accessed by developed countries until
today
58. The purpose of the text is to.... (C) Consumers in developing countries
(A) Explain the advantages of electronic prefer traditional commerce
commerce (D) Society uses electronic commerce for
(B) Describe innovation of electronic particular product only
commerce (E) Electronic commerce can accelerate
services to the public

bimbelocean.com
UTUL UGM 149
PAKET 9

PAKET TES KEMAMPUAN DASAR UMUM

9
WAKTU : 100 MENIT
JUMLAH SOAL : 60

MATEMATIKA DASAR

1. Diketahui barisan geometri dengan (A). 𝑦 ≥ 0, 2𝑦 − 𝑥 ≤ 1, 𝑥 + 𝑦 ≤ 4


jumlah suku ke-1 dan ke-3 adalah 100 (B). 𝑦 ≥ 0, 2𝑦 − 𝑥 ≤ 2, 𝑥 + 𝑦 ≤ 4
dan jumlah suku ke-2 dan ke-4 adalah 75, (C). 𝑦 ≥ 0, 2𝑦 − 𝑥 ≥ 1, 𝑥 + 𝑦 ≤ 4
maka suku pertama barisan tersebut (D). 𝑦 ≥ 0, 2𝑦 + 𝑥 ≤ 2, 𝑥 + 𝑦 ≥ 4
adalah … (E). 𝑦 ≥ 0, 2𝑦 + 𝑥 ≤ 2, 𝑥 + 𝑦 ≤ 4
(A) 24
(B) 27 5. Jika jumlah suku ke-1 dan ke-3 deret
(C) 36 geometri adalah -5 dan suku ke-2
(D) 48 dikurangi suku ke-3 sama dengan 6 maka
(E) 64 jumlah suku ke-3 dan suku ke-4 deret
tersebut adalah …
2. Jika 𝐴 memenuhi (A). −18 atau −12
2 1 −1 −2 −1 0 (B). −9 atau −4
( )𝐴 +( )=( )
1 1 0 −1 1 1 (C). 18 atau 12
Maka det 𝐴 =…
(D). 9 atau 4
(A). 0
1
(E). 18 atau 4
(B). − 2
(C). −1 6. Diketahui parabola 𝑦 = 𝑥 2 − 4𝑥 + 6
(D). −2 dipotong oleh garis 𝑙 di dua titik berbeda.
(E). −3 Jika garis 𝑙 melalui titik (3,2) dan
mempunyai gradien 𝑚, maka …
1+√4−𝑥 2 (A). −4 < 𝑚 < 0
3. Semua nilai yang memenuhi >
𝑥 2 −𝑥 (B). 0 < 𝑚 < 4
0 adalah… (C). 𝑚 < 0 atau 𝑚 > 4
(A). −2 ≤ 𝑥 < 0 atau 1 < 𝑥 ≤ 2
(D). 𝑚 < 1 atau 𝑚 > 4
(E). 𝑚 < −4 atau 𝑚 > 1
(B). −2 < 𝑥 < 0 atau 1 < 𝑥 < 2
7. Jika (𝑥, 𝑦) adalah salah satu solusi sistem
(C). −2 ≤ 𝑥 < −1 atau 0 < 𝑥 ≤ 2
persamaan
(D). 𝑥 < 0 atau 𝑥 > 1
𝑥 2 + 𝑦 2 − 16𝑥 + 39 = 0
(E). 0 < 𝑥 < 1 {
𝑥2 − 𝑦2 − 9 = 0
Maka 𝑥 + 𝑦 = …
4. Pada gambar di bawah ini, daerah yang
(A). 9
diarsir memenuhi system pertidaksamaan
(B). 6
(C). 5
(D). 3
(E). 4

bimbelocean.com
150
UTUL UGM
PAKET 9
1
8. Jika dapat disajikan sebagai 12. Jika 𝑥 dan 𝑦 memenuhi
√2+√3+√5 1
2
𝑎√2+𝑏√3+𝑐√30 log 𝑥 2 +3 log 𝑦 3 = 4 dan
,
12 2
log 𝑥+3 log 𝑦 4 = 13, maka
Maka 𝑎 + 𝑏 + 𝑐 =… 4
log 𝑥−𝑦 log 9 = ⋯
(A). 0
(A). 9
(B). 1
(B). 6
(C). 2
(C). 5
(D). 3
(D). 3
(E). 4
(E). 4
9. Jika 𝑎 𝑥 = 𝑏 𝑦 = 𝑐 𝑧 dan 𝑏2 = 𝑎𝑐, maka
13. Garis lurus yang menyinggung kurva 𝑦 =
𝑥 =… 3
2𝑦𝑧 √6 − 𝑥 di titik 𝑥 = −2 akan memotong
(A). 𝑦+𝑧 sumbu-x di titik …
2𝑦𝑧
(B). (A). (18, 0)
2𝑧−𝑦
2𝑦𝑧 (B). (19, 0)
(C). 2𝑦−𝑧 (C). (20, 0)
𝑦𝑧
(D). 2𝑦−𝑧
(D). (21, 0)
(E).
𝑦𝑧 (E). (22, 0)
2𝑧−𝑦

14. Luas minimum segitiga yang dibentuk


10. Diketahui persamaan kuadrat oleh garis lurus yang melalui titik (4,3)
𝑥 2 − 2𝑥 − 3 = 0 ……………… (1) dengan sumbu-sumbu koordinat adalah
𝑥 2 − 𝑎𝑥 + 𝑏 = 0 ……………… (2) …
Jika jumlah kedua akar persamaan (2) (A). 12
sama dengan tiga kali jumlah kedua akar (B). 16
persamaan (1) dan kuadrat selisih kedua
(C). 20
akar persamaan (1) sama dengan
(D). 24
kuadrat selisih kedua akar persamaan
(E). 26
(2), maka 𝑏 =…
(A). 𝑏 = 4
15. Semua nilai 𝑥 yang memenuhi
(B). 𝑏 = 5 pertidaksamaan
(C). 𝑏 = 6 2 2
(2 log(𝑥 + 6))(𝑥 −3 log 8)+𝑥 −3 log 8 > 3
(D). 𝑏 = 7
Berada pada…
(E). 𝑏 = 8
(A). −3 < 𝑥 < −2 atau 2 < 𝑥 < 5
(B). −5 < 𝑥 < −2 atau 2 < 𝑥 < 3
11. Titik 𝑃1 (𝑥1 , 𝑦1 ), 𝑃2 (𝑥2 , 𝑦2 ), … , 𝑃10 (𝑥10 , 𝑦10 )
(C). −3 < 𝑥 < −√3 atau √3 < 𝑥 < 5
dilalui oleh garis 𝑔 yang mempunyai
(D). 𝑥 < −2 atau 𝑥 > 2
persamaan 𝑦 + 2𝑥 − 3 = 0. Bilangan-
(E). −3 < 𝑥 < 5
bilangan 𝑥1 , 𝑥2 , … , 𝑥10 membentuk barisan
aritmatika. Jika 𝑥10 = 2 dan 𝑦5 = 7, maka
16. Diberikan fungsi 𝑓 dan 𝑔 dengan
𝑦7 =…
19 𝑓(𝑥 − 2) = 3𝑥 2 − 16𝑥 + 26 dan
(A). 5 𝑔(𝑥 ) = 𝑎𝑥 − 1. Jika (𝑓 ∘ 𝑔)(3) = 61, maka
17
(B). 5
nilai 𝑎 yang memenuhi adalah …
(C).
15 (A). −2
5 8
13 (B).
(D). 9
5 9
11 (C).
(E). 8
5
(D). 2
(E). 4

bimbelocean.com
UTUL UGM 151
PAKET 9
𝑥 2 +𝑎𝑥+𝑏 19. Jika 𝑐𝑜𝑠 2 𝑥 = √3 sin 𝑥, maka 𝑠𝑖𝑛 𝑥 = …
17. Jika lim = −4, maka nilai𝑎 + 𝑏
𝑥→−1 𝑥 2 +3𝑥+2 1−2√3
adalah … (A). 2
(A). −1 (B).
1−√3
2
(B). −2
2−√3
(C). −3 (C). 2
(D). −4 √7+√3
(D).
(E). −5 2
√7−√3
(E). 2
18. Mimi mendapatkan nilai rata-rata 6 untuk
3 kali ulangan Matematika, nilai rata-rata 20. Panitia jalan sehat akan membuat kupon
7 untuk 3 kali ulangan Biologi dan nilai bernomor yang terdiri dari empat angka
rata-rata 8 untuk 4 kali ulangan Bahasa berbeda yang disusun dari angka 0,1,3,5,
Inggris, dan masih ada 5 ulangan dari dan 7. Jika angka pertama atau terakhir
ketiga pelajaran tersebut yang akan tidak boleh nol, maka banyak kupon yang
diikuti Mimi. Agar Mimi mendapatkan nilai dapat dibuat adalah…
rata-rata untuk tiga mata pelajaran (A). 48
minimal 7,2, maka Mimi harus (B). 72
mendapatkan nilai rata-rata 5 ulangan (C). 96
minimal … (D). 108
(A). 7,2 (E). 120
(B). 7,3
(C). 7,4
(D). 7,5
(E). 7,6

BAHASA INDONESIA

Bacaan berikut digunakan untuk menjawab Faktor yang membuat FOMO semakin
soal nomor 21 sampai dengan nomor 24. fenomenal adalah tingkat pengguna media
sosial yang sangat tinggi saat ini, terutama di
Kemajuan teknologi membuat siapa tanah air. Hasil penelitian tentang FOMO
saja memanfaatkannya. Tak jarang kemajuan menunjukkan bahwa sampel objek di bawah
teknologi membuat orang kecanduan dan tidak usia 30 tahun memiliki kecenderungan
bisa lepas dari perangkat digital dalam tertinggi mengalami FOMO. Selain itu,
hubungan sosial. Ada sindrom yang membuat uniknya, perempuan disebut lebih banyak
orang-orang takut dan gelisah jika berjauhan mengalami FOMO daripada laki-laki.
dengan perangkat teknologi dan kehidupan di (Sumber: Kartini, September 2015, hlm. 68)
media sosial, yaitu Fear of Missing Out
(FOMO). Lalu, mengapa fenomena FOMO ini 21. Pernyataan yang tidak berhubungan
lebih banyak terjadi pada perempuan? dengan bacaan di atas adalah ...
FOMO adalah sebuah fenomena (A) FOMO lebih banyak dialami oleh
mengenai seseorang yang merasa takut dan perempuan.
gelisah jika tidak mengikuti obrolan dan isu-isu (B) FOMO adalah penyakit yang sama
di media sosial. FOMO disebut-sebut sebagai dengan nomophobia.
pemicu terjadinya nomophobia atau ketakutan (C) Pengguna media sosial yang sangat
jika berjauhan dengan telepon genggam. tinggi membuat FOMO semakin
Beberapa jenis gangguan kejiwaan tersebut fenomenal.
disebabkan oleh fenomena FOMO. FOMO (D) Kemajuan teknologi membuat orang
telah diteliti secara mendalam dan kecanduan menggunakan perangkat
dipublikasikan di jurnal Computers in Human digital dalam hubungan sosial.
Behaviour pada tahun 2013 silam.

bimbelocean.com
152
UTUL UGM
PAKET 9
(E) Kecenderungan tertinggi FOMO (C) Aksi pendidikan berwawasan
dialami oleh orang di bawah usia 30 meliputi usaha-usaha melokalisir
tahun. bidang-bidang yang berpotensi
berhasil mengarahkan warga negara
22. Maksud “fenomenal” dalam bacaan di dalam bidang hukum.
atas adalah .... (D) Pemerintah diharapkan dapat
(A) ajaib memberikan respons positif terhadap
(B) menghebohkan gagasan bahwa membentuk negara
(C) luar biasa sebagai sarana membangun jiwa.
(D) menggejala (E) Alih-alih bersikap konfrontatif, dia
(E) bombastis justru menunjukkan sikap kooperatip
dalam tindak pidana yang
23. Simpulan isi bacaan di atas adalah ... didakwakan kepadanya.
(A) FOMO lebih banyak dialami oleh
perempuan yang usianya di bawah 26. Pemakaian tanda baca yang tepat
30 tahun. terdapat dalam kalimat
(B) Nomophobia dipicu oleh FOMO. (A) Kita sering mendengar, upaya
(C) Kemajuan teknologi memunculkan membangun kota cerdas, kota
sindrom baru dan kecanduan. berwawasan lingkungan kota hijau
(D) FOMO adalah sindrom baru yang hingga kota kreatif
menjadi perhatian khusus. (B) Indonesia berkepentingan
(E) Media sosial membuat orang menumbuhkan pelaku wirausaha
cenderung mengalami nomophobia karena berciri inovatif, berani
dan FOMO. mengambil risiko, berorientasi ke
depan, berjiwa merdeka
24. Pertanyaan yang jawabannya terdapat (C) Karakteristik pembangunan kota ─
dalam bacaan di atas adalah ... hampir tidak dapat dipungkiri ─
(A) Mengapa FOMO menjadi pemicu mengandaikan pemerintah kota yang
terjadinya nomophobia? paling menentukan dan paling
(B) Di mana dilakukan penelitian terkait bertanggung jawab
FOMO dan nomophobia? (D) Pekerjaan sebagai petani
(C) Kapan FOMO mulai fenomenal? ditinggalkannya, jumlah penduduk
(D) Mengapa orang takut dan gelisah bertambah cepat, sumber daya desa
ketika tidak menggenggam telepon menyusut, dan kemakmuran di kota
genggam? meningkat.
(E) Apa hasil penelitian yang ada dalam (E) Besarnya jumlah orang desa tidak
jurnal Computers in Human berketerampilan khusus yang
Behaviour? berpindah ke kota menimbulkan
masalah sosial; pengangguran
25. Bentuk serapan yang benar terdapat bertambah, kriminalitas meningkat.
dalam kalimat
(A) Perluasan pembangunan sampai ke 27. Pemakaian huruf kapital yang tepat
wilayah pedalaman dilakukan terdapat dalam kalimat
dengan tetap menempatkan warga (A) Bukit yang sekarang dikenal sebagai
sekitar sebagai subyek kemajuan. Punthuk Setumbu terletak sekitar 4
(B) Pemerintah memiliki program Km di sebelah barat Candi
bantuan langsung tunai untuk Borobudur.
menurunkan angka kemiskinan, (B) Raffles adalah seorang Gubernur
tetapi sebenarnya akan lebih bagus pemerintahan kolonial yang bertugas
apabila dana tersebut dimanfaatkan di Jawa pada tahun 1811 ─ 1815.
untuk memacu kreatifitas (C) Bali memiliki banyak tempat wisata,
masyarakat. salah satunya Air Terjun Nungnung

bimbelocean.com
UTUL UGM 153
PAKET 9
di wilayah Badung, Dusun mendukung strategi penumbuhan
Nungnung, Desa Plaga, Kecamatan ekonomi.
Petang. (E) Sudah ada upaya penanaman, tetapi
(D) Dari aspek biologi, kawasan Danau hasilnya belum sesuai dengan
Toba memiliki keunikan tersendiri, harapan.
yaitu adanya Pohon Hanara.
(E) Beberapa Objek Wisata yang dapat 30. Kalimat yang tidak baku adalah ...
dikunjungi di sekitar Pantai Tanjung (A) Forum ekonomi dunia menerbitkan
Kelayang antara lain adalah Pulau peta risiko yang dihadapi dunia
Burung Garuda, Pulau Batu berlayar, berdasarkan indikator ekonomi,
dan Pulau Lengkuas. lingkungan sosial, geopolitik, dan
teknologi.
28. Makna pengulangan kejar-kejaran dalam (B) Perekonomian dunia sudah
kalimat Di kanan kiri kapal yang saya beberapa waktu hadapi kondisi yang
tumpangi, ikan lumba-lumba berenang sangat tidak menentu.
kejar-kejaran beradu cepat juga terdapat (C) Negara-negara pemilik hutan tropis
dalam kalimat ... terbesar terus mengubah hutan
(A) Antrean para calon penumpang KA mereka menjadi lahan peternakan,
khusus liburan panjang akhir pekan area tambang batu bara, dan
berurut-urutan sampai gerbang perkebunan kelapa sawit.
stasiun. (D) Sesuai dengan siklus biologinya,
(B) Proses belajar-mengajar di sekolah pohon akan menghisap dan
itu berlangsung dengan disiplin tinggi mengubah kabon menjadi biomassa.
da peuh semangat kejuangan. (E) Dunia mungkin tidak akan dapat lagi
(C) Masa studinya yang panjang di salah bergantung pada hamparan pohon
satu negara Semenanjung Arabia dan tumbuhan raksasa.
menyebabkan gaya bicaranya
kearab-araban. 31. Selain antisipasi terhadap situasi global
(D) Setinggi-tinginya harga lukisan akhir yang belum akan membaik dua tahun ke
abad ke-19 itu tida akan mencapai depan, langkah mempertahankan suku
tiga kali lipat tawaran harga lelang bunga The Fed paling tidak akan
yang pertama. mengurangi potensi gejolak di pasar uang
(E) Meskipun hanya melakukan latihan dan bursa saham negara berkembang.
rutin menjelang pertandingan, atlet Inti kalimat di atas adalah ...
bela diri yang tengah naik daun itu (A) Perlu antisipasi terhadap situasi
terlihat pukul-memukul dengan mitra global.
tandingnya. (B) Situasi global belum akan membaik.
(C) Langkah mempertahankan suku
29. Penggunaan kata berimbuhan yang tepat bunga.
terdapat dalam kalimat ... (D) Langkah mengurangi potensi
(A) Untuk menata kembali koleksi dan gejolak.
administrasi, dibutuhkan program (E) Bursa saham negara berkembang.
kegiatan penyiangan.
(B) Dalam kegiatan tersebut, dilakukan 32. Hipotesis mengenai asal mula Gunung
penghitungan koleksi untuk Nglanggeran sebagai pusat erupsi purba
mengetahui jumlah keseluruhan dimunculkan karena pertimbangan
koleksi yang ada. morfologinya yang kontras dengan
(C) Selain itu, dilakukan pula perbaikan sekitarnya, padahal masih dalam satu
atas koleksi yang rusak agar dapat formasi dengan batuan yang sama.
dimanfaatkan lagi. Kalimat inti di atas adalah ...
(D) Indonesia membutuhkan revolusi (A) Hipotesis dimunculkan
kebijakan investasi untuk (B) Asal mula Gunung Nglanggeran

bimbelocean.com
154
UTUL UGM
PAKET 9
(C) Pusat erupsi purba dimunculkan adalah meningkatkan efisiensi dan
(D) Gunung Nglanggeran satu formasi produktivitas bank-bank pemerintah,
dengan batuan yang sama baik milik pemerintah pusat maupun
(E) Asal mula Gunung Nglanggeran pemerintah daerah.
kontras
35. Orang terpesona, terpana, dan mengira
33. Kalimat yang tidak baku adalah bahwa dia sedang berada di suatu tempat
(A) Berdasarkan keterangan Direktur yang sangat eksotis ketika gerhana
perusahaan menerangkan bahwa matahari penuh.
pihaknya tidak tahu adanya Padanan kat yang tepat untuk kata
pemalsuan terhadap produk mereka. eksotis dalam kalimat di atas adalah ....
(B) Penjelasan daripada karyawan yang (A) indah
bekerja di perusahaan itu sangat (B) asing
diperlukan. (C) seksi
(C) Kenferensi pers akan dilakukan agar (D) merangsang
supaya permasalahan pemalsuan (E) menarik
produk ini dapat menemukan titik
terang. 36. kata bercetak miring yang mengalami
(D) Mereka mendatangi perusahaan itu perluasan makna terdapat dalam kalimat
hanya sekedar untuk mengonfirmasi (A) meskipun tahun 2016 perekonomian
isu yang beredar saja. Indonesia diperkirakan masih suram,
(E) Pada akhirnya, semua pihak opttimisme terhadap Indonesia
mendapatkan bagian sesuai dengan sebagai kekuatan ekonomi dan
yang sudah mereka usahakan. pasar besar masih kuat.
(B) Kariernya dalam dunia hiburan tidak
34. Tingkat suku bunga yang tinggi akan redup walaupun beberapa kali isu
mengganggu daya saing dunia usaha di tidak sedap menerpanya.
pasar regional dan internasional pada era (C) Di muka sidang pengadilan,
MEA yang sudah dimulai. terdakwa membeberkan pihak-pihak
Kalimat di atas mempunyai pola yang yang menerima amplop dan tindak
sama dengan kalimat pidana korupsi yang didakwakan
(A) Dalam mendorong investasi swasta, kepadanya.
tantangan berada pada kemampuan (D) Ketimpangan kepemilikan lahan
pemerintah mewujudkan janji dalam berakibat pada ketimpangan
paket stimulus perekonomian. kemakmuran, terutama bagi rakyat
(B) Rekomendasi Bank Dunia adalah yang menggantungkan hidupnya
memperkuat investasi swasta pada penguasaan tanah.
karena perluasan fiskal tidak akan (E) Dalam praktik sehari-hari, fungsi
dapat mendorong pertumbuhan tanah semakin jauh dari isi Pasal 6
ekonomi 5%. Undang-Undang Nomor 5 Tahun
(C) Kelompok bank pemerintah 1960 tentang Pokok-Pokok Agama.
menguasai lebih dari separuh pasar
industri perbankan di setiap provinsi 37. (1) Persepsi masyarakat yang bersifat
sejak runtuhnya bank-bank swasta positif terhadap cendana tersebut dapat
pada beberapa tahun yang lalu. menjadi modal sosial bagi upaya
(D) Pemerintah mengeuarkan Surat pelestarian cendana.
Utang Negara (SUN) untuk (2) Meskipun dilakukan secara
menambah modal bank nasional tradisional, tingkat keberhasilannya cukup
yang kekurangan modal dan menggembirakan, terlebih lagi jika
mengatasi kredit yang bermasalah. dilakukan peningkatan penguasaan teknik
(E) Cara paling mudah untuk budi daya sesuai dengan standar
menurunkan tingkat suku bunga petunjuk teknis budi daya cendana.

bimbelocean.com
UTUL UGM 155
PAKET 9
(3) di antara pihak-pihak tersebut, menumpuknya gas rumah kaca. Gas ini
terdapat masyarakat yang secara akan menjadi isolator yang menahan
swadaya dan mandiri sudah melakukan lebih banyak panas matahari yang
upaya pelestarian cendana. dipancarkan ke bumi. Memahami berapa
(4) dalam upaya pelestarian tersebut, banyak metana yang dipancarkan ke
masyarakat masih menggunakan teknik bumi dan memahami berapa banyak
budi daya dan pemeliharaan cendana metana yang dipancarkan ke atmosfer
secara tradisional. dari semua sumber akan membantu para
(5) penurunan populasi cendana yang ilmuwan untuk memperhitungkan jumlah
terjadi di Nusa Tenggara Timur(NTT) gas rumah kaca global dan mengambil
beberapa dekade terakhir menyebabkan langkah-langkah untuk mengurangi
berbagai pihak mengupayakan aksi dampaknya.
pelestarian cendana. Simpulan paragraf di atas adalah ...
Urutan yang tepat agar menjadi paragraf (A) Suhu rata-rata atmosfer, laut, dan
yang padu adalah .... daratan meningkat.
(A) (1), (2), (3), 4), (5) (B) Tumpukan karbon dioksida, metana,
(B) (1), (4), (2), (5), (3) dan dinito oksida mengumpulkan di
(C) (5), (4), (1), (3), (2) atmosfer.
(D) (5), (3), (4), (2), (1) (C) Gas rumah kaca akan menghalangi
(E) (5), (4), (2), (3), (1) sinar matahari ke bumi.
(D) Pemanasan global muncul karena
38. Para peneliti mempelajari tulang dan gigi peningkatan rerata suhu atmosfer.
dari 904 fosil mamalia berplasenta untuk (E) Jumlah gas rumah kaca dapat
mengukur perbedaan anatomi dan dihitung dan direduksi.
antarspesies. Informasi yang didapat
kemudian digunakan untuk memperbarui 40. Saturnus awalnya memiliki beberapa ring
pohon silsilah 177 spesies Eutheria ─ purba yang lebih banyak mengandung
kelompok mamalia yang lebih dekat partikel daripada yang mereka punya saat
kekerabatannya dengan manusia ini. Satelit-satelit Saturnus terbentuk dari
daripada dengan kanguru. Sebanyak 94 partikel-partikel yang saling bergabung.
fosil di antaranya berasal dari era Pada tahap akhir formasi, beberapa
Paleosan. Ini adalah pohon silsilah satelit kecil terbentuk dalam orbit
terbesar yang menggambarkan mamalia berdekatan. Data yang diperoleh dari
dari era Paleosan. Silsilah yang dianalisis wahana Cassini mengindikasikan bahwa
dengan pembagian waktu mulai 140 juta satelit-satelit kecil yang mengorbit di sisi
tahun lalu hingga hari ini tersebut luar sistem cincin utama mempunyai inti
menunjukkan perubahan keragaman padat.
spesies mamalia. Pernyataan yang tidak dikemukakan
Ide pokok paragraf di atas adalah .... dalam paragraf di atas adalah ...
(A) Fosil mamalia (A) Jumlah partikel ring purba
(B) Asal fosil mamalia berkurang.
(C) Silsilah mamalia (B) Partikel-partikel saling berhubungan.
(D) Keragaman spesies mamalia (C) Temuan itu dapat mengungkap
(E) Perubahan spesies mamalia sistem satelit lain.
(D) Letak orbit satelit kecil tidak
39. Pemanasan global terjadi ketika suhu berjauhan.
rata-rata atmosfer, laut, dan daratan (E) Beberapa satelit kecil tidak berada
meningkat. Hal itu disebabkan oleh dalam cincin.

bimbelocean.com
156
UTUL UGM
PAKET 9
BAHASA INGGRIS

I. The text below has incomplete sentences. Choose the one word or phrase from each number that
best completes the sentences.

Modern parents of intelligent children who understand underperform at school can easily relate to
this story. The problems Thomas Edison experienced, such as a difficulty (41)_________ for long
periods of time on one topic are common in many intelligent children. Their problems in school may
be (42)__________ a number of different causes, ranging from other boredom to dyslexia or some
other learning disorder.
Many sources attribute Thomas Edison’s troubles in school to attention deficit hyperactivity disorder
(ADHD). Edison did seem (43)________ many common caharacteristics of children with ADHD, such
as fidgeting or squirming frequently; drifting off into daydreams more than other children, having
difficulty following instructions or finishing activities, such as homework, interrupting a speaker, such
as a teacher, (44)________, and being impulsive.
Although ADHD is considered to be (45)________ that can adversely affect a child’s learning, a
number of persons in the field of child behavior now feel that this is a misnomer rather than thinking of
ADHD as an impairment this school of thoughts prefers (46)_________ normal children and ADHD
children are primarily different only in (47)___________ not in their ability to learn. An ADHD child, for
example, might learn well in (48)_______ or hands-on experiment, whereas a normal child might
learn well by listening to a teacher give a lecture or completing a worksheet. One analogy is to think of
ADHD children as (49)____________ left-handed children, they are certainly different than most other
kids, (50)_____ that is not to say that they are wrong.

Modified from Scott Teel 2007


Defending and Parenting Children Who Learn Differently

41. (C) Easily distracted


(A) Focused (D) Distracted easily
(B) On focusing (E) Is easily distracted
(C) Focused on
(D) Focusing 45.
(E) Focusing on (A) Impaired health
(B) Impairing health
42. (C) Impair health
(A) Because (D) A health impairment
(B) Caused (E) Health impairing
(C) Due to
(D) Resulted from 46.
(E) Resulting in (A) In believing that
(B) To believe
43. (C) Believe that
(A) Sharing (D) Believing in
(B) To share (E) To believe in
(C) Shared
(D) To be shared 47.
(E) To be sharing (A) How to learn
(B) How do they learn
44. (C) They learn how to
(A) Being easily distracted (D) Learning how to
(B) Being easy to distract (E) How they learn

bimbelocean.com
UTUL UGM 157
PAKET 9
48. (C) Similarity to
(A) A visually oriented activity (D) Being similar to
(B) A visual orientation of activity (E) Being similarly with
(C) An activity of visual orientation
(D) An active visual orientation 50.
(E) A visually active orientation (A) And
(B) So
49. (C) But
(A) Similarly between (D) Thus
(B) Similar with (E) Moreover

II. Study the text and choose the best answer to the questions that follow
1 Nanotechnology has been around for two decades, but the first wave of applications is only now
beginning to break. As it does, it will make the computer revolution look like small change. It will affect
everything from the batteries we use to the pants we wear to the way we treat cancer.
The main thing to know about nanotechnology is that it is small. Really small Nano, a prefix that
5 means “dwarf” in Greek, is shorthand for nanometer, one billionth of a meter a distance so minute that
comparing it to anything in the regular world is a bit of joke. This comma, for instance, spans about half a
million nanometers. To put it another way, a nanometer is the amount a man’s beard grows in the time it
takes him to lift a razor to his face.
Nanotechnology matters because familiar materials begin to develop odd properties when they ar
10 nanosize. Tear a piece of aluminium foil into tiny strips, and it will still behave like alumunium even after
the strips have become so small that you need a microscope to see them. But keep chopping them
smaller, and at some point—20 to 30 nanometers, in this case—the pieces can explode. Not all nanosize
materials change properties so usefully, but the fact that some do is a boon. With them, scientists can
engineer a cornucopia of exotic new materials, such as plastic that conducts electricity and coatings that
15 prevent iron from rusting. It is like you shrink a cat, and keep shrinking it, and then at some point, all at
once, it turns into a dog.
Substances behave magically at the nano scale because that is where the essential properties of
matter are determined. Arrange calcium carbonate molecules in a sawtooth pattern for instance, and you
get fragile, crumbly chalk. Stack the same molecules like bricks, and they help form the layers of the
20 tough, iridescent shell of an abalone.
It is a tantalizing idea creating a material with ideal properties by customizing its atomic structure.
Scientists have already developed rarefied tools, such as the scanning tunneling microscope, capable of
viewing and moving individual atoms via an exquisitely honed tip just one atom wide.
In many ways Nano’s invention is like that of plastic. It will be everywhere, in the scalpels doctors use for
25 surgery and in the fabrics we wear. When coffee is spilled on a pair of stain-resistant nanopants from the
Gap, made from fibers treated with fluorinated nanopolymer, it will roll right off.

Adapted from J Kahn, Welcome to the world of nanotechnology

51. What is the main idea of the text? (E) The nanosized structure of a material
(A) Nano is a new technology so useful can be customized to create ideal
that it is comparable to the use of properties
plastics
(B) Nano is the latest and smallest 52. The word “minute” in line 5 is closest in
measurement man has ever invented meaning to....
(C) Being nanosized, materials change its (A) Atomic
properties and behavior (B) Cosmic
(D) Nanotechnology has been around for (C) Enormous
two decades (D) A brief period
(E) Sixty seconds

bimbelocean.com
158
UTUL UGM
PAKET 9

53. What can be inferred from paragraph 1? (E) One nanometer is equal to half the
(A) The application of nanotechnology is size of the dot in the alphabet “I”
more important than the development
of computer technology. 57. Which of the following is TRUE according
(B) There has not been much to the text?
development in the application of (A) We do not have to use a microscope
nanotechnology since it was first to look at materials of 30 nanosize
invented. (B) Not all materials can turn into
(C) When nanotechnology is applied, it is nanosize
like computer technology, which (C) We can develop calcium carbonate
makes small changes into an abalone
(D) Computer technology and (D) One meter is equal to one million
nanotechnology will influence the way nanometers
we use our batteries and the way (E) Useful properties of nanosize
doctors treat cancer materials can be engineered into
(E) Like computer revolution that looks refined tools
small, nanotechnology still gives small
impacts on life. 58. Which of the following is closest in
meaning to the word “boon” in line 13?
54. The pronoun “them” in line 13 refers to (A) Surprise
(A) Properties (B) Advantage
(B) Familiar materials (C) Miracle
(C) Nanosize materials (D) Bonus
(D) Tiny alumunium strips (E) Mystery
(E) New exotic materials
59. Where in the text does the author begin to
. Which of the following is NOT stated in the mention how common materials when they
text? are nanosize show extraordinary quality?
(A) The application of nanotechnology is (A) Paragraph 1
about to begin (B) Paragraph 2
(B) Nanometer is the tiniest distance (C) Paragraph 3
human has ever discovered (D) Paragraph 4
(C) Nanosize materials change their (E) Paragraph 5
properties
(D) Nanotechnology can help develop 60. What is the purpose of paragraph 6?
surgical instruments (A) To describe the future of
(E) At nanoscale the materials change nanotechnology
into useful properties (B) To explain the explosive danger of
microscopic matters
56. It is implied in the text that (C) To illustrate the possible development
(A) Not all common materials when they of products
are nanosize will acquire new (D) To compare the invention of
properties for further use nanotechnology to that of plastic
(B) Plastic is the result of nanotechnology (E) To describe the remarkable substance
(C) Plastic isolates electricity of plastic
(D) Nanotechnology will be exclusively
useful for medical and safety purposes

bimbelocean.com
UTUL UGM 159
PAKET 10

PAKET TES KEMAMPUAN DASAR UMUM

10
WAKTU : 100 MENIT
JUMLAH SOAL : 60

MATEMATIKA DASAR

20√2−25 5
1. Jika 𝑟 = (10+20 , maka (4𝑟 − (C)
√2)(2−√2) 6

2) 2
= …. (D) 1
5
(A) 5 (E) 3
(B) 4
(C) 3 −𝑥+𝑦 1
5. JIka 𝑥 dan 𝑦 memenuhi =2
2𝑥+3𝑦+5
(D) 2 1 2
(E) 1 dan = 2𝑥+𝑦, maka nilai 8(𝑥 + 𝑦)
−𝑥+𝑦
adalah ….
2
2. Jika log(𝑎 − 𝑏) = 4, maka (A) 25
2 2
4
log ( + ) = …. (B) 20
√𝑎+√𝑏 √𝑎−√𝑏
2 log 𝑎−4 (C) −15
(A) (D) −20
4
2 log 𝑎+4
(B) (E) −25
4
2 log 𝑎−2
(C) 2
2 log 𝑎+2
(D) 6. Nilai 𝑝 yang memenuhi
2
2 log 𝑎−1 pertidaksamaan (2𝑝 + 4)(𝑝 − 1)2 >
(E) (𝑝 + 2) adalah ….
2
2
(A) 𝑝 > 5
3. Berdasarkan perkiraan kebutuhan 5
(B) 0 < 𝑝 < 2
ketela kota P pada 𝑥 tahun setelah
5
2017 sebesar (C) 𝑝 < 0 atau 𝑝 ≥
2
ℎ(𝑥) = 180𝑥 2 + 540𝑥 + 1080 kuintal. 5
(D) −2 < 𝑝 < 0 atau 𝑝 > 2
Produk ketela kota tersebut pada
tahun yang sama sebesar 𝑓(𝑥) = (E) −2 < 𝑝 < 0
720𝑥 + 20880 kuintal. Untuk
mencukupi kebutuhannya, kota 7. Nilai minimum z=6x+3y di daerah
tersebut harus mendatangkan ketela yang diarsir adalah ….
dari luar kota mulai pada tahun ….
(A) 2020
(B) 2023
(C) 2028
(D) 2029
(E) 2032

4. Selisih akar-akar persamaan 𝑥2 + (A) 3


4 4
2𝑎𝑥 + 3 𝑎 = 0 adalah 1. Selisih 𝑎 dan (B) 6
6
adalah …. (C) 8
1 (D) 10
(A) 2 (E) 12
2
(B) 3

bimbelocean.com
160
UTUL UGM
PAKET 10
1
8. Suku tengah deret aritmatika adalah (D) 20
34. Jika suku pertamanya 4 dan 1
(E)
sukuke-4 adalah 22, maka jumlah 30
semua suku deret tersebut adalah ….
(A) 384 13. Suatu desab berpenduduk 5000 jiwa,
(B) 374 terdiri atas kelompok berpendidikan
(C) 264 terakhir SD, SMP, SMA dan
(D) 228 Perguruan Tinggi (PT). Perbandingan
(E) 154 jumlah penduduk berpendidikan
terakhir SD, SMP, dan SMA sebesar 2
9. Ani memasak di dapur. Dia memiliki : 6 : 4. Jika presentase penduduk
10 liter air. Setiap 40 menit dia berpendidikan PT sebesar 4% dari
menuangkan 10% airnya ke dalam total penduduk desa, maka jumlah
panic masakan. Jika proses memasak penduduk berpendidikan terakhir SD
membutuhkan waktu selama 3 jam, sebesar ….
maka selesai masak, sisa air Ani (A) 2400
sebanyak …. ml. (B) 200
(A) 8100 (C) 1600
(B) 7290 (D) 100
(C) 6561 (E) 800
(D) 5904,9
3
(E) 5314,41 𝑥(2𝑥 2 −3𝑥+1)2
14. lim = ….
𝑥→0 (𝑥 2 −1)√𝑥−1
2 (A) −1
10. Jika 𝑎 memenuhi (𝑎 3)=
0 6𝑎 (B) 0
𝑎 5 20 −1 𝑇 1
( )+( 2 ) dengan 𝐴𝑇 (C) 2
1 0 −2 𝑎 + 5
menyatakan transpose matriks A, (D) 1
3
maka 𝑎2 + 𝑎 = …. (E) 2
(A) 2
(B) 12 𝑥+1
15. Jika 𝑓 (𝑥 + 2) = 𝑥−2 , 𝑥 ≠ 2 dan 𝑔(𝑥 ) =
(C) 20
𝑥 + 1, maka semua nilai 𝑦 = (𝑓 ∘ 𝑔)(𝑥)
(D) 30
yang mungkin untuk 𝑥 ≥ 6 adalah ….
(E) 42
(A) 𝑦 ≥ 2
(B) 1 ≤ 𝑦 ≤ 2
11. Jika 0 < 𝑥 < 2𝜋 dan cot 2 𝑥 +
𝜋
(C) 0 ≤ 𝑦 ≤ 2
2 csc 𝑥 + 2 = 0, maka cos (𝑥 + 2 ) = …. (D) −2 ≤ 𝑦 ≤ 0
(A) −1 (E) 𝑦 < −2
1
(B) − 2
(C) 0 16. Fungsi dengan persamaan 𝑓 (𝑥 ) =
2𝑥+𝑎
(D)
1 memenuhi 𝑓 ′ (1) = 1 dan 𝑓 (𝑏) =
𝑥+2𝑏
2 2
(E) 1 − 3. Nilai 𝑏 yang memenuhi adalah ….
(A) −1
12. 𝐴, 𝐵, 𝐶, 𝐷 dan 𝐸 akan berfoto bersama. 4
(B) − 5
Peluang 𝐴 dan 𝐵 selalu berdampingan 2
(C) − 3
dan 𝐸 selalu berada di ujung kanan
1
adalah …. (D) − 4
2 1
(A) (E)
5 2
1
(B) 5
1
(C) 10

bimbelocean.com
UTUL UGM 161
PAKET 10
√𝑥 2 +4 𝑥 19. Diberikan bilangan asli 𝑎, 𝑏, 𝑐, 𝑑 yang
17. Fungsi 𝑓 (𝑥 ) = − 5, mencapai
3 memenuhi 4 ≤ 𝑎 ≤ 𝑏 ≤ 6 ≤ 𝑐 ≤ 𝑑 ≤ 8.
minimum relative di 𝑥 = …. Rata-rata 4, 𝑎, 𝑏, 6, 𝑐, 𝑑, 8 adalah 6.
5
(A) Banyaknya susunan (𝑎, 𝑏, 𝑐, 𝑑) yang
2
(B)
3 mungkin adalah ….
2
2 (A) 24
(C) 3 (B) 12
1
(D) (C) 9
2
2 (D) 8
(E) 5 (E) 7

18. Jika 2 log(𝑥 + 3) , 2 log(6𝑥 + 2), dan 20. Jika 2𝑦+3𝑥 = 32 dan 𝑥 log(𝑥 + 2) −
2
log(26𝑥 − 2) membentuk barisan 3 𝑥 log 2 = −1, maka 2𝑥 + 𝑦 = ….
aritmetika, maka beda barisan (A) 1
tersebut adalah …. (B) 2
(A) 1
(C) 3
(B) 2 (D) 5
(C) 3 (E) 9
(D) 4
(E) 5

BAHASA INDONESIA

Bacaan ini dipergunakan untuk menjawab soal (C) biaya perpanjangan trayek yang
nomor 1 sampai 2. mahal membuat pemilik angkutan
tidak memperpanjang izin trayeknya
Dinas Perhubungan Gunungkidul telah (D) terjadi perubahan jumlah kendaraan
melakukan kajian terhadap penataan angkutan di Gunungkidul
di daerah tersebut. Armada angkutan (E) penumpang semakin berkurang
perkotaan yang ada ternyata menambah sehingga banyak angkutan yang
beban konsumen, terutama yang berasal dari tidak beroperasi
daerah-daerah sehingga angkutan perkotaan
(angkot) akan dilebur menjadi satu dalam 22. Ide pokok bacaan di atas adalah ....
angkutan pedesaan (angkudes). Para (A) hasil kajian terhadap penataan
penumpang yang berasal dari desa terpaksa angkutan di Gunungkidul
berganti angkutan untuk sampai ke kota, baik (B) penyatuan angkot dan angkudes
pusat pemerintahan kabupaten, pasar, (C) alasan keberatan penumpang
maupun tempat lainnya sehingga menambah angkutan
biaya. Di samping itu, jumlah kendaraan, (D) menyusutnya jumlah angkutan di
khususnya angkudes dan angkot, juga Gunungkidul
semakin menyusut karena dijual ke luar (E) sepinya penumpang angkutan di
daerah atau tidak diperpanjang izin trayeknya Gunungkidul
akibat semakin sepinya penumpang.
(Sumber: Kedaulatan Rakyat, 11 Februari 23. Penulisan unsur serapan yang tidak
2017) sesuai dengan pedoman ejaan terdapat
21. Pernyataan yang tidak berhubungan dalam kalimat ....
dengan isi bacaan di atas adalah .... (A) duduk di bawah pohon rindang dapat
(A) angkutan perkotaan ternyata membuat hati dan pikiran menjadi
menambah beban konsumen lebih tenang sehingga stress pun
(B) penumpang dari desa harus naik hilang
angkutan lebih dari satu kali untuk (B) cara tersebut cukup efektif bila
menuju pusat pemerintahan dilakukan oleh perempuan yang
bimbelocean.com
162
UTUL UGM
PAKET 10
menderita gangguan kecemasan berupa denda atau hukuman
(C) ragam aktivitas di luar ruang
berpengaruh besar terhadap para 26. Pemakaian kata berimbuhan yang tidak
remaja tepat terdapat dalam kalimat ....
(D) untuk mendapatkan sensasi rileks (A) kekeliruan penyimpanan memori di
yang optimal, segala macam gawai otak bisa terjadi setelah mendengar
sebaiknya ditinggalkan atau membaca banyak informasi
(E) Jurnalis sains, Florence Williams, (B) pemecahan ombak untuk
melaporkan bahwa beristirahat di mengurangi abrasi dapat dilakukan
bawah pohon rindang sangat dengan beberapa cara
bermanfaat (C) sebuah organisasi memberikan
pemahaman yang bagus tentang
24. Penulisan kata yang tidak sesuai dengan bakau kepada Kelompok Cinta
pedoman ejaan terdapat dalam kalimat Lingkungan
.... (D) penanaman pohon jati dilakukan di
(A) buah dan sayuran nonorganik tidak lahan seluas dua hektare
selalu berarti tidak sehat (E) penambahan jumlah warga yang
(B) puting beliung telah memporak- dengan sadar ikut menjaga
porandakan kawasan perumahan ekosistem cukup besar
yang baru saja dibangun
(C) buku ini memberi tahu para pembaca 27. Gabungan kata bercetak miring yang
tentang pemikiran brilian sang bukan kata majemuk terdapat dalam
proklamator kalimat ....
(D) rompi antipeluru ini selalu mereka (A) kurikulum terbaru mengamanatkan
pakai ketika melakukan operasi agar setiap mata pelajaran di jenjang
(E) la dapat menyelesaikan tugas itu pendidikan dasar sampai sekolah
dengan baik karena jalan pikirannya tingkat atas mengandung pendidikan
sangat sistematis karakter
(B) kapal-kapal modern juga didorong
25. Penggunaan tanda baca koma (,) yang mematuhi aturan sandar, tidak
tepat terdapat dalam kalimat .... melewati garis batas zona kapal
(A) beberapa mahasiswa yang lolos tes tradisional
dan memenuhi persyaratan, akan (C) pameran pendidikan dan peralatan
mendapat beasiswa dari universitas sekolah paling bergengsi di
terkenal di Tokyo Indonesia diselenggarakan oleh
(B) para mahasiswa yang akan pemerintah bersama universitas
mengikuti tes kemampuan bahasa terkenal
Jepang secara gratis diharapkan (D) pembangunan rumah susun model
mendaftarkan diri di ruang kantor linear menunjukkan kepedulian
(C) para ahli matematika merasa kagum, pemerintah terhadap kebutuhan
ketika mengetahui perhitungan lebah hunian warga ibu kota
yang sangat cermat (E) duta besar luar biasa dan berkuasa
(D) seorang pedagang di pasar penuh adalah pejabat diplomatik
mengatakan, sambil melayani yang ditugaskan ke negara lain
pelanggannya, bahwa kenaikan untuk mewakili negara
harga cabai dan bawang merah
sudah berlangsung selama tiga 28. Kalimat yang polanya tidak sama dengan
tahun kalimat yang lain adalah ....
(E) para penyelenggara negara yang (A) banyak buku dan tulisan mampu
dalam batas waktu tertentu tidak mengubah nasib sebuah bangsa
melaporkan harta kekayaannya ke (B) kegiatan membaca akan dapat
KPK, akan dikenakan sanksi yang membangun karakter masyarakat

bimbelocean.com
UTUL UGM 163
PAKET 10
(C) setiap pelajar diharuskan 32. Pemerintah merasa bangga telah
menghasilkan sebuah tulisan pendek memprioritaskan hasil karya sendiri
(D) membaca tidak cukup dijadikan dalam menopang pembangunan
sebagai ajakan atau imbauan konstruksi nasional.
(E) mereka pasti akan menemukan buku Kata yang tepat untuk menggantikan kata
yang tepat memprioritaskan dalam kalimat di atas
adalah ....
29. Akibat vertical speed yang terlalu tinggi, (A) mengunggulkan
pesawat menjadi sulit dikontrol saat roda (B) mendahulukan
hendak menyentuh landasan sehingga (C) memperhatikan
benturan keras terjadi dan pesawat pun (D) mempertahankan
bisa terpental ke luar landasan. (E) menggunakan
Kalimat inti dari kalimat di atas adalah ....
(A) vertical speed terlalu tinggi 33. Insiden pesawat keluar landasan
(B) pesawat sulit dikontrol merupakan kecelakaan yang paling
(C) roda pesawat menyentuh landasan sering terjadi.
(D) terjadi benturan keras Kata yang tepat untuk menggantikan kata
(E) pesawat terpental ke luar landasan insiden dalam kalimat di atas adalah ....
(A) tindakan
30. Dalam Konferensi Internasional (B) kejadian
Kebudayaan Papua itu, tidak diputuskan (C) keadaan
tempat penyelenggaraan konferensi (D) kecelakaan
berikutnya. (E) kenyataan
Kalimat yang memiliki pola yang sama
dengan kalimat di atas adalah .... 34. Meskipun beberapa link yang dituliskan
(A) Untung Subagyo, pejabat yang sudah berkali-kali dicermati, informasi
terpilih itu, mengawali kariernya dari yang diperlukannya belum memadai.
bawah Padanan kata yang tepat untuk kata link
(B) seperti telah kita ketahui, dalam kalimat di atas adalah ....
perekonomian Amerika Serikat (A) pranala
sangat berpengaruh (B) saltik
(C) telah kami bicarakan dalam rapat (C) luring
terdahulu bahwa kenaikan gaji (D) portofan
pegawai baru bisa dilaksanakan (E) warganet
tahun depan
(D) kita sadari bahwa bencana alam Bacalah bacaan berikut dengan cermat,
dapat datang kapan saja tanpa kita kemudian jawablah pertanyaan nomor 15
duga sampai dengan 17.
(E) berdasarkan pengarahan pimpinan,
pembangunan kampus baru itu Ethiopia adalah negara tertua di dunia.
dapat dilakukan secara bertahap Negeri itu telah terbentuk sejak sebelum 4900
tahun yang lalu. Ethiopia terletak di Afrika
31. Kita tidak boleh berpangku tangan Timur, tepatnya di sebelah selatan pintu
melihat dampak negatif media sosial. masuk Laut Merah (Terusan Suez). Negara
Makna ungkapan berpangku tangan tetangganya adalah Kongo, Sudan, Somalia,
dalam kalimat di atas adalah .... dan Djiboti. Wilayah Ethiopia cukup luas,
(A) suka bermalas-malasan hampir dua kali luas Pulau Sumatera. Negeri
(B) tidak berbuat apa-apa ini sering dilanda cuaca buruk dan musim
(C) tidak bergeming kering berkepanjangan. Ethiopia adalah
(D) tidak bekerja negara peternakan kelas 1 dunia. Jumlah sapi
(E) merenung saja dan kudanya menempati urutan ke-10 dunia
dengan 2,6 juta ekor sapi dan 1,5 juta kuda.

bimbelocean.com
164
UTUL UGM
PAKET 10
Ethiopia juga merupakan negara pengekspor
kopi terbesar keenam dunia. Pada tahun 1985, 39. Kata-kata yang dicetak miring dalam
akibat kemarau yang sangat panjang, negeri kalimat berikut yang penulisannya
ini menggemparkan dunia karena dilanda mengikuti ejaan resmi adalah
krisis pangan dan penduduknya menderita (A) Menurut analisa para pakar sistem
kelaparan hebat. itu sudah tidak layak untuk
diterapkan lagi.
35. Ide pokok dalam bacaan di atas adalah (B) Sudah enam tahun ia tidak naik
(A) berdirinya negara Ethiopia. pangkat karena kondite dan
(B) letak negara Ethiopia. perilakunya buruk.
(C) usia negara Ethiopia. (C) Sistem koordinasi antarbagian di
(D) ciri-ciri geografis negara Ethiopia. perusahaan itu sangat baik.
(E) keadaan alam negara Ethiopia. (D) Kongres itu tidak dapat dimulai
karena dutabesar yang membukanya
36. Pernyataan berikut berhubungan, baik belum datang.
secara eksplisit maupun implisit, dengan (E) Jadual keluarnya produk tetap
bacaan di atas, KECUALI ditepati agar tidak mengecewakan
(A) Ethiopia menempati peringkat 10 konsumen.
dunia dalam hal ternak karena
kualitasnya yang baik. 40. Kalimat yang menggunakan ragam baku
(B) Kekeringan merupakan salah satu (A) Di Eropa dan AS terdapat banyak
masalah serius yang dihadapi kesempatan untuk mengembangkan
Ethiopia. ilmu pengetahuan.
(C) Ethiopia merupakan negara (B) Di wilayah kami setiap malam
penghasil kopi dunia. mengadakan koordinasi keamanan.
(D) Krisis pangan yang terjadi di Ethiopia (C) Bagi yang belum mendaftarkan diri
pernah menggemparkan dunia. diharap menghubungi bagian
(E) Jumlah ternak di Ethiopia cukup personalia.
banyak. (D) Di desa yang terletak di pinggir
sungai itu setiap tahun dilanda banjir.
37. Agar hubungan antarkalimat menjadi (E) Dalam pemeriksaan itu
jelas, kata atau ungkapan penghubung menyimpulkan bahwa pembunuhan
yang tepat antara kalimat pertama dan sadis tidak selalu berhubungan
kedua adalah dengan kelainan jiwa.
(A) Dalam hal ini.
(B) Karena.
(C) Menurut catatan sejarah.
(D) Oleh karena itu.
(E) Sebab.

38. Penulisan kata dengan huruf miring pada


kalimat-kalimat di bawah ini benar,
KECUALI
(A) Kata ateis berasal dari
penggabungan a dan teis.
(B) Fonem kedua kata akhir adalah kh.
(C) Perhitungan yang rumit itu dilakukan
dengan kalkulator.
(D) Habis Gelap Terbitlah Terang adalah
buku ciptaan R.A. Kartini.
(E) Dia berlangganan koran The Jakarta
Post.

bimbelocean.com
UTUL UGM 165
PAKET 10

BAHASA INGGRIS

I. Read the text carefully; then choose the best alternative to answer each of the
questions.

The threat to our health from environmental poisons, and specifically heavy metals, is
increasing every day. Lead, mercury, cadmium, arsenic, and copper poisoning are becoming
epidemic. Lead and mercury come mostly from polluted air plus industrial and medical uses,
such as lead- containing paint and mercury-containing amalgam dental fillings. Contaminated
5 fish are also a common source of mercury poisoning. Copper enters our bodies usually from
copper water piper, commonly used in plumbing.
Heavy metal poisoning is difficult to treat. Chelation treatment is the only known
medical treatment that is effective in detoxifying the body of heavy metal. Now, the
Japanese study conducted by Drs Ikezoe and Kitahara, shows that Kyolic, a raw garlic
10 extract developed in Japan, is effective in protecting the body from the toxic effects of heavy
metal poisoning.
Dr. Kitahara and his co-workers, Ikezoe and Yamada, conducted controlled studies
on animals (rabbits) and humans. The method of study was: observation of release of
potassium and hemoglobin by heavy metals from erythocytes, and destruction of the
15 erythrocyte membrane. The conclusion of the study was that garlic preparation prevented
the poisoning effect arising from heavy
Metals and protected the erythrocyte membrane from destruction.
In another study, conducted in Russia, a drug made from garlic extract was given to
workers in industrial plants who were suffering from chronic lead poisoning. The daily doses of
20 garlic improved the symptoms of chronic lead poisoning and lowered the high porphyrin
levels in the urine. The preparation also normalized the elevated blood pressure in the majority
of workers. Russian researchers believe that the efficacy of the garlic preparation is due to
garlic’s high content of sulfur compounds.
Extracted from The Miracle of Garlic by Paavo Airola.

41. The passage tells us about which of the (A) Chelation treatment is not the
following? only possible method to protect our
body from heavy metal poisoning.
(A) The problem of heavy metals in (B) The only thing we can do to protect
everyday life our body from metal poisoning is by
(B) The increasing number of heavy way of Chelation treatment.
metals (C) Chelation treatment is the only
(C) The threat of heavy metals to our possible medical treatment to detoxify
health the body of heavy metals.
(D) Heavy metal poisoning and garlic (D) We can do nothing to protect our
(E) Environmental poisons body from heavy metal poisoning.
(E) Kyolic is the only possible
42. It can be inferred that motorbike treatment to detoxify our body from
riders without masks may inhale heavy metal poisoning.

(A) copper 44. The word “contaminated” in line 5 can


(B) cadmium best be replaced by
(C) lead
(D) arsenic (A) poisoned
(E) amalgam (B) polluted
(C) rotten
43. Which of the following is TRUE (D) dead
according to the passage? (E) unhealthy

bimbelocean.com
166
UTUL UGM
PAKET 10
45. Which of the following is TRUE 48. According to the passage, the use of garlic
according to the passage?
(A) may increase blood pressure
(A) Drs. kezoe and Kitahara developed a (B) may decrease high blood pressure
raw garlic extract. (C) regulates blood pressure
(B) Drs. Ikezoe and Kitahara were (D) maintain normal blood pressure
probably not the persons who (E) is good to elevate blood pressure
developed Kyolic.
(C) It was Dr. Kitahara and his co- 49. The word “efficacy” in line 25 is closest in
workers who developed Kyolic. meaning to which of the following?
(D) Chelation treatment was developed
by Drs. Ikezoe and Kitahara. (A) effectiveness
(E) Both Chelation treatment and (B) use
Kyolic originated from Japan. (C) consumption
(D) result
46. Which of the following is TRUE according (E) strength
to the pasaage?
50. What makes Russian researchers
(A) The erythrocyte membrane is recommend the use of garlic is that it
important to protect us from heavy
metal poisoning. (A) is inexpensive
(B) Heavy metals may harm the (B) is available everywhere
erythrocyte membrane. (C) is suitable for eating
(C) The damage of the (D) contains certain chemical compounds
erythrocyte membrane is harmful to (E) has a lot of functions
human beings.
(D) The erythrocyte membrane is immune
to heavy metals.
(E) The erythrocyte membrane
is indestructible.

47. Who reported the use of garlic to help


workers in industrial plants suffering from
chronic lead poisoning?
(A) The Japanese government
(B) Drs. Ikezoe and Kitahara
(C) Dr Kitahara, Ikezoe and Yamada
(D) The Russian government
(E) Russian researchers

bimbelocean.com
UTUL UGM 167
PAKET 10

II. Read the following paragraph and find the appropriate word to fill in each of the blank
Torajans believe that their ancestors originated from a marriage of people from the
heaven and sea, creating their offspring together with unique culture of Aluk Todolo.
That is the folklore version; more contemporary anthropological research reveals that
ancestors of Torajans came from Indochina and belong (51)_ _ proto Malays, like
the Batak of North Sumatra and the Dayak of Kalimantan, (52) ______ migrated to
the land many centuries ago.
The number of Torajans adhering to the Aluk Todolo tradition. (53) _____, continues to
decrease and is now estimated at only 5 percent of the 400.000 population. Although most
Torajans (54) ______Christianity, many Torajans Christians still perform traditional rituals,
especially (55) ______for death and marriage.
Major parts of Aluk Todolo are based on the belief (56) _____ is about upholding its
principles, especially ancestor worship, in accordance with the time-honoured adat (customes).
For Torajans, the world is the place to see goodness, (57) _____ people fulfill all
obligations and responsibilities to the community, to live (58) ____ until death and the transition
to the spirit world.
The final purpose of life (59) ______ for death, so that when one dies the
customary ceremonies reflect the (60) ______ efforts during the ephemeral world.
Extracted from The Jakarta Post, 30 Nov. 2004.

51. (A) in 56 (A) that life


(B) with (B) of life
(C) from (C) that this life
(D) into (D) life
(E) to (E) of life that

52. (A) who 57. (A) which


(B) that (B) in that
(C) which (C) when
(D) whom (D) where
(E) whose (E) for which

53. (A) however 58 (A) content


(B) in addition (B) contented
(C) furthermore (C) contending
(D) accordingly (D) to content
(E) in fact (E) contentedly

54 (A) converted with 59. (A) is prepared to


(B) have to convert (B) is to prepare
(C) are converted to (C) prepares to be
(D) have been converted (D) is prepared to be
(E) have converted to (E) is to be preparing

55. (A) that 60. (A) deceased


(B) these (B) deceasing
(C) those (C) deceased’s
(D) them (D) decease
(E) theirs (E) deceases

bimbelocean.com
168
UTUL UGM

Anda mungkin juga menyukai